review questions for human histology

438

Upload: joeguy69

Post on 16-Oct-2014

771 views

Category:

Documents


21 download

TRANSCRIPT

Page 1: Review Questions for Human Histology
Page 2: Review Questions for Human Histology

Review Questions for

HUMAN HISTOLOGY

Page 3: Review Questions for Human Histology

Review Questions for

HUMAN HISTOLOGY

by

E. Robert Burns, PhD

Professor, Department of Anatomy

University of Arkansas for Medical Sciences

Review Questions Series

Series Editor: Thomas R. Gest, PhD

University of Arkansas for Medical Sciences

Page 4: Review Questions for Human Histology

AcknowledgmentsThe freshman medical students in Microscopic Anatomy in the College of Medicineat the University of Arkansas for Medical Sciences from 1968 to 1994 helped me todifferentiate into the teacher/author I am. The freshmen medical students at UAMSfrom 1995 to ? most certainly will continue to challenge and change me.

Horace N. Marvin, Ph.D., Professor Emeritus of Anatomy (deceased): From him Iam still learning about teaching, students, histology, embryology, administration,education and life.

My children, Keith and Sherry, two mature, well educated, motivated, hard workingand honest "kids" are a source of continuing strength and pride to me.

Mary Stuart Lindsey provides a source of energy, guidance, advice, intellect,motivation, support, joy and compassion which has been of immeasurable wealthto me.

John P. Shock, M.D., Chairman of Ophthalmology at UAMS was bold enough andso clinically proficient and caring that darkness was not allowed to preventcompletion of this and other projects.

Thomas R. Gest, Ph.D., Series Editor, trusted in my competence and asked me toundertake this project; I thank him for that and his constant help.

E. Robert Burns, Ph.D.

Dedicated to my parents

to my dad who demonstrated that with hard work, a strong sense of fairness, nativeintelligence, impeccable honesty and only a high school education one couldbegin as a toll collector and become The Executive Administrative Officer of theNew York State Bridge Authority

to my mom with her intense interest in everything and everyone around her; withspecial devotion to her two sons and her husband, achieved a place in life fewattain but many admire

Page 5: Review Questions for Human Histology

PrefaceThis book is designed to present students who have completed a course inMicroscopic Anatomy (i.e. Cell Biology, Histology, Organology) with a review ofthis subject matter from a structure-function point of view. Using this review as afirst exposure to the material will not be an effective learning situation. It should beused first as a personal assessment of command of the material, then as a tool toreview and refine understanding of the subject.

I recommend the user of this book first cover up the answer and its minidescription.Second, work the question to the best of your ability. Third, uncover just the correctanswer and rework the question in light of the correct answer. Lastly, uncover thehints supporting the correct answer and integrate all steps into an understanding ofthe subject material involved in the question.

Most of the questions are of the type A or single best answer. Some of thequestions are of the relatively new extended match type. Both types are used onthe National Board Exams.

The cellular basis of tissues, organs, organ systems and organisms is offundamental importance to all medicine and biology. Permit me to say that I neversee a dead, fixed, stained cell. I always see a living, functioning cell. I challengemyself to correlate its structure and its function with its role in the body economy.The fact is that most of the practice of medicine deals with cells that have too muchor too little or no functional capabilities. In a variety of ways, doctoring is an attemptto fix sick cells.

Good luck!

Page 6: Review Questions for Human Histology

Contents

1. Techniques (42) ................................................................................ 1

2. Cell Biology (150) ............................................................................. 8

3. Epithelial Tissue (52) ....................................................................... 48

4. Connective Tissue (49) .................................................................... 59

5. Muscle Tissue (50) .......................................................................... 68

6. Nervous Tissue (40) ........................................................................ 77

7. Integument (30) .............................................................................. 84

8. Cartilage and Bone (37) .................................................................. 90

9. Blood and Bone Marrow (57) ......................................................... 97

10. Defense System (40) ......................................................................109

11. Cardiovascular System (65) ............................................................ 117

12. Respiratory System (40) .................................................................128

13. Gastrointestinal System (78) ........................................................... 137

14. Urinary System (42) .......................................................................151

15. Endocrine System (62) ...................................................................159

16. Male Reproductive System (66) ..................................................... 170

17. Female Reproductive System (63) .................................................. 182

18. Eye and Ear (79) ............................................................................194

Number in parentheses indicates number of questions available.

Page 7: Review Questions for Human Histology

SECTION 1: TECHNIQUES

1.001 A tissue is put through the following solutions or chemicals: glutaraldehyde,100% alcohol, osmium tetroxide and uranyl acetate. Which technique is beingused?

A. immunocytochemistry

B. autoradiography

C. phase contrast microscopy

D. transmission electron microscopy

E. polarizing microscopy

1.002 A chemical constituent of cells in a tissue is bound to unlabeled rabbitantibody which is then bound to labeled anti-rabbit, goat antibody. What techniqueis being used?

A. direct immunocytochemistry

B. indirect immunocytochemistry

C. autoradiography for LM

D. autoradiography for EM

E. in situ hybridization

1.003 An embedded block of tissue is sectioned on a microtome. Which of thefollowing techniques does NOT apply?

A. transmission electron microscopy

B. phase contrast microscopy

C. autoradiography

D. immunocytochemistry

E. histochemistry

1.004 Reduced silver bromide particles are seen over the nucleus. Which of thefollowing correctly applies?

Page 8: Review Questions for Human Histology

A. immunocytochemistry

B. cryofracture study of the faces of unit membranes

C. scanning electron microscopy

D. autoradiography

E. polarizing microscopy

1.005 All of the following have been used to label antibodies duringimmunocytochemistry EXCEPT:

A. fluorescein isothiocyanate

B. peroxidase

C. gold

D. ferritin

E. tritium

D. is correct.

The solutions mentioned are part of the procedural steps used in preparing tissuesfor regular or transmission electron microscopy.

B. is correct.

Direct immunocytochemistry is when the antibody to the cellular antigen inquestion is tagged or labeled. Indirect immunocytochemistry is when the cellularantigen in question is found or located by an antibody from one species such asthe rabbit, but the tag or label is attached to a second antibody raised in a differentspecies such as goat. The labeled goat antibody is used to find the rabbit antibodywhich was initially used to find the cellular constituent under investigation.

B. is correct.

Phase contrast microscopy studies living cells, not fixed and stained cells.

D. is correct.

The radioactivity of the isotope irradiates the silver bromide particles located in thephotographic emulsion only over the region of the cell which incorporated theradioactive isotope, which in this case was probably tritiated thymidine or uridine,

Page 9: Review Questions for Human Histology

precursors of DNA or RNA synthesis respectively.

E. is correct.

Tritium is a radioisotope form of hydrogen which is bound to precursor moleculessuch as uridine, thymidine, amino acids, etc. for use in both LM and EMautoradiography. All other choices listed can be used to label antibodies forimmunocytochemistry.

1.006 Which of the following usually is not isolated and purified by cellfractionation/differential centrifugation?

A. nuclei

B. ribosomes

C. mitochondria

D. collagen fibers

E. microsomes

1.007 Ultraviolet light is the usual source of illumination for:

A. brightfield microscopy

B. darkfield microscopy

C. phase contrast microscopy

D. polarizing light microscopy

E. fluorescence microscopy

F. transmission electron microscopy

G. scanning electron microscopy

1.008 The following is used with H & E stained specimens:

A. brightfield microscopy

B. darkfield microscopy

C. phase contrast microscopy

D. polarizing light microscopy

Page 10: Review Questions for Human Histology

E. fluorescence microscopy

F. transmission electron microscopy

G. scanning electron microscopy

1.009 An opaque disc is placed in the center of the condenser so that a hollowcone of light illuminates the specimen and no light is transmitted directly throughthe specimen for:

A. brightfield microscopy

B. darkfield microscopy

C. phase contrast microscopy

D. polarizing light microscopy

E. fluorescence microscopy

F. transmission electron microscopy

G. scanning electron microscopy

1.010 A plate coated with a transparent film which has a refractive index and athickness sufficient to delay the normal refracted rays by an additional one-fourthwavelength is used in:

A. brightfield microscopy

B. darkfield microscopy

C. phase contrast microscopy

D. polarizing light microscopy

E. fluorescence microscopy

F. transmission electron microscopy

G. scanning electron microscopy

D. is correct.

Collagen fibers do not require cell fractionation to isolate. All others can be isolatedby rupturing cells and centrifuging the homogenate at different speeds for different

Page 11: Review Questions for Human Histology

lengths of time.

E. is correct.

The fluorescence microscope detects light emitted by fluorescent moleculestagging specific components in the specimen. All light wavelengths are filtered outexcept for the wavelength best absorbed by the dye, which, when bombarded bythe wavelength it best absorbs, releases light at a different but characteristicwavelength. Much fluorescence microscopy uses a UV light source andfluorescent dyes that absorb light in the UV range.

A. is correct.

Here light rays from a bulb or the sun are focused on the specimen by a condenserlens. The difference in the absorption of these light rays is enhanced greatly bydifferential staining of the chemical constituents of the specimen.

B. is correct.

In darkfield microscopy the opaque disc forces light to pass around its edges, thus,a hollow cone of light illuminates the specimen. In this way only light scattered tocertain angles by the substructure within the specimen is allowed to enter theobjective lens. This makes certain points or objects appear bright against the darkbackground.

C. is correct.

In the phase contrast microscope living cells can be visualized with somesignificant differences in contrast between the components of their substructurebecause the normal differences in the phase of the wavelength of light exitingstructures is enhanced from one-fourth wavelength to one-half wavelength. Thisadditional difference in phase is obtained by placing a phase plate in the path ofsome of the more refracted rays coming from the specimen.

1.011 The specimen is viewed in light vibrating in only one plane in which of thefollowing?

A. brightfield microscopy

B. darkfield microscopy

C. phase contrast microscopy

D. polarizing light microscopy

E. fluorescence microscopy

Page 12: Review Questions for Human Histology

F. transmission electron microscopy

G. scanning electron microscopy

1.012 A focused beam does not remain stationary but moves rapidly back and forthover the specimen in which of the following?

A. brightfield microscopy

B. darkfield microscopy

C. phase contrast microscopy

D. polarizing light microscopy

E. fluorescence microscopy

F. transmission electron microscopy

G. scanning electron microscopy

1.013 The image is formed by energy released in the form of high-energysecondary or scattered electrons reflected from a surface which has been coatedwith a thin film of heavy metal in:

A. brightfield microscopy

B. darkfield microscopy

C. phase contrast microscopy

D. polarizing light microscopy

E. fluorescence microscopy

F. transmission electron microscopy

G. scanning electron microscopy

1.014 The "stain" is a tagged antibody in which of the following?

A. brightfield microscopy

B. darkfield microscopy

C. phase contrast microscopy

Page 13: Review Questions for Human Histology

D. polarizing light microscopy

E. fluorescence microscopy

F. transmission electron microscopy

G. scanning electron microscopy

1.015 The scattering of primary electrons produces contrast in which of thefollowing?

A. brightfield microscopy

B. darkfield microscopy

C. phase contrast microscopy

D. polarizing light microscopy

E. fluorescence microscopy

F. transmission electron microscopy

G. scanning electron microscopy

D. is correct.

In brightfield microscopy the light source contains waves vibrating in many differentplanes. In the polarizing microscope the specimen is viewed using light vibrating inonly one plane. A filter is placed between the light source and the specimen whichblocks all waves except for those vibrating in one particular plane. Light filtered inthis way is polarized.

G. is correct.

In all other kinds of microscopy the illuminating beam is kept in a constant location.However, in scanning electron microscopy the properly prepared specimen isactually scanned with a beam of electrons. The reflected electrons are captured bya detector and displayed as a three dimensional image.

G. is correct.

This is a description of the technique called scanning electron microscopy. Theimage is formed from the secondary electrons scattered from the different regionsof the specimen rather than from the scattering of primary or "illuminating"electrons.

Page 14: Review Questions for Human Histology

E. is correct.

Antibody to a specific substance, such as myosin or actin, raised in a rabbit can beattached (tagged) to a fluorescein molecule which can be activated to fluorescewith the ultraviolet microscope. In this way the antigen used to raise the antibodycan be detected and localized within cells, tissue and organs.

F. is correct.

The heavy metal staining of thin sections will scatter the electrons as they passthrough the different densities of stain, producing good contrast between theseareas. This is different from the contrast produced by the different absorption oflight by different structures as is the case of brightfield microscopy. In TEM it is thescattering of electrons, not their absorption, that produces the contrast.

1.016 What centrifugal force is necessary to sediment mitochondria in a centrifugeafter disruption of a cell?

A. 1000g

B. 20,000g

C. 300,000g

1.017 What centrifugal force is necessary to sediment nuclei in a centrifuge afterdisruption of a cell?

A. 1000g

B. 20,000g

C. 300,000g

1.018 What centrifugal force is necessary to sediment ribosomes in a centrifugeafter disruption of a cell?

A. 1000g

B. 20,000g

C. 300,000g

1.019 What centrifugal force is necessary to sediment lysosomes in a centrifugeafter disruption of a cell?

A. 1000g

Page 15: Review Questions for Human Histology

B. 20,000g

C. 300,000g

1.020 What centrifugal force is necessary to sediment virus particles in a centrifugeafter disruption of a cell?

A. 1000g

B. 20,000g

C. 300,000g

1.021 Which of the following would be used to visualize that the cytoplasm of aplasma cell stains basophilic?

A. a compound light microscope

B. a fluorescent light microscope

C. an electron microscope (SEM or TEM)

D. a biochemical or non-microscopic technique

1.022 Which of the following would be used to visualize the cytoskeletal actinmicrofilament arrangement in cultured cells?

A. a compound light microscope

B. a fluorescent light microscope

C. an electron microscope (SEM or TEM)

D. a biochemical or non-microscopic technique

1.023 Which of the following would be used to localize cell surface receptors forestrogen?

A. a compound light microscope

B. a fluorescent light microscope

C. an electron microscope (SEM or TEM)

D. a biochemical or non-microscopic technique

B. is correct.

Page 16: Review Questions for Human Histology

Mitochondria can be sedimented using about 20,000g.

A. is correct.

Nuclei are so large that they will sediment with a minimal amount of centrifugalforce. Intact cells would also sediment with 1000g.

C. is correct.

Ribosomes are very small particles and therefore would require the highest g-forceto sediment them.

B. is correct.

Lysosomes are relatively "big" particles and they can be sedimented using about20,000g in a centrifuge.

C. is correct.

Since virus particles are so small the highest centrifugal force would be necessaryto "bring them down".

A. is correct.

This is routinely accomplished using the light microscope.

B. is correct.

The use of fluorescein tagged monoclonal antibodies to the different chemicalconstituents of the microfilaments and intermediate filaments comprising thecytoskeleton of the cell is an entire science by itself.

B. is correct.

Fluorescein tagged monoclonal antibodies to these receptors can be used todetect these receptors on the cell surface. Of course, many cell surface receptorscan be studied by biochemical means, but to localize them to the cell surface andon which cells is best accomplished with the fluorescent microscope.

1.024 Which of the following would be used to obtain a 3D view of the cell surface?

A. a compound light microscope

B. a fluorescent light microscope

C. an electron microscope (SEM or TEM)

Page 17: Review Questions for Human Histology

D. a biochemical or non-microscopic technique

1.025 Which of the following would demonstrate that the major basic protein of aneosinophil stains red?

A. a compound light microscope

B. a fluorescent light microscope

C. an electron microscope (SEM or TEM)

D. a biochemical or non-microscopic technique

1.026 Which of the following would be used to demonstrate that chromosomesattract hematoxylin?

A. a compound light microscope

B. a fluorescent light microscope

C. an electron microscope (SEM or TEM)

D. a biochemical or non-microscopic technique

1.027 Which of the following would demonstrate the hexagon of actin filamentsseen surrounding each myosin molecule in a cross-section of skeletal muscle?

A. a compound light microscope

B. a fluorescent light microscope

C. an electron microscope (SEM or TEM)

D. a biochemical or non-microscopic technique

1.028 Which of the following would be used to demonstrate pinocytotic vesicles inendothelial cytoplasm?

A. a compound light microscope

B. a fluorescent light microscope

C. an electron microscope (SEM or TEM)

D. a biochemical or non-microscopic technique

1.029 Which of the following would be used to demonstrate that the specific

Page 18: Review Questions for Human Histology

granules of a neutrophil fuse with ingested E. coli?

A. a compound light microscope

B. a fluorescent light microscope

C. an electron microscope (SEM or TEM)

D. a biochemical or non-microscopic technique

1.030 Which of the following would be used to demonstrate that mitochondrialDNA is circular?

A. a compound light microscope

B. a fluorescent light microscope

C. an electron microscope (SEM or TEM)

D. a biochemical or non-microscopic technique

C. is correct.

Although phase contrast-light microscopy might be able to do this with limitedresolution, the structure of the cell surface can be easily studied under a variety ofexperimental situations using the scanning electron microscope.

A. is correct.

This is easily accomplished with the light microscope.

A. is correct.

This is a standard function of the light microscope and H & E stained specimens.

C. is correct.

The electron microscope has been used to see this ultrastructural arrangement.Although actin and myosin can be localized within cells using fluorescentmicroscopy, this technique would not be able to resolve the hexagon of thinfilaments surrounding each thick filament.

C. is correct.

This is a common finding seen in electron micrographs.

C. is correct.

Page 19: Review Questions for Human Histology

The best tool would be the electron microscope. This biological phenomenon hasbeen observed many times using this technique.

C. is correct.

First a biochemical isolation of the mitochondrial DNA is done and then the DNA isspread on a platform and studied with an electron microscope. In this way it wasdiscovered that mitochondrial DNA is circular.

1.031 Which of the following would be used to demonstrate the Golgi apparatus asa light area in the cytoplasm of a plasma cell?

A. a compound light microscope

B. a fluorescent light microscope

C. an electron microscope (SEM or TEM)

D. a biochemical or non-microscopic technique

1.032 Which of the following would be used to demonstrate synaptic vesiclesexocytosing their contents into a synaptic cleft?

A. a compound light microscope

B. a fluorescent light microscope

C. an electron microscope (SEM or TEM)

D. a biochemical or non-microscopic technique

1.033 Which of the following would be used to visualize a messenger RNAmolecule in a polysome?

A. a compound light microscope

B. a fluorescent light microscope

C. an electron microscope (SEM or TEM)

D. a biochemical or non-microscopic technique

1.034 Which of the following would be used to demonstrate a trisomy 21karyotype?

A. a compound light microscope

Page 20: Review Questions for Human Histology

B. a fluorescent light microscope

C. an electron microscope (SEM or TEM)

D. a biochemical or non-microscopic technique

1.035 Which of the following would be used to distinguish CD4 lymphocytes fromCD8 lymphocytes?

A. a compound light microscope

B. a fluorescent light microscope

C. an electron microscope (SEM or TEM)

D. a biochemical or non-microscopic technique

1.036 Which of the following would be used to demonstrate elementary particleson the inner mitochondrial membrane?

A. a compound light microscope

B. a fluorescent light microscope

C. an electron microscope (SEM or TEM)

D. a biochemical or non-microscopic technique

A. is correct.

The "hof" area of the plasma cell is a light microscopic description of the light ornon-staining (H&E) area in which the Golgi apparatus is found.

C. is correct.

This is easily within the realm of the electron microscope, but outside the realm ofthe light microscope.

C. is correct.

This can be done with high power electron microscopy.

A. is correct.

Trisomy 21 and many other numerical chromosomal abnormalities can be detectedusing the light microscope.

Page 21: Review Questions for Human Histology

B. is correct.

Monoclonal antibodies exist for the T helper and T suppresser surface antigens.These antibodies are tagged with a fluorescein and tissue sections can be stainedwith these monoclonal "stains". In fact, this technique has been used to map thetissue/organ locations of many of the different types of lymphocytes, especially thedistribution of T cell and B cells.

C. is correct.

The elementary particles studding the mitochondrial cristae have been visualizedwith the electron microscope, especially when used in conjunction with negativestaining where the background is stained but not the object in question.

1.037 Which of the following would be used to demonstrate the signal sequenceon the end of a mRNA molecule?

A. a compound light microscope

B. a fluorescent light microscope

C. an electron microscope (SEM or TEM)

D. a biochemical or non-microscopic technique

1.038 Which of the following would be used to demonstrate that lipid in thecytoplasm of a cell is absorbed vs. synthesized de novo?

A. a compound light microscope

B. a fluorescent light microscope

C. an electron microscope (SEM or TEM)

D. a biochemical or non-microscopic technique

1.039 Which of the following would be used to identify an unknown intermediatefilament as a desmin containing intermediate filament?

A. a compound light microscope

B. a fluorescent light microscope

C. an electron microscope (SEM or TEM)

D. a biochemical or non-microscopic technique

Page 22: Review Questions for Human Histology

1.040 Which of the following would be used to demonstrate theheterochromatinized X chromosome in a neutrophil of a female?

A. a compound light microscope

B. a fluorescent light microscope

C. an electron microscope (SEM or TEM)

D. a biochemical or non-microscopic technique

1.041 Which of the following would be used to demonstrate the 64 nanometerperiodicity of a collagen fiber?

A. a compound light microscope

B. a fluorescent light microscope

C. an electron microscope (SEM or TEM)

D. a biochemical or non-microscopic technique

1.042 Which of the following would be used to demonstrate the conversion of ATPto ADP during muscle contraction?

A. a compound light microscope

B. a fluorescent light microscope

C. an electron microscope (SEM or TEM)

D. a biochemical or non-microscopic technique

D. is correct.

This can only be detected by biochemical techniques. No microscopy is powerfulenough to see the signal sequence on a mRNA molecule. However, high powerelectron microscopy can resolve individual mRNA molecules.

C. is correct.

The electron microscope would be able to resolve the presence or absence of aunit membrane surrounding the lipid droplet. If no membrane surrounds the lipid, itwas not taken in by endocytosis but synthesized de novo in the cytoplasm.

B. is correct.

Page 23: Review Questions for Human Histology

Using a monoclonal antibody against desmin tagged with fluorescein is thestandard technique to detect any of the different types of intermediate filaments.

A. is correct.

The light microscope will easily resolve the "drumstick" or heterochromatinized Xchromosome which can be seen in about 3% of the PMNs in a female.

C. is correct.

The electron microscope would be the technique to use to see the periodicity ofcollagen.

D. is correct.

This kind of reaction is a biochemical one and can not be detected by anymicroscopic technique.

Page 24: Review Questions for Human Histology

SECTION 2: CELL BIOLOGY

2.001 You see a cell on a slide stained with H&E. The cell has intensely basophiliccytoplasm. All of the following correctly apply to this situation EXCEPT:

A. extensive RER

B. extensive SER

C. high concentration of polysomes

D. hematoxylin is basic in aqueous solution

E. eosin is acidic in aqueous solution

2.002 All of the following are found on or are associated with the E or externalsurface of the plasmalemma EXCEPT:

A. glycocalyx

B. peripheral proteins

C. receptors for hormones

D. hydrophilic regions of phospholipids

E. hydrophobic regions of phospholipids

2.003 In freeze-fracture electron microscopy of the plasmalemma most of thegranules seen are on/in which of the following.

A. E face

B. P face

C. E surface

D. P surface

E. glyocalyx

2.004 A substance enters a cell with the result that the concentration of thesubstance inside the cell is much higher than outside the cell. Energy wasexpended during this process. Which of the following apply?

Page 25: Review Questions for Human Histology

A. clathrin coated pit

B. carrier mediated or facilitated diffusion

C. active transport

D. endocytosis

E. simple diffusion

2.005 Which of the following enzymes is found in all lysosomes?

A. DNAse

B. acid phosphatase

C. lipase

D. catalase

E. cathepsin

B. is correct.

The SER would not have any significant amounts of RNA associated with it whichwould attract the hematoxylin. The RNA could be ribosomal RNA associated withmembrane of the RER or with strands of mRNA forming polysomes.

E. is correct.

The hydrophobic ends of the amphipathic phospholipids of the plasmalemma facetoward the inner region of the membrane whereas the hydrophilic regions of thephospholipids are located at either surface of the membrane.

B. is correct.

The P face is more granular than the E face. The granules seen on both facesrepresent integral or intrinsic membrane proteins.

C. is correct.

The substance enters the cell through the cell membrane by carrier mediatedtransport which used energy and therefore is active transport which can movesubstances against a concentration gradient.

B. is correct.

Page 26: Review Questions for Human Histology

Acid phosphatase is found in all lysosomes. It is a "marker" enzyme for thelysosome. Catalase, for example, is a marker enzyme for the peroxisome.

2.006 Which of the following cells in a 70 yr. old man would show the higherconcentration of residual bodies on EM and lipofuscin granules on LM?

A. enterocytes on tip of intestinal villus

B. myeloblasts

C. cerebellar neurons

D. fibroblasts in a healing wound

E. goblet cells in epithelial lining of trachea

2.007 Which of the following is related to a defect in lysosomal metabolism?

A. autophagy

B. hemosiderin pigment in macrophages

C. heterophagy

D. intracellular glycogen storage due to absence of glucosidase

E. silicosis

2.008 In a rapidly growing, undifferentiated tumor cell population one would expectto see all of the following using the EM EXCEPT:

A. extensive RER

B prominent nucleolus

C. lots of polysomes

D. many microtubules

E. chromosomes

2.009 Which of the following is mainly responsible for ribosomes attaching to themembrane of the endoplasmic reticulum?

A. mRNA molecule

B. signal sequence in forming polypeptide

Page 27: Review Questions for Human Histology

C. tRNA molecules

D. circular DNA molecules

E. condensing vacuoles

2.010 Where in a mitochondrion is ADP combined with inorganic phosphate toform ATP using some energy derived from proton reflux?

A. dense granules in the matrix

B. elementary particles on the inner surface of the cristae

C. outer mitochondrial membrane

D. intermembranous space

E. intracristal space

C. is correct.

Residual bodies/lipofuscin granules represent undigested lysosomal material thathas accumulated in cells which do not divide and live for a very long time. Thecardiac myocyte would be another good place to find residual bodies/liposfuscingranules.

D. is correct.

Glycogen storage disease type II(Pompe's disease) is caused by a genetic defectin which no glucosidase is made. Glucosidase breaks down glycogen to glucose.Many clinical diseases are caused by genetic defects in which just one enzyme ismissing.

A. is correct.

Rapidly growing, undifferentiated tumor cells would have all of the above except awell developed RER. These cells are busy growing and dividing and thereforewould not be expected to be involved in making protein for extracellular use.

B. is correct.

As a polysome begins to synthesize a polypeptide which is destined to enter thecisternae of the RER the initial amino acid sequence on the polypeptide contains asignal(signal sequence) which is recognized by a signal recognition particle whichdocks with a receptor in the membrane of the endoplasmic reticulum.

Page 28: Review Questions for Human Histology

B. is correct.

The elementary particles or globular units which "stud" the inner mitochondrialmembrane are the sites of the phosphorylating system.

2.011 Mitochondrial DNA carries the code for which of the following?

A. all species of mitochondrial transfer RNA

B. most of the enzymes found in the mitochondrion

C. most of the structural proteins of the organelle

D. ribosomal RNA in the small subunit of ribosomes attached to the RER

E. nucleolar organizing region

2.012 During the production of a glycoprotein where in the cell does the final orterminal glycosylation take place?

A. cisternae of the rough endoplasmic reticulum

B. transporting vesicles approaching the immature face of the Golgi apparatus

C. in the cisternae of the Golgi apparatus near its mature face

D. in the condensing vacuoles budding off from the mature face of the Golgiapparatus

E. in a coated pit near the cell surface

2.013 The protein dynein has ATPase activity. Where is dynein found?

A. in basal bodies

B. in centrioles

C. in cilia and flagella

D. in microvilli

E. in mitochondria

2.014 Sometimes a pathologist can not diagnose the tissue of origin of a veryundifferentiated, malignant neoplasm using routine H & E preparations. To obtainsome data on the possible tissue of origin of the cancer the cancer cells will bestained for the chemical composition of their intermediate filaments. If a cancer cell

Page 29: Review Questions for Human Histology

stains positively with immunocytochemistry for desmin, the pathologist can berelatively certain that the cell or tissue of origin of the neoplasm is?

A. astrocyte

B. neuron

C. bone marrow

D. epidermis

E. muscle

2.015 In an electron microscopic autoradiograph prepared after a short labelingperiod with tritiated uridine, a precursor for RNA, where would one expect to seelabel?

A. over euchromatin

B. over heterochromatin

C. over the Barr body or sex chromatin

D. over the perinuclear cistema

E. over the Golgi apparatus

A. is correct.

Most of the proteins found in the mitochondrion are coded for by nuclear DNA.Mitochondrial DNA does code for all of the transfer RNA molecules found in theorganelle as well as for mitochondrial ribosomal RNA and a few subunits ofrespiratory chain enzymes

C. is correct.

The initial glycosylation occurs in the endoplasmic reticulum. The terminalglycosylation takes place in the Golgi cisternae from the midway point to themature face.

C. is correct.

Dynein is found in the dynein arms which extend from the A microtubule toward theB microtubule in a cilium or flagellum. Centrioles and basal bodies do not havedynein arms, they have three associated microtubules instead of the doubletarrangements characteristic of cilia and flagella.

Page 30: Review Questions for Human Histology

E. is correct.

Desmin, also known as skeletin, is a microfilamentous protein found in musclecells. One of the keratins would be characteristic of epidermis and neurofilamentsor glial fibrillary protein would be detected in cells found in nervous tissue.

A. is correct.

Euchromatin is the part of the genome "open" for mRNA synthesis. All otherregions listed are not areas of mRNA synthesis and therefore, would notincorporate the tritiated uridine.

2.016 How many Barr bodies would you expect to see in an individual with thefollowing chromosome constitution: XXXXY?

A. 1

B 2

C. 3

D. 4

E. 5

2.017 What is a nucleosome?

A. the region of the nucleolus with linear RNA

B. the region of the nucleolus with granular RNA

C. the region of the nucleolus which contains the nucleolar organizer DNA

D. the region of the DNA molecule that is wrapped around 4 different types ofhistone molecules

E. another name for a heterochromatinized X chromosome

2.018 When does the nuclear membrane temporarily disappear?

A. during early prophase

B. at the end of prophase

C. just as the kinetochores separate in early anaphase

D. between GI and G2 phases of the cell cycle

Page 31: Review Questions for Human Histology

E. during cytokinesis events in late telophase

2.019 If the gamete amount of DNA is 1.5 picograms, how much DNA in picogramsis there in a cell in metaphase of mitosis?

A. 1.5

B. 3.0

C. 4.5

D. 6.0

E. 0.75

2.020 If the gamete amount of DNA is 1.5 picograms, how much DNA in picogramsis there in a cell in metaphase II of meiosis?

A. 1.5

B. 3.0

C. 4.5

D. 6.0

E. 0.75

C. is correct.

The rule is that only one X chromosome needs to remain functional. All other Xchromosomes in a cell are then heterochromatinized. Thus the sex chromatin orBarr body count is one less than the number of X chromosomes in the cell.

D. is correct.

The nucleosome is that part of the DNA which is wrapped around histones giving abeads-on-the-string appearance.

B. is correct.

The disappearance of the nuclear membrane is the official end of the prophasestage of mitosis. After the nuclear membrane disappears there is a short time spanbefore the chromosomes arrange themselves on the equatorial plate in metaphase.This short time span between the end of prophase and metaphase isprometaphase.

Page 32: Review Questions for Human Histology

D. is correct.

Two gametes, the sperm and the egg, must first meet during fertilization to form adiploid cell which in this case would contain 3.0 picograms of DNA in the G1phase of the cell cycle. A cell in metaphase of mitosis has passed through the Sphase of the cell cycle and would have 6.0 picograms of DNA, half of which wouldgo to each pole of the future daughter cell.

B. is correct.

Meiosis I is the reductional division in which the number of chromosomes, thenumber of chromatids and the concentration or C amount of DNA is halved. Thusthe secondary spermatocyte in metaphase II of meiosis would have 3.0 picogramsof DNA which soon would be parceled out to two daughter cells, the gametes,each with 1.5 picograms of DNA.

2.021 How many sex chromosomes are there in a secondary spermatocyte?

A. none

B. one

C. two

D. four

E. twenty three

2.022 Cells have different cell lives with respect to their capability, or lack of it, toprogress through the various stages of the cell cycle. Which of the followingassociations is INCORRECT for a 42 year old man?

A. stem cell - vegetative intermitotic

B. neuron - occasional replicator

C. hepatocyte - reverting postmitotic

D. red blood cell - fixed postmitotic

E. plasma cell - nonreplicator

2.023 Which of the following sequences is correct from youngest to oldest withregard to the manufacturing of ribosomes: 1=ribosome on endoplasmic reticulum;2=455 rRNA molecule; 3=nucleolar organizer DNA; 4=18S & 28S rRNAmolecules; 5 =protein is added to rRNA to form 40S and 60S particles:

Page 33: Review Questions for Human Histology

A. 1-2-34-5

B. 5-4-3-2-1

C. 3-2-4-5-1

D. 4-1-3-5-1

E. 2-4-3-1-5

2.024 When this factor becomes activated the following events occur: mitoticspindle assembly, chromosome condensation and nuclear membrane breakdown.Identify the factor.

A. cyclin

B. maturation promoting factor or MPF

C. nuclear lamin

D. epidermal growth factor

E. tumor angiogenesis factor

B. is correct.

During meiosis I the chromosome pairs undergo disjunction after having paired upto form tetrad formations during prophase I. Thus one member of each pair goes toeach daughter cell. Half of a pair of two sex chromosomes would be onechromosome. A secondary spermatocyte therefore would have only one sexchromosome and it could be a X OR a Y but not both. This single sex chromosomein a secondary spermatocyte would have 2 chromatids which will separate atanaphase H.

B. is correct.

The adult neuron is a nonreplicator or fixed postmitotic cell. All other associationsare correct.

C. is correct.

B. is correct.

MPF, now also known as M phase promoting factor, exists during the interphase ofthe cell cycle in an inactive form. Once it becomes active it causes the changeslisted in the question. Cyclin is a protein which accumulates during interphase.

Page 34: Review Questions for Human Histology

MPF activation also activates cyclin protease which means that cyclin is brokendown during the M phase. Nuclear lamin is a very durable intermediate filamentprotein which when phosphorylated by MPF solubilizes. Dephosphorylation oflamin causes its reassembly and the reassembly of the nuclear envelope.

2.025 The functional activity of which of the following substances is needed topropel cells along the cell cycle?

A. DNAse

B. RNAse

C. cyclin-dependent kinase

D. cyclin protease

E. tubulin

2.026 Which of the following correctly apply for "the molecule of the year" for 1993:p53?

A. its gene is the most commonly inactivated gene in human cancer, is inactive in50% of cancers

B. it promotes programmed cell death or apoptosis

C. it functions as a translation factor

D. A & B are correct

E. A & B & C are correct

2.027 Programmed cell death or PCD is also known as:

A. hypertrophy

B. hyperplasia

C. necrosis

D. apoptosis

E. dysplasia

C. is correct.

There are 8-10 different proteins which belong to a family of proteins known as

Page 35: Review Questions for Human Histology

cyclins. They were originally named cyclins because their intracellularconcentrations rose and fell as the cell progressed along the cell cycle. Theyfunction by turning on, at the appropriate moment, enzymes called cyclindependentkinases or CDKs, whose activity is needed to propel cells through the cell cycle.The cyclin D's are active during GI and help the cell make the decision to enterDNA synthesis. Cyclin E reaches maximal concentration at about the time of thelate G1 or early S phase. Cyclin A reaches maximal concentration during the Sphase. Cyclin B reaches maximal concentration during G2. Overexpression of thegene for cyclin Dl may be involved in pushing cells into a cancerous state as hasbeen demonstrated in the breast, esophagus and B cell lymphomas.

A. is correct.

The 53-kilodalton protein product of the p53 gene functions as a transcription factorwhich controls the expression of other genes. In this role it turns on a gene whoseprotein product, p21, stops cells from progressing around the cell cycle by bindingto a cyclin-dependent kinase. This arrest of progress around the cell cycle givesthe cell the time it needs to repair any damaged DNA it has. Absence of p53 doesnot stop cycle progress and cells with damaged or defective DNA are allowed toreproduce and many of these apparently progress to cancer.

D. is correct.

Apoptosis comes from Greek and refers to the falling of dead leaves in autumn. It isa specific set of morphological steps taken by cells destined to die physiologically:nuclei become highly condensed and pyknotic, nuclear DNA is cut between thenucleosomes by endonucleases into small pieces, the cell eventually fragmentsinto small membraneenclosed pieces. Hypertrophy refers to an increase in size.Hyperplasia refers to an increase in number. Necrosis is another form of cell deathin which cells swell, the membrane ruptures and the cell contents are released.Dysplasia is an abnormal formation or organization as in precancerous changes inan epithelium.

2.028 Although apoptosis can occur in all cell types with the possible exception ofthe blastomeres in the early embryo, which of the following cell types would youexpect to demonstrate the highest rate of apoptosis in a 10 year old child?

A. cardiac myocytes

B. neurons

C. osteocytes

D. T lymphocytes

E. GI smooth muscle

Page 36: Review Questions for Human Histology

2.029 Which of the following are components of the cytoskeleton?

A. microtubules

B. microfrlaments

C. intermediate filaments

D. A & B are correct

E. A & B & C are correct

2.030 Chromatin actually represents which of the following?

A. all of the nuclear DNA molecules

B. histone chromosomal proteins

C. nonhistone chromosomal proteins

D. A & B are correct

E. A & B & C are correct

2.031 When in the cell cycle are the histone chromosomal proteins synthesized?

A. M phase

B. GI phase

C. S phase

D. G2 phase

E. GO phase

2.032 Which of the following pass through the nuclear envelope in the form ofsubunits?

A. mRNA

B. tRNA

C. rRNA

2.033 The integral or intrinsic membrane protein glycophorin of the vertebrate

Page 37: Review Questions for Human Histology

erythrocyte contains which of the following regions in sequence:

A. hydrophilic - hydrophilic - hydrophilic

B. hydrophobic - hydrophilic - hydrophobic

C. hydrophilic - hydrophobic - hydrophilic

D. hydrophobic - hydrophobic - hydrophobic

E. hydrophobic - hydrophobic - hydrophilic

D. is correct.

In the thymus, 99 of the developing thymocytes or T lymphocytes die by apoptosisfor every T lymphocyte which is allowed to live and leave the thymus.

E. is correct.

All three choices form supportive networks collectively known as the cytoskeleton.

E. is correct.

Chromatin is defined as the nuclear DNA in combination with the histone andnonhistone chromosomal proteins.

C. is correct.

Histones are synthesized when DNA is synthesized during the S phase. This isunderstandable because histones play a role in covering/opening DNA for RNAsynthesis and it is important to keep the covering/opening machinery in place assoon as complimentary copies of the DNA are manufactured.

C. is correct.

The subunits of the ribosome pass through the nuclear membrane and then join bytwos with mRNA molecules to form a functional ribosomal unit. mRNAs and tRNAspass through as intact molecules.

C. is correct.

Integral membrane proteins have two hydrophilic ends, one on the external surfaceand the other on the internal surface of the plasmalemma. The single hydrophobicportion is located in the lipid bi-layer of the membrane. These proteins areamphiphilic, i.e. distinct polar (hydrophilic) and non-polar (hydrophobic) regions.

2.034 Characterize a peripheral or extrinsic membrane protein.

Page 38: Review Questions for Human Histology

A. hydrophilic

B. hydrophobic

C. half hydrophobic; half hydrophilic

D. 75% hydrophobic; 25% hydrophilic

E. 75% hydrophilic; 25% hydrophobic

2.035 When a human cell with its integral membrane proteins labeled with a greenfluorescent dye is fused with a mouse cell with its integral membrane proteinslabeled with a red fluorescent dye, what will the fluorescent pattern be in the fused,hybrid cell about one hour after fusion?

A. total green

B. total yellow

C. one half of cell surface is green, the other yellow

D. random, even mix of green and yellow

2.036 Using labeled amino acids in a time course study of the origin of integralmembrane proteins demonstrated a sequence of labeling in all of the followingsites EXCEPT:

A. nucleus

B. rough ER

C. Golgi

D. plasmalemma

2.037 Given the fact that there are 20 amino acids and only 4 nucleotides in DNA,DNA codons must therefore utilize the nucleotides?

A. one at a time

B. two at a time

C. three at a time

D. four at a time

Page 39: Review Questions for Human Histology

E. five at a time

2.038 Some amino acids have only one nuclear DNA codon, others have two,three, four, and six codons. With few exceptions the same codons code for thesame amino acid whether the life form is man, bacterium, or virus. Some codons,however, do not code for any amino acid at all. Which of the following mRNAcodons do not code for any amino acid?

A. UAA

B. UGA

C. UAG

D. A & B are correct

E. A & B & C are correct

A. is correct.

These proteins are found on the external and internal surfaces of the plasmalemmaand therefore are not found in the lipid bilayer because they are hydrophilic.

D. is correct.

This is the experiment which was used to demonstrate that integral membraneproteins could move laterally in the cell membrane.

A. is correct.

Amino acids would not be taken up by the nucleus as an indication of proteinsynthesis.

C. is correct.

Choice A would only result in 4 codons from 4 nucleotides (41=4). Choice B wouldstill not result in enough (42=16). Choice C results in 43=64 which is more than the20 required. It turns out that codons are formed from triplets of nucleotides or, inother words, codons are three nucleotides in length. If as little as one nucleotide ischanged in a triplet, then one amino acid can be changed (the valine-glutamic acidswap) which can drastically alter the final shape of the protein (in this casehemoglobin A becomes hemoglobin S of sickle cell trait or anemia).

E. is correct.

These three codons are read as terminators of protein synthesis.

Page 40: Review Questions for Human Histology

2.039 What would the A-T-G-C-C-G-T-A-A nucleotide sequence in one DNAstrand become in the newly synthesized complementary strand during DNAreplication?

A. A-T-G-C-C-G-T-A-A

B. A-U-G-C-C-G-U-A-A

C. T-A-C-G-G-C-A-T-T

D. U-A-G-C-C-G-A-U-U

2.040 Chromatin is composed of which of the following?

A. DNA

B. histones

C. nonhistone proteins

D. A & B are correct

E. A & B & C are correct

2.041 Which of the following histones is NOT involved in the formation of thenucleosome?

A. Hl

B. H2A

C. 1-1213

D. H3

E. H4

2.042 In the nucleosome DNA is wound around a core of histones two times. Howmany core histones construct the core of the nucleosome around which the DNA iswound?

A. one each of each of the 4 core histones

B. two each of each of the 4 core histones

C. three each of each of the 4 core histones

Page 41: Review Questions for Human Histology

D. four each of each of the 4 core histones

E. five each of each of the 4 core histones

2.043 If the total DNA content of a human cell is stretched out its estimated 6 billionnucleotide pairs would be about 400cm long. The winding of the DNA to formnucleosomes and then the winding of the nucleosomes into solenoids compactsthe length of the total DNA in a human cell by a factor of about?

A. 100

B. 1000

C. 10,000

D. 100,000

E. 1,000,000

2.044 Chromosomes are probably held in place in the nucleus by attachment towhich of the following:

A. nucleolus

B. outer nuclear envelope

C. perinuclear cistern

D. lumen of nuclear pore

E. lamins on inside of inner nuclear envelope

C. is correct.

The complementary combinations for DNA are A- T:G-C. In RNA T=U.

E. is correct.

All three of these: DNA and the two different classes of proteins arrangethemselves together to form chromatin.

A. is correct.

Choices B-E are referred to as the core histones because they form the bead-likestructure around which DNA wraps to form a nucleosome. Hl is not involved informing the core of the nucleosome. Hl is involved in linking one nucleosome to thenext nucleosome and sometimes is referred to as linker histone.

Page 42: Review Questions for Human Histology

B. is correct.

Each of the 4 core histones is represented twice in the histone core of thenucleosome, thus forming an octamer of core histones. Nucleosomes are groupedtogether in larger structures called nucleosome coils or solenoids which contain 6-8 nucleosomes.

C. is correct.

E. is correct.

Lamins are part of the nucleoskeleton of the nucleus and some data indicate thatchromosomes may be attached to the lamins during interphase. The fibrous coatformed by the nuclear lamins on the inner aspect of the inner nuclear envelope iscalled the nuclear lamina.

2.045 The proteins involved in facilitated diffusion and active transport across theplasmalemma are?

A. integral membrane proteins

B. peripheral membrane proteins found only on the exterior surface of the cellmembrane

C. peripheral membrane proteins found only on the internal surface of the cellmembrane

2.046 Substances moving across the cell membrane against a concentrationgradient such as Na, H, Ca, glucose and amino acids are moved by whichmechanism?

A. osmosis

B. facilitated diffusion (not energy dependent)

C. active transport (expenditure of cellular energy)

2.047 All of the following are characteristic of active transport EXCEPT:

A. moves material against chemical gradient

B. can be prevented if poisons given to disrupt source of metabolic energy

C. depends on presence of peripheral membrane proteins

D. moves material against electrical gradient

Page 43: Review Questions for Human Histology

E. depends on presence of integral membrane proteins

2.048 The enzymes neuraminidase and galactosidase specifically remove sialicacid units and galactose units, respectively, from the cell surface and therebydestroy the receptor sites on the cell surface. This means that all cell surfacereceptors are ?

A. polypeptides

B. phospholipids

C. glycoproteins

D. nucleic acids

E. fatty acids

2.049 Which of the following is an example of a second messenger as a cellresponds to a substance attaching to its specific receptor on the cell surface?

A. peptide hormone

B. growth factor

C. neurotransmitter

D. ABO blood grouping glycolipids

E. cyclic AMP

A. is correct.

Artificial phospholipid membranes without proteins pass all molecules and ions,except water, purely on the basis of lipid solubility and molecular size. In livingcells, however, amino acids, glucose and Na ions for example, penetrate mustfaster than expected. Addition of integral but not peripheral membrane proteins toartificial phospholipid membranes permitted the passage of ions and polarmolecules. In order to do this the protein must span the entire phospholipid bilayer,i.e. has both hydrophilic and hydrophobic regions.

C. is correct.

Active transport can move some substances against a concentration gradient withdifferences as much as a million fold. The energy utilized for this mechanism isderived from the hydrolysis of ATP.

Page 44: Review Questions for Human Histology

C. is correct.

Peripheral or extrinsic membrane proteins are not involved in active transport.

C. is correct.

It is known that all cell surface receptors and recognition sites are glycoproteins orglycolipids.

E. is correct.

Receptors only have to bind their ligand at the cell surface. This is an example of afirst messenger. Once this happens there is a generation of second messengers,which are small molecules eventually causing activation of protein kinases whichare involved in generating the cell response to the first messenger. Cyclic AMP is awell known second messenger.

2.050 Which cell type 1) initially differentiates a cell adhesion molecule (CAM)which keeps it "stuck" in a particular region of the embryo, 2) then loses this CAMwhich allows this cell type to migrate extensively and finally 3) CAMs reappear onthe cell surface specific for "holding" these cells at their normal anatomical sitesafter their extensive period of migration when they did not have CAMs?

A. bone marrow cells

B. fibroblasts

C. cardiac myocytes

D. neural crest

E. hepatocytes

2.051 A radioactive amino acid is exposed to cells in vitro (the pulse) and then theculture medium is changed back to normal by removal of the "hot" medium bynormal medium. Samples of cells are taken for EM autoradiography at multipletime points (the chase). Arrange the following in the correct sequence for theappearance of label in this pulse-chase experiment: 1=over the Golgi apparatus;l=over the cisternae of the RER; 3=over membrane vesicles in the cytoplasm;4=exocytosed into medium at cell surface.

A. 1-2-3-4

B. 2-1-3-4

C. 3-2-1-4

Page 45: Review Questions for Human Histology

D. 2-3-1-4

E. 3-1-2-4

2.052 How many DNA replication origins or points of initiation of DNA synthesisare there in the human genome?

A. less than the haploid number of chromosomes

B. equal to the haploid number of chromosomes

C. equal to the diploid number of chromosomes

D. greater than the diploid number of chromosomes

2.053 All of the following are special sequences in nuclear chromosomal DNAEXCEPT.

A. centromere

B. kinetochore

C. telomere

D. replication initiation site

E. nucleolar organizer

D. is correct.

Cell adhesion molecules or CAMs are responsible for cells adhering or sticking toa particular location. The cell type, of those listed, famous for its migration from theregion of the neural tube to a variety of distant sites is the neural crest. Whenneural crest cells are not migrating they express CAMs. When they are migratingthese CAMs are not expressed.

B. is correct.

D. is correct.

Each human chromosome has many replication origins along its length. Thesepoints become activated asynchronously during the S phase of the cell cycle.

B. is correct.

The kinetochore region of each chromosome is a nucleoprotein area associatedwith the centromere DNA sequence. Each G2 to metaphase chromosome has one

Page 46: Review Questions for Human Histology

centromere DNA sequence region which is "attached" to a kinetochore region oneach side of the chromosome. Microtubules of the mitotic spindle insert into each ofthe two kinetochore regions of each chromosome.

2.054 Which of the following causes a bending/folding of a guanosine-rich DNAstrand forming a structure that protects the ends of each chromosome?

A. kinetochorease

B. centromerease

C. telomerase

D. RNAse

E. DNAse

2.055 All of the following are located on or in the inner mitochondrial membraneEXCEPT.

A. most of the citric acid cycle enzymes

B. proteins of ATP synthesis

C. proteins involved in the respiratory chain

D. proteins involved in the transport of substrates into and out of the mitochondrion

E. the phospholipid cardiolipin

2.056 DNA is inactive in transcription for any particular kind of cell in which of thefollowing?

A. constitutive heterochromatin

B. facultative heterochromatin

C. both constitutive and facultative heterochromatin

D. euchromatin

E. neither euchromatin nor heterochromatin

2.057 DNA is permanently inactive in transcription for all cell types in the organismin which of the following?

A. constitutive heterochromatin

Page 47: Review Questions for Human Histology

B. facultative heterochromatin

C. both constitutive and facultative heterochromatin

D. euchromatin

E. neither euchromatin nor heterochromatin

2.058 DNA is active in transcription for any particular kind of cell in which of thefollowing?

A. constitutive heterochromatin

B. facultative heterochromatin

C. both constitutive and facultative heterochromatin

D. euchromatin

E. neither euchromatin nor heterochromatin

2.059 In electron microscopic autoradiography, exposing a cell to tritiated uridinewill label which of the following?

A. constitutive heterochromatin

B. facultative heterochromatin

C. both constitutive and facultative heterochromatin

D. euchromatin

E. neither euchromatin or heterochromatin

C. is correct.

The telomere is the sequence in DNA located at the ends of each chromosomewhich determines or defines the ends of each chromosome. Telomerase is theenzyme which packages the telomere into a protected end on each chromosome.

A. is correct.

Most of the citric acid cycle enzymes are located in the matrix region along withmitochondrial DNA and mitochondrial enzymes.

C. is correct.

Page 48: Review Questions for Human Histology

This could apply to both constitutive and facultative heterochromatin meaning thatin any particular cell in the body its constitutive heterochromatin would bepermanently inactive and major parts of its facultative could also be inactive.

A. is correct.

This is a definition of constitutive heterochromatin.

D. is correct.

This is a description of the functional role of euchromatin as opposed to that ofheterochromatin.

D. is correct.

Euchromatin is the functional state of chromatin, i.e. it is where RNA is beingsynthesized from a DNA template. The labeled uridine would be taken up by theseregions of the genome.

2.060 In electron microscopic autoradiography, exposing a cell to tritiated leucinewill label which of these areas when performed immediately after exposure to theisotope?

A. constitutive heterochromatin

B. facultative heterochromatin

C. both constitutive and facultative heterochromatin

D. euchromatin

E. neither euchromatin nor heterochromatin

2.061 DNA contributes to the formation of chromosomes in which of the following?

A. constitutive heterochromatin

B. facultative heterochromatin

C. both constitutive and facultative heterochromatin

D. euchromatin

E. neither euchromatin or heterochromatin

2.062 Which of the following applies to mitochondrial DNA?

Page 49: Review Questions for Human Histology

A. constitutive heterochromatin

B. facultative heterochromatin

C. both constitutive and facultative heterochromatin

D. euchromatin

E. neither euchromatin or heterochromatin

2.063 If a cell in metaphase of mitosis is exposed to tritiated uridine for 5 minutesand ARG is performed immediately (note: cell cycle times are 4hr=G1, 8hr=S,2hr=G2, 1hr=M), then:

A. in an autoradiograph, label would appear over the morphologically apparentchromosomes

B. in an autoradiograph, label would NOT appear over the morphologicallyapparent chromosomes

2.064 If a cell in metaphase of mitosis is exposed to tritiated thymidine for 5minutes and ARG performed immediately (note: cell cycle times are 4hr-G1, 8hr=S,2hr=G2, lhr=M), then:

A. in an autoradiograph, label would appear over the morphologically apparentchromosomes

B. in an autoradiograph, label would NOT appear over the morphologicallyapparent chromosomes

2.065 If cells in S phase are exposed to tritiated thymidine for 5 minutes and ARGperformed 1 hour later (note: cell cycle times are 4hr=G1, 8hr=S, 2hr--G2, 1hr=M),then:

A. in an autoradiograph, label would appear over the morphologically apparentchromosomes

B. in an autoradiograph, label would NOT appear over the morphologicallyapparent chromosomes

E. is correct.

This label would be incorporated over the RER/free polysomes in the cytoplasm,not over any kind of chromatin.

C. is correct.

Page 50: Review Questions for Human Histology

Nuclear or chromosomal DNA can be classified as constitutive heterochromatin,which is permanent heterochromatin that is present in this part of the nuclear DNAfor the life of the organism, or as facultative heterochromatin, which is DNA that canbecome or is functional (serves as a template for RNA synthesis).

E. is correct.

Mitochondrial DNA does not have the descriptors euchromatin or heterochromatinapplied to it. These terms only apply to nuclear DNA.

B. is correct.

Since the DNA in the chromosomes is completely heterochromatinized it can notserve as a template for any RNA synthesis.

B. is correct.

A cell in metaphase is not undergoing DNA synthesis and could not incorporatethe tritiated thymidine.

B. is correct.

This is not enough time between the act of labeling and the preparation of the ARGfor the labeled DNA to "show up" in (over) individual chromosomes. The labelwould be present in (over) the interphaseconfigured nuclei, but not over anymorphologically apparent chromosomes.

2.066 If a cell in prophase of mitosis is exposed to tritiated uridine for 15 minutesand ARG performed immediately (note: cell cycle times are 4hr Gl, 8hr=S, 2hr=G2,lhr=M), then:

A. in an autoradiograph, label would appear over the morphologically apparentchromosomes

B. in an autoradiograph, label would NOT appear over the morphologicallyapparent chromosomes

2.067 Synthesis of the DNA of sex chromosomes occurs:

A. in the cytoplasm

B. in the nucleus

2.068 Synthesis of circular DNA occurs:

A. in the cytoplasm

Page 51: Review Questions for Human Histology

B. in the nucleus

2.069 Synthesis of mRNA occurs:

A. in the cytoplasm

B. in the nucleus

2.070 Synthesis of tRNA occurs:

A. in the cytoplasm

B. in the nucleus

2.071 Synthesis of rRNA occurs:

A. in the cytoplasm

B. in the nucleus

2.072 Synthesis of all 5 types of histones occurs:

A. in the cytoplasm

B. in the nucleus

2.073 Synthesis of nuclear matrix proteins occurs:

A. in the cytoplasm

B. in the nucleus

2.074 Synthesis of nuclear lamins A, B & C occurs:

A. in the cytoplasm

B. in the nucleus

B. is correct.

B. is correct.

Since the nuclear DNA is packaged into heterochromatin for the mitotic event, it isnot available for "readout" and therefore can not serve as a template for any kind ofRNA synthesis at all. There would be no label over the chromosomes in thisparticular situation.

Page 52: Review Questions for Human Histology

This DNA like the DNA of the non-sex chromosomes is synthesized in the nucleusduring the S phase of the cell cycle.

A. is correct.

Mitochondrial DNA is circular. Mitochondrial DNA is duplicated within themitochondrion.

B. is correct.

All messenger RNAs are synthesized from nuclear DNA in the nucleus and thentransported to the cytoplasm.

B. is correct.

All of the transfer RNAs (at least 31 species) are synthesized in the nucleus andtransported to the cytoplasm.

B. is correct.

Ribosomal RNA is coded for in the nucleolar organizer DNA. Ribosomal RNA issynthesized in the nucleolus which is in the nucleus.

A. is correct.

All classes of histones are synthesized in the cytoplasm during the S phase of thecell cycle and are immediately transported into the nucleus to perform theirfunctional role in the nucleosome arrangement and other structural roles.

A. is correct.

The proteins of the nuclear matrix are synthesized in the cytoplasm andtransported from there into the nucleus.

A. is correct.

These skeletal proteins of the nuclear envelope are synthesized in the cytoplasmand then transported to the inner aspect of the nuclear membrane where theyperform their function.

2.075 Synthesis of mitochondrial tRNAs occurs:

A. in the cytoplasm

B. in the nucleus

2.076 Synthesis of mitochondrial ribosomes occurs:

Page 53: Review Questions for Human Histology

A. in the cytoplasm

B. in the nucleus

2.077 Synthesis of nuclear proteins regulating gene expression occurs:

A. in the cytoplasm

B. in the nucleus

2.078 The DNA content of the skeletal muscle cell in a 42 year old jogger is:

A. equal to 1 C

B. equal to 2C

C. equal to 4C

D. greater than 4C

2.079 The DNA content of the cardiac muscle cell of a 7th month fetus is:

A. equal to 1 C

B. equal to 2C

C. equal to 4C

D. greater than 4C

2.080 The DNA content of the smooth muscle cell in myometrium of an 8 year oldgirl is:

A. equal to 1 C

B. equal to 2C

C. equal to 4C

D. greater than 4C

2.081 The DNA content of sperm in the head, not the tail, of the epididymis is:

A. equal to 1 C

B. equal to 2C

Page 54: Review Questions for Human Histology

C. equal to 4C

D. greater than 4C

2.082 The DNA content of a primary spermatocyte in zygotene stage is:

A. equal to 1 C

B. equal to 2C

C. equal to 4C

D. greater than 4C

A. is correct.

The tRNAs found in the mitochondrion are coded for in the mitochondrial DNA.

A. is correct.

Mitochondrial ribosomes are synthesized from mitochondrial DNA, not nuclearDNA.

A. is correct.

The nuclear proteins regulating gene expression are synthesized in the cytoplasmand transported to the nucleus.

D. is correct.

Since skeletal muscle cells are multinucleated they automatically contain amountsof DNA greater than 4C, whether the subject is a jogger or not.

B. is correct.

The cardiac muscle cell this early in development would probably be in GO/G I andhave 2C DNA. However, after birth and with an increasing cardiac work load, it isknown that cardiac muscle cells become polyploid in response to this work load.

B. is correct.

Smooth muscle cells in this non-pregnant situation would have 2C DNA. Duringpregnancy however, the smooth muscle cells of the myometrium can undergo bothhypertrophy and hyperplasia. During hyperplasia smooth muscle cells would berunning the cell cycle and would have amounts of DNA appropriate for the stage ofthe cell cycle they were in when sampling occurred.

Page 55: Review Questions for Human Histology

A. is correct.

It really does not matter whether the sperm is taken from the head or tail of theepididymis, from the testis, or from the ampulla of the ductus (vas) deferens. Asperm head contains the haploid number of chromosome and I C DNA.

C. is correct.

The primary spermatocyte is in prophase I of meiosis and therefore has the 4Camount of DNA.

2.083 The DNA content of mature megakaryocyte is:

A. equal to 1 C

B. equal to 2C

C. equal to 4C

D. greater than 4C

2.084 The DNA content of a purkinje cell in the cerebellum of an 80 year old is:

A. equal to 1C

B. equal to 2C

C. equal to 4C

D. greater than 4C

2.085 The DNA content of the primary oocyte in a primordial follicle of an 8 yearold girl is:

A. equal to 1C

B. equal to 2C

C. equal to 4C

D. greater than 4C

2.086 The DNA content of a plasma cell synthesizing monoclonal antibody is:

A. equal to 1 C

B. equal to 2C

Page 56: Review Questions for Human Histology

C. equal to 4C

D. greater than 4C

2.087 The DNA content of an orthochromatophilic normoblast in red bone marrowis:

A. equal to 1 C

B. equal to 2C

C. equal to 4C

D. greater than 4C

2.088 The DNA content of a ganglion cell in the retina is:

A. equal to 1 C

B. equal to 2C

C . equal to 4C

D. greater than 4C

2.089 The DNA content of a cell from a chorionic villus biopsy grown in vitro butstopped in its progress around the cell cycle by vincristine is:

A. equal to 1 C

B. equal to 2C

C. equal to 4C

D. greater than 4C

D. is correct.

A mature megakaryocyte has undergone several rounds of endoreduplication of itsDNA content without accompanying cytokinetic events. Such a cell would have aDNA content much higher than 4C.

B. is correct.

These cells are neurons living in an extended G1/GO phase and having 2C DNA.

Page 57: Review Questions for Human Histology

C. is correct.

By the 7-8th fetal month the fetal ovaries have all of the oogonia they will everyhave and these cells have completed a round of DNA synthesis. They are inprophase I of meiosis and will stay there until death due to atresia or until justbefore ovulation. It is just before ovulation when meiosis I is completed duringoogenesis.

B. is correct.

Plasma cells are terminally differentiated cells which are non-replicators living outtheir 2-3 week life span in G1/GO and therefore having 2C DNA.

B. is correct.

This is the stage in erythrocytopoiesis in which the nucleus is extruded from thecell. Orthochromatophilic normoblasts are examples of non-cycling cells andtherefore would have 2C DNA.

B. is correct.

A ganglion cell in the ganglion cell layer of the retina is a neuron. Neurons arefixed postmitotic cells living out their lives in an extended Gl or GO phase, thusthey would have 2C DNA.

C. is correct.

This would be the technique used to karyotype the fetus. Vincristine is used toprevent anaphase movement of the chromosomes so they can be used in a squashpreparation for the karyotype. Since the cell in question is blocked in metaphase itwould have 4C DNA.

2.090 The DNA content of a cell treated with hydroxyurea for 48 hrs. is:

A. equal to I C

B. equal to 2C

C. equal to 4C

D. greater than 4C

2.091 The DNA content of an ovulated but unfertilized 'ovum" is:

A. equal to 1 C

B. equal to 2C

Page 58: Review Questions for Human Histology

C. equal to 4C

D. greater than 4C

2.092 Which of the following codes for 31 transfer RNAs?

A. DNA-nuclear

B. DNA-mitochondrial

C. rough endoplasmic reticulum (RER)

D. smooth endoplasmic reticulum (SER)

E. filament-intermediate

F. filament-micro (microfilament)

G. Golgi apparatus

H. lysosome-primary

I. lysosome-secondary

J. microtubule

K. mitochondrion

L. nucleolus

M. peroxisome

N. plasmalemma-E face or surface

0. plasmalemma-P face or surface

P. fixed ribosome

Q. free ribosome

2.093 Which of the following is associated with glycocalyx?

A. DNA-nuclear

B. DNA-mitochondrial

Page 59: Review Questions for Human Histology

C. rough endoplasmic reticulum (RER)

D. smooth endoplasmic reticulum (SER)

E. filament-intermediate

F. filament-micro (microfilament)

G. Golgi apparatus

H. lysosome-primary

1. lysosome-secondary

J. microtubule

K. mitochondrion

L. nucleolus

M. peroxisome

N. plasmalemma-E face or surface

0. plasmalemma-P face or surface

P. fixed ribosome

Q. free ribosome

B. is correct.

Hydroxyurea prevents the G1 to S conversion. Cells treated with HU are preventedfrom entering S phase, therefore, they are in late G1 and they would have 2C DNA.

B. is correct.

The "ovum" actually is a secondary oocyte. It will die as a secondary oocyte unlessit is fertilized. It has 2C amount of DNA, 23 chromosomes (haploid) and a total of46 chromatids.

A. is correct.

This is a function of nuclear DNA. Mitochondrial DNA is know to code for only the22 species of tRNA found in the mitochondrial matrix.

N. is correct.

Page 60: Review Questions for Human Histology

The glycocalyx or cell coat is associated with the external surface of theplasmalemma. The glycocalyx represents the carbohydrate "ends" of theglycolipids and glycoproteins found in the plasmalemma. These "ends" areassociated with glycosaminoglycans and are bound together on the exteriorsurface of the plasmalemma forming a "fuzzy" coat as seen with the electronmicroscope.

2.094 Which of the following is associated with nuclear lamins A, B & C?

A. DNA-nuclear

B. DNA-mitochondrial

C. rough endoplasmic reticulum (RER)

D. smooth endoplasmic reticulum (SER)

E. filament-intermediate

F. filament-micro (microfilament)

G. Golgi apparatus

H. lysosome-primary

1. lysosome-secondary

J. microtubule

K. mitochondrion

L. nucleolus

M. peroxisome

N. plasmalemma-E face or surface

0. plasmalemma-P face or surface

P. fixed ribosome

Q. free ribosome

2.095 Which of the following is associated with tubulin?

A. DNA-nuclear

Page 61: Review Questions for Human Histology

B. DNA-mitochondrial

C. rough endoplasmic reticulum (RER)

D. smooth endoplasmic reticulum (SER)

E. filament-intermediate

F. filament-micro (microfilament)

G. Golgi apparatus

H. lysosome-primary

I. lysosome-secondary

J. microtubule

K. mitochondrion

L. nucleolus

M. peroxisome

N. plasmalemma-E face or surface

0. plasmalemma-P face or surface

P. fixed ribosome

Q. free ribosome

E. is correct.

There are three lamins (A, B & C) and they are a type of intermediate filamentwhich is involved in a skeletal support for the nuclear envelope.

J. is correct.

Microtubules are polymers of the protein tubulin. Tubulin dimers can be quicklyassembled into microtubules and microtubules can be quickly disassembled intotubulin dimers.

2.096 Which of the following represents the ribosomal gene loci?

A. DNA-nuclear

Page 62: Review Questions for Human Histology

B. DNA-mitochondrial

C. rough endoplasmic reticulum (RER)

D. smooth endoplasmic reticulum (SER)

E. filament-intermediate

F. filament-micro (microfilament)

G. Golgi apparatus

H. lysosome-primary

I. lysosome-secondary

J. microtubule

K. mitochondrion

L. nucleolus

M. peroxisome

N. plasmalemma-E face or surface

0. plasmalemma-P face or surface

P. fixed ribosome

Q. free ribosome

2.097 Where is protein intended for exocytosis synthesized?

A. DNA-nuclear

B. DNA-mitochondria

C. rough endoplasmic reticulum (RER)

D. smooth endoplasmic reticulum (SER)

E. filament-intermediate

F. filament-micro (microfilament)

G. Golgi apparatus

Page 63: Review Questions for Human Histology

H. lysosome-primary

I. lysosome-secondary

J. microtubule

K. mitochondrion

L. nucleolus

M. peroxisome

N. plasmalemma-E face or surface

0. plasmalemma-P face or surface

P. free ribosome

L. is correct.

The nucleolus is the cellular site of localization of the genes coding for thesynthesis of rRNA. In the human 5 chromosomes contain secondary constrictionswhich are the DNA sites for ribosomal RNA. These regions of DNA are callednucleolar organizer regions. They are found on chromosomes 13, 14, 15, 21 and22. During interphase the 10 ribosomal gene loci (5 derived from the maternalparent and 5 derived from the paternal parent) aggregate to form an area whereribosomal RNA will be transcribed. This region is the nucleolus.

C. is correct.

If a protein or glycoprotein is going to be released by a cell as a secretory product,such as a digestive enzyme from the acinar cells of the pancreas, it is synthesizedon ribosomes attached to the endoplasmic reticulum.

2.098 Where is sphingomyelin located primarily?

A. DNA-nuclear

B. DNA-mitochondrial

C. rough endoplasmic reticulum (RER)

D. smooth endoplasmic reticulum (SER)

E. filament-intermediate

Page 64: Review Questions for Human Histology

F. filament-micro (microfilament)

G. Golgi apparatus

H. lysosome-primary

I. lysosome-secondary

J. microtubule

K. mitochondrion

L. nucleolus

M. peroxisome

N. plasmalemma-E face or surface

0. plasmalemma-P face or surface

P. fixed ribosome

Q. free ribosome

2.099 Taxol is a drug which blocks assembly of which of the following?

A. DNA-nuclear

B. DNA-mitochondrial

C. rough endoplasmic reticulum (RER)

D. smooth endoplasmic reticulum (SER)

E. filament-intermediate

F. filament-micro (microfilament)

G. Golgi apparatus

H. lysosome-primary

I. lysosome-secondary

J. microtubule

K. mitochondrion

Page 65: Review Questions for Human Histology

L. nucleolus

M. peroxisome

N. plasmalemma-E face or surface

0. plasmalemma-P face or surface

P. fixed ribosome

Q. free ribosome

N. is correct.

Sphingomyelin is the type of phospholipid primarily found in the outer leaflet of theplasmalemma.

J. is correct.

Taxol is a drug which prevents the disassembly of the kinetochore microtubules.This disassembly is necessary for the movement of the chromosomes to theopposite poles of the cell during anaphase.

2.100 For which of the following is the following statement true: the signalrecognition particle docks with its receptor, which is an integral membrane protein?

A. DNA-nuclear

B. DNA-mitochondrial

C. rough endoplasmic reticulum (RER)

D. smooth endoplasmic reticulum (SER)

E. filament-intermediate

F. filament-micro (microfilament)

G. Golgi apparatus

H. lysosome-primary

I. lysosome-secondary

J. microtubule

K. mitochondrion

Page 66: Review Questions for Human Histology

L. nucleolus

M. peroxisome

N. plasmalemma-E face or surface

0. plasmalemma-P face or surface

P. free ribosome

2.101 Which of the following is associated with actin in skeletal muscle cells?

A. DNA-nuclear

B. DNA-mitochondrial

C. rough endoplasmic reticulum (RER)

D. smooth endoplasmic reticulum (SER)

E. filament-intermediate

F. filament-micro (microfilament)

G. Golgi apparatus

H. lysosome-primary

1. lysosome-secondary

J. microtubule

K. mitochondrion

L. nucleolus

M. peroxisome

N. plasmalemma-E face or surface

0. plasmalemma-P face or surface

P. fixed ribosome

Q. free ribosome

Page 67: Review Questions for Human Histology

C. is correct.

The signal recognition sequence on the mRNA attaches to a signal sequencerecognition particle (SRP) which in turn then docks with a receptor protein in themembrane of the endoplasmic reticulum. This docking procedure insures that theprotein to be manufactured will enter the lumen of the endoplasmic reticulum.

F. is correct.

Actin containing microfilaments help form the thin filaments of striated muscle..

2.102 Which of the following is associated with 45S precursor RNA found in afibrillar region?

A. DNA-nuclear

B. DNA-mitochondrial

C. rough endoplasmic reticulum (RER)

D. smooth endoplasmic reticulum (SER)

E. filament-intermediate

F. filament-micro (microfilament)

G. Golgi apparatus

H. lysosome-primary

I. lysosome-secondary

J. microtubule

K. mitochondrion

L. nucleolus

M. peroxisome

N. plasmalemma-E face or surface

0. plasmalemma-P face or surface

P. fixed ribosome

Q. free ribosome

Page 68: Review Questions for Human Histology

2.103 Which of the following is associated with desmin filaments?

A. DNA-nuclear

B. DNA-mitochondrial

C. rough endoplasmic reticulum (RER)

D. smooth endoplasmic reticulum (SER)

E. filament-intermediate

F. filament-micro (microfilament)

G. Golgi apparatus

H. lysosome-primary

1. lysosome-secondary

J. microtubule

K. mitochondrion

L. nucleolus

M. peroxisome

N. plasmalemma-E face or surface

0. plasmalemma-P face or surface

P. fixed ribosome

Q. free ribosome

L. is correct.

This is a description of what is going on and/or is a component of the nucleolus.The fibrillar areas in a nucleolus represent areas of concentration of the 45S RNAprecursor molecule. This molecule is then cleaved or processed into smaller units(5.8, 18 and 28S RNAs). These RNAs are complexed with proteins to eventuallyform the small and large subunits of the ribosome. The granular region of thenucleolus is where the ribosomal subunits are forming ribosomes.

E. is correct.

Page 69: Review Questions for Human Histology

Desmin filaments are one of the types of intermediate filaments. Desminintermediate filaments are found in skeletal, cardiac and smooth muscle cells. Incardiac and skeletal muscle the desmin filaments are primarily located in the Zlines. In smooth muscle cells the desmin filaments are connected to the densebodies (physiological equivalent for the smooth muscle cell as the Z line is forstriated muscle cells).

2.104 Which of the following fuses with heterophagosome?

A. DNA-nuclear

B. DNA-mitochondrial

C. rough endoplasmic reticulum (RER)

D. smooth endoplasmic reticulum (SER)

E. filament-intermediate

F. filament-micro (microfilament)

G. Golgi apparatus

H. lysosome-primary

1. lysosome-secondary

J. microtubule

K. mitochondrion

L. nucleolus

M. peroxisome

N. plasmalemma-E face or surface

0. plasmalemma-P face or surface

P. fixed ribosome

Q. free ribosome

2.105 Where is spectrin located primarily?

A. DNA-nuclear

Page 70: Review Questions for Human Histology

B. DNA-mitochondrial

C. rough endoplasmic reticulum (RER)

D. smooth endoplasmic reticulum (SER)

E. filament-intermediate

F. filament-micro (microfilament)

G. Golgi apparatus

H. lysosome-primary

1. lysosome-secondary

J. microtubule

K. mitochondrion

L. nucleolus

M. peroxisome

N. plasmalemma-E face or surface

0. plasmalemma-P face or surface

P. fixed ribosome

Q. free ribosome

H. is correct.

A heterophagosome is formed when a cell takes up, for example by phagocytosis,a piece of something other than itself (which would be autophagy). To digest this"foreign" material a primary lysosome fuses with the heterophagosome to form asecondary lysosome in which the enzymes of the lysosome attack the "foreign"material.

0. is correct.

Spectrin forms a skeleton for the plasmalemma. The spectrin membrane skeletonis composed of proteins and it is responsible for maintaining cell shape andmembrane stability. The spectrin membrane skeleton is associated with the Psurface of the plasmalemma.

Page 71: Review Questions for Human Histology

2.106 Which of the following forms cytoskeletal scaffolding in microvilli?

A. DNA-nuclear

B. DNA-mitochondrial

C. rough endoplasmic reticulum (RER)

D. smooth endoplasmic reticulum (SER)

E. filament-intermediate

F. filament-micro (microfilament)

G. Golgi apparatus

H. lysosome-primary

I. lysosome-secondary

J. microtubule

K. mitochondrion

L. nucleolus

M. peroxisome

N. plasmalemma-E face or surface

0. plasmalemma-P face or surface

P. fixed ribosome

Q. free ribosome

2.107 Where are peripheral membrane proteins synthesized?

A. DNA-nuclear

B. DNA-mitochondrial

C. rough endoplasmic reticulum (RER)

D. smooth endoplasmic reticulum (SER)

E. filament-intermediate

Page 72: Review Questions for Human Histology

F. filament-micro (microfilament)

G. Golgi apparatus

H. lysosome-primary

I. lysosome-secondary

J. microtubule

K. mitochondrion

L. nucleolus

M. peroxisome

N. plasmalemma-E face or surface

0. plasmalemma-P face or surface

P. fixed ribosome

Q. free ribosome

F. is correct.

The skeletal core of each microvillus is composed of about 30 parallelmicrofilaments of actin bundled together by the proteins villin and fimbrin.

Q. is correct.

Peripheral membrane proteins are synthesized on free polysomes, not onpolysomes attached to the endoplasmic reticulum.

2.108 Where is phosphatidylserine located primarily?

A. DNA-nuclear

B. DNA-mitochondrial

C. rough endoplasmic reticulum (RER)

D. smooth endoplasmic reticulum (SER)

E. filament-intermediate

F. filament-micro (microfilament)

Page 73: Review Questions for Human Histology

G. Golgi apparatus

H. lysosome-primary

I. lysosome-secondary

J. microtubule

K. mitochondrion

L. nucleolus

M. peroxisome

N. plasmalemma-E face or surface

0. plasmalemma-P face or surface

P. fixed ribosome

Q. free ribosome

2.109 Which of the following applies to neurofilaments?

A. DNA-nuclear

B. DNA-mitochondrial

C. rough endoplasmic reticulum (RER)

D. smooth endoplasmic reticulum (SER)

E. filament-intermediate

F. filament-micro (microfilament)

G. Golgi apparatus

H. lysosome-primary

I. lysosome-secondary

J. microtubule

K. mitochondrion

Page 74: Review Questions for Human Histology

L. nucleolus

M. peroxisome

N. plasmalemma-E face or surface

0. plasmalemma-P face or surface

P. fixed ribosome

Q. free ribosome

0. is correct.

The phospholipid distribution in the plasmalemma is asymmetrical. The cholinecontaining phospholipids (phosphatidylcholine and sphingomyelin) are mostlyassociated with the outer leaflet of the plasmalemma. In contrast to this the aminecontaining phospholipids (phosphatidylserine and phosphatidylethanolamine) areprimarily found on the inner leaflet of the plasmalemma.

E. is correct.

Neurofilaments are one of the many types of intermediate filaments. Intermediatefilaments were named as intermediate because they are intermediate in sizebetween microtubules (about 25 nm in diameter) and the microfilaments (6 urn indiameter). Intermediate filaments are about 8-10 nm thick.

2.110 Where are integral membrane proteins synthesized?

A. DNA-nuclear

B. DNA-mitochondrial

C. rough endoplasmic reticulum (RER)

D. smooth endoplasmic reticulum (SER)

E. filament-intermediate

F. filament-micro (microfilament)

G. Golgi apparatus

H. lysosome-primary

I. lysosome-secondary

Page 75: Review Questions for Human Histology

J. microtubule

K. mitochondrion

L. nucleolus

M. peroxisome

N. plasmalemma-E face or surface

0. plasmalemma-P face or surface

P. fixed ribosome

Q. free ribosome

2.111 Which of the following is associated with mitotic spindle?

A. DNA-nuclear

B. DNA-mitochondrial

C. rough endoplasmic reticulum (RER)

D. smooth endoplasmic reticulum (SER)

E. filament-intermediate

F. filament-micro (microfilament)

G. Golgi apparatus

H. lysosome-primary

1. lysosome-secondary

J. microtubule

K. mitochondrion

L. nucleolus

M. peroxisome

N. plasmalemma-E face or surface

0. plasmalemma-P face or surface

Page 76: Review Questions for Human Histology

P. fixed ribosome

Q. free ribosome

P. is correct.

Integral membrane proteins are synthesized on ribosomes attached to theendoplasmic reticulum. Peripheral membrane proteins, on the other hand aresynthesized on free polysomes.

J. is correct.

The mitotic spindle is composed of microtubules. Some of these microtubulesattach to the kinetochores of the chromosomes (at this point in time eachchromosome would be composed of two chromatids, each with a kinetochoreregion on its "noncentromeric" side). These are kinetochore microtubules. Othermicrotubules of the mitotic spindle do not attach to kinetochores and they arecalled polar microtubules. At anaphase the kinetochore microtubules disassembleand this is necessary for the anaphase movement of the chromosomes.

2.112 Which is electrically more negative than positive?

A. DNA-nuclear

B. DNA-mitochondrial

C. rough endoplasmic reticulum (RER)

D. smooth endoplasmic reticulum (SER)

E. filament-intermediate

F. filament-micro (microfilament)

G. Golgi apparatus

H. lysosome-primary

I. lysosome-secondary

J. microtubule

K. mitochondrion

L. nucleolus

Page 77: Review Questions for Human Histology

M. peroxisome

N. plasmalemma-E face or surface

0. plasmalemma-P face or surface

P. fixed ribosome

Q. free ribosome

2.113 For which of the following is the following statement true: a "trans face" budsoff this vesicle containing proteases, nucleases, lipases and other -ases?

A. DNA-nuclear

B. DNA-mitochondrial

C. rough endoplasmic reticulum (RER)

D. smooth endoplasmic reticulum (SER)

E. filament-intermediate

F. filament-micro (microfilament)

G. Golgi apparatus

H. lysosome-primary

I. lysosome-secondary

J. microtubule

K. mitochondrion

L. nucleolus

M. peroxisome

N. plasmalemma-E face or surface

0. plasmalemma-P face or surface

P. fixed ribosome

Q. free ribosome

Page 78: Review Questions for Human Histology

0. is correct.

The inside of the cell membrane is more negatively charged than the outside of thecell membrane because of the distribution of positive and negative charged ionsacross the plasmalemma.

H. is correct.

The vesicle in question has just budded from the mature face of the Golgiapparatus and it contains a wide variety of enzymes such as proteases,hydrolyases, DNA and RNAases, lipases, etc. This is a description of a primarylysosome or a zymogen granule depending on the final destination of thismembrane bound bag of enzymes.

2.114 Which is located inside both cilia and flagella?

A. DNA-nuclear

B. DNA-mitochondrial

C. rough endoplasmic reticulum (RER)

D. smooth endoplasmic reticulum (SER)

E. filament-intermediate

F. filament-micro (microfilament)

G. Golgi apparatus

H. lysosome-primary

1. lysosome-secondary

J. microtubule

K. mitochondrion

L. nucleolus

M. peroxisome

N. plasmalemma-E face or surface

0. plasmalemma-P face or surface

P. fixed ribosome

Page 79: Review Questions for Human Histology

Q. free ribosome

2.115 Where are cytosolic proteins synthesized?

A. DNA-nuclear

B. DNA-mitochondrial

C. rough endoplasmic reticulum (RER)

D. smooth endoplasmic reticulum (SER)

E. filament-intermediate

F. filament-micro (microfilament)

G. Golgi apparatus

H. lysosome-primary

I. lysosome-secondary

J. microtubule

K. mitochondrion

L. nucleolus

M. peroxisome

N. plasmalemma-E face or surface

0. plasmalemma-P face or surface

P. fixed ribosome

Q. free ribosome

J. is correct.

Cilia and flagella are quite similar ultrastructurally. They contain an axonome. Theaxonome is composed of a nine-plus-two array of microtubules; nine peripheraldoublets surrounding one central doublet of microtubules.

Q. is correct.

Page 80: Review Questions for Human Histology

In general, proteins synthesized for intracellular use rather than for extracellularuse, are synthesized on the free polysomes or the polysomes not attached to theendoplasmic reticulum.

2.116 Which is more electrically more positive than negative?

A. DNA-nuclear

B. DNA-mitochondrial

C. rough endoplasmic reticulum (RER)

D. smooth endoplasmic reticulum (SER)

E. filament-intermediate

F. filament-micro (microfilament)

G. Golgi apparatus

H. lysosome-primary

I. lysosome-secondary

J. microtubule

K. mitochondrion

L. nucleolus

M. peroxisome

N. plasmalemma-E face or surface

0. plasmalemma-P face or surface

P. fixed ribosome

Q. free ribosome

2.117 Where does initial glycosylation of a protein occur?

A. DNA-nuclear

B. DNA-mitochondrial

C. rough endoplasmic reticulum (RER)

Page 81: Review Questions for Human Histology

D. smooth endoplasmic reticulum (SER)

E. filament-intermediate

F. filament-micro (microfilament)

G. Golgi apparatus

H. lysosome-primary

I. lysosome-secondary

J. microtubule

K. mitochondrion

L. nucleolus

M. peroxisome

N. plasmalemma-E face or surface

0. plasmalemma-P face or surface

P. fixed ribosome

Q. free ribosome

N. is correct.

The outside of the plasmalemma, i.e. the E surface of the plasmalemma iselectrically more positive because slightly more positive ions are located here thanon the cytosol side, which contains more negative than positive ions. Because ofthis difference there is a voltage difference or membrane potential between theinside and the outside of the cell.

C. is correct.

Most proteins that are to be glycosylated are glycosylated in the RER as then leavethe ribosome and enter the lumen of the RER. In general, proteins that are notdestined to be glycosylated are synthesized on the free polysomes. This differencestrongly suggests that the enzymes responsible for glycosylation are found on thecisternal side of the RER.

2.118 Where are enzymes of the citric acid cycle found?

Page 82: Review Questions for Human Histology

A. DNA-nuclear

B. DNA-mitochondrial

C. rough endoplasmic reticulum (RER)

D. smooth endoplasmic reticulum (SER)

E. filament-intermediate

F. filament-micro (microfilament)

G. Golgi apparatus

H. lysosome-primary

1. lysosome-secondary

J. microtubule

K. mitochondrion

L. nucleolus

M. peroxisome

N. plasmalemma-E face or surface

0. plasmalemma-P face or surface

P. fixed ribosome

Q. free ribosome

2.119 Where are more 11V1Ps (intramembranous particles) found?

A. DNA-nuclear

B. DNA-mitochondria

C. rough endoplasmic reticulum (RER)

D. smooth endoplasmic reticulum (SER)

E. filament-intermediate

F. filament-micro (microfilament)

Page 83: Review Questions for Human Histology

G. Golgi apparatus

H. lysosome-primary

1. lysosome-secondary

J. microtubule

K. mitochondrion

L. nucleolus

M. peroxisome

N. plasmalemma-E face or surface

0. plasmalemma-P face or surface

P. fixed ribosome

Q. free ribosome

K. is correct.

The enzymes of the TCA cycle are found on the inner membrane and in the matrixof the mitochondrion.

0. is correct.

During freeze-fracture of the plasmalemma to expose the faces, not the surfaces, ofthe plasmalemma most of the particles stay with the P face of the plasmalemmaand not the E face. The particles represent the integral membrane proteins.

2.120 Which of the following is most closely associated with glial filaments inastrocytes?

A. DNA-nuclear

B. DNA-mitochondrial

C. rough endoplasmic reticulum (RER)

D. smooth endoplasmic reticulum (SER)

E. filament-intermediate

F. filament-micro (microfilament)

Page 84: Review Questions for Human Histology

G. Golgi apparatus

H. lysosome-primary

I. lysosome-secondary

J. microtubule

K. mitochondrion

L. nucleolus

M. peroxisome

N. plasmalemma-E face or surface

0. plasmalemma-P face or surface

P. fixed ribosome

Q. free ribosome

2.121 Which of the following codes for non-enzyme proteins of the outermitochondrial membrane?

A. DNA-nuclear

B. DNA-mitochondrial

C. rough endoplasmic reticulum (RER)

D. smooth endoplasmic reticulum (SER)

E. filament-intermediate

F. filament-micro (microfilament)

G. Golgi apparatus

H. lysosome-primary

I. lysosome-secondary

J. microtubule

K. mitochondrion

Page 85: Review Questions for Human Histology

L. nucleolus

M. peroxisome

N. plasmalemma-E face or surface

0. plasmalemma-P face or surface

P. fixed ribosome

Q. free ribosome

E. is correct.

Glial cell filaments are classified as intermediate filaments. Intermediate filamentsare intermediate in size between microfilaments and the microtubules. Glialfibrillary acid protein or GFAP is the polypeptide component of the intermediatefilaments characteristic for astrocytes.

A. is correct.

This is one of the "mitochondrial roles" played by nuclear DNA.

2.122 Where does intracellular transport of organelles and vesicles occur?

A. DNA-nuclear

B. DNA-mitochondrial

C. rough endoplasmic reticulum (RER)

D. smooth endoplasmic reticulum (SER)

E. filament-intermediate

F. filament-micro (microfilament)

G. Golgi apparatus

H. lysosome-primary

I. lysosome-secondary

J. microtubule

K. mitochondrion

Page 86: Review Questions for Human Histology

L. nucleolus

M. peroxisome

N. plasmalemma-E face or surface

0. plasmalemma-P face or surface

P. fixed ribosome

Q. free ribosome

2.123 Which of the following plays an important role in the phosphorylation oflysosomal proteins?

A. DNA-nuclear

B. DNA-mitochondrial

C. rough endoplasmic reticulum (RER)

D. smooth endoplasmic reticulum (SER)

E. filament-intermediate

F. filament-micro (microfilament)

G. Golgi apparatus

H. lysosome-primary

1. lysosome-secondary

J. microtubule

K. mitochondrion

L. nucleolus

M. peroxisome

N. plasmalemma-E face or surface

0. plasmalemma-P face or surface

P. fixed ribosome

Page 87: Review Questions for Human Histology

Q. free ribosome

J. is correct.

Microtubules are sort of the intracellular interstate highway system. They areinvolved in the intracellular movement of a variety of things such as chromosomesand, in the neuron, axoplasmic flow. They are believed to have a positive and anegative end. Kinesin may be the molecule involved in moving vesicles from thenegative end to the positive end of a microtubule; dynein may be the moleculeinvolved in the transport of vesicles from the positive end to the negative end of themicrotubule.

G. is correct.

The Golgi apparatus plays an important role in the phosphorylation of manydifferent kinds of proteins. The proteins destined for lysosomes are phosphorylatedin the cis-cisternae of the Golgi apparatus.

2.124 Autophagosome is associated with which of the following?

A. DNA-nuclear

B. DNA-mitochondrial

C. rough endoplasmic reticulum (RER)

D. smooth endoplasmic reticulum (SER)

E. filament-intermediate

F. filament-micro (microfilament)

G. Golgi apparatus

H. lysosome-primary

I. lysosome-secondary

J. microtubule

K. mitochondrion

L. nucleolus

M. peroxisome

N. plasmalemma-E face or surface

Page 88: Review Questions for Human Histology

0. plasmalemma-P face or surface

P. fixed ribosome

Q. free ribosome

2.125 Clathrin is associated with which of the following?

A. DNA-nuclear

B. DNA-mitochondrial

C. rough endoplasmic reticulum (RER)

D. smooth endoplasmic reticulum (SER)

E. filament-intermediate

F. filament-micro (microfilament)

G. Golgi apparatus

H. lysosome-primary

1. lysosome-secondary

J. microtubule

K. mitochondrion

L. nucleolus

M. peroxisome

N. plasmalemma-E face or surface

0. plasmalemma-P face or surface

P. fixed ribosome

Q. free ribosome

1. is correct.

Lysosomes are involved in both heterophagic and autophagic activities. When aprimary lysosome fuses with a heterophagic vesicle or with an intracellular

Page 89: Review Questions for Human Histology

organelle such as a mitochondrion, it becomes a secondary lysosome.

0. is correct.

Clathrin is a protein which is found associated with coated pits in receptor-mediated endocytosis. It is concentrated on the cytoplasmic or P surface of theplasmalemma. When a molecule or tiny particle binds to its appropriate receptorswhich are clustered together forming a coated pit, a coated endocytotic vesicleforms. Clathrin coats the P surface of a coated vesicle and is somehow involved inthe formation of the coated endocytotic vesicle. Once the vesicle is formed,however, clathrin is removed from the outside of the vesicle by an uncoatingenzyme.

2.126 Where is porin, a major integral protein, found?

A. DNA-nuclear

B. DNA-mitochondrial

C. rough endoplasmic reticulum (RER)

D. smooth endoplasmic reticulum (SER)

E. filament-intermediate

F. filament-micro (microfilament)

G. Golgi apparatus

H. lysosome-primary

1. lysosome-secondary

J. microtubule

K. mitochondrion

L. nucleolus

M. peroxisome

N. plasmalemma-E face or surface

0. plasmalemma-P face or surface

P. fixed ribosome

Page 90: Review Questions for Human Histology

Q. free ribosome

2.127 Where does epidermal growth factor bind with its receptor?

A. DNA-nuclear

B. DNA-mitochondrial

C. rough endoplasmic reticulum (RER)

D. smooth endoplasmic reticulum (SER)

E. filament-intermediate

F. filament-micro (microfilament)

G. Golgi apparatus

H. lysosome-primary

1. lysosome-secondary

J. microtubule

K. mitochondrion

L. nucleolus

M. peroxisome

N. plasmalemma-E face or surface

0. plasmalemma-P face or surface

P. fixed ribosome

Q. free ribosome

K. is correct.

Porin is an integral membrane protein which can form channels through whichmolecules less than a MW of 10,000 can freely pass. In the human, the outermitochondrial membrane contains porin. In bacteria porin is found in the outermembrane. Since some authors favor the hypothesis that mitochondria aremodified bacteria that have taken up a symbiotic relationship inside of cells, it iseasier to see the "porin connection" between mitochondria and bacteria.

Page 91: Review Questions for Human Histology

N. is correct.

The E surface, not face, is the part of the plasmalemma exposed to theextracellular environment. On this surface of the plasmalemma one would findreceptors for all kinds of appropriate hormones, including growth factors.

2.128 Which of the following is most closely associated with digestion ofmitochondrial cristae by acid hydrolases?

A. DNA-nuclear

B. DNA-mitochondrial

C. rough endoplasmic reticulum (RER)

D. smooth endoplasmic reticulum (SER)

E. filament-intermediate

F. filament-micro (microfilament)

G. Golgi apparatus

H. lysosome-primary

I. lysosome-secondary

J. microtubule

K. mitochondrion

L. nucleolus

M. peroxisome

N. plasmalemma-E face or surface

0. plasmalemma-P face or surface

P. fixed ribosome

Q. free ribosome

2.129 Which of the following is replicated during S phase of cell cycle?

A. DNA-nuclear

Page 92: Review Questions for Human Histology

B. DNA-mitochondrial

C. rough endoplasmic reticulum (RER)

D. smooth endoplasmic reticulum (SER)

E. filament-intermediate

F. filament-micro (microfilament)

G. Golgi apparatus

H. lysosome-primary

I. lysosome-secondary

J. microtubule

K. mitochondrion

L. nucleolus

M. peroxisome

N. plasmalemma-E face or surface

0. plasmalemma-P face or surface

P. fixed ribosome

Q. free ribosome

I. is correct.

This would be an example of a secondary lysosome involved in autophagic, notheterophagic, activity. The lysosomal enzymes gain access to the structures of the"worn-out" mitochondrion when the membranes of the lysosome and the"defective" mitochondrion fuse during this autophagic activity.

A. is correct.

Nuclear DNA is replicated only during the S phase of the cell cycle. MitochondrialDNA is replicated throughout the entire length of the cell cycle.

2.130 In which of the following is catalase almost 40% of the total protein?

A. DNA-nuclear

Page 93: Review Questions for Human Histology

B. DNA-mitochondrial

C. rough endoplasmic reticulum (RER)

D. smooth endoplasmic reticulum (SER)

E. filament-intermediate

F. filament-micro (microfilament)

G. Golgi apparatus

H. lysosome-primary

I. lysosome-secondary

J. microtubule

K. mitochondrion

L. nucleolus

M. peroxisome

N. plasmalemma-E face or surface

0. plasmalemma-P face or surface

P. fixed ribosome

Q. free ribosome

2.131 In which of the following does telomerase replicate repeated sequencesenriched with cytosine and guanine (telomeres)?

A. DNA-nuclear

B. DNA-mitochondrial

C. rough endoplasmic reticulum (RER)

D. smooth endoplasmic reticulum (SER)

E. filament-intermediate

F. filament-micro (microfilament)

Page 94: Review Questions for Human Histology

G. Golgi apparatus

H. lysosome-primary

1. lysosome-secondary

J. microtubule

K. mitochondrion

L. nucleolus

M. peroxisome

N. plasmalemma-E face or surface

0. plasmalemma-P face or surface

P. fixed ribosome

Q. free ribosome

M. is correct.

Catalase is found in peroxisomes in significant quantity/concentration. It breaksdown the hydrogen peroxide formed in the peroxisome as a result of oxidativeactivity.

A. is correct.

The ends of each chromosome have a special sequence called the telomere. Thisregion protects the ends of the chromosomes and gives them individual integrity.

2.132 Where are most proteins of oxidative phosphorylation coded for?

A. DNA-nuclear

B. DNA-mitochondrial

C. rough endoplasmic reticulum (RER)

D. smooth endoplasmic reticulum (SER)

E. filament-intermediate

F. filament-micro (microfilament)

Page 95: Review Questions for Human Histology

G. Golgi apparatus

H. lysosome-primary

I. lysosome-secondary

J. microtubule

K. mitochondrion

L. nucleolus

M. peroxisome

N. plasmalemma-E face or surface

0. plasmalemma-P face or surface

P. fixed ribosome

Q. free ribosome

2.133 Which of the following contains a sequence element (centromere) thatinteracts with a set of proteins (kinetochore)?

A. DNA-nuclear

B. DNA-mitochondrial

C. rough endoplasmic reticulum (RER)

D. smooth endoplasmic reticulum (SER)

E. filament-intermediate

F. filament-micro (microfilament)

G. Golgi apparatus

H. lysosome-primary

I. lysosome-secondary

J. microtubule

K. mitochondrion

Page 96: Review Questions for Human Histology

L. nucleolus

M. peroxisome

N. plasmalemma-E face or surface

0. plasmalemma-P face or surface

P. fixed ribosome

Q. free ribosome

A. is correct.

Mitochondrial DNA does not have enough bases in it to code for all of the proteinsand ribosomes and tRNA molecules which are normal components of themitochondrion. Mitochondrial DNA codes for mitochondrial rRNA, mitochondrialtRNA and just a few of the respiratory chain enzymes localized on the innermitochondrial membrane. Everything else in the mitochondrion is coded for bynuclear DNA.

A. is correct.

The centromere region of each chromosome is a specific nucleotide sequence ofchromosomal DNA which can be "stained" with fluorescein tagged monoclonalantibodies. Each centromere region of a chromosome plays a significant role in theattachment of the kinetochore.

2.134 Which of the following is associated with hydrogen peroxide?

A. DNA-nuclear

B. DNA-mitochondrial

C. rough endoplasmic reticulum (RER)

D. smooth endoplasmic reticulum (SER)

E. filament-intermediate

F. filament-micro (microfilament)

G. Golgi apparatus

H. lysosome-primary

I. lysosome-secondary

Page 97: Review Questions for Human Histology

J. microtubule

K. mitochondrion

L. nucleolus

M. peroxisome

N. plasmalemma-E face or surface

0. plasmalemma-P face or surface

P. fixed ribosome

Q. free ribosome

2.135 True or false: the nucleolus is membrane-bound.

A. True

B. False

2.136 True or false: the primary lysosome is membranebound.

A. True

B. False

2.137 True or false: the mitochondrion is membranebound.

A. True

B. False

2.138 True or false: the absorbed lipid is membranebound.

A. True

B. False

2.139 True or false: the chromosome is membrane-bound.

A. True

B. False

Page 98: Review Questions for Human Histology

M. is correct.

Hydrogen peroxide accumulates within peroxisomes as a result of oxidativeactivity of its oxidase enzymes, namely catalase.

B. is correct.

The nucleolus itself is not membrane bound, even through it resides in the nucleus,which is membranebound.

A. is correct.

The primary lysosome, or the lysosome which has not yet fused with somethingelse from within the cell (autophagic activity) or from outside the cell (heterophagicactivity), is a membrane-bound product of the mature face of the Golgi apparatus.

A. is correct.

The mitochondrion is a membrane-bound organelle. In fact it has two membranes:an outer membrane and an inner membrane. It is the inner mitochondrialmembrane that is folded into folds, the cristae.

A. is correct.

Lipid absorbed through the apical plasmalemma, as is the case for the enterocytelining the small intestine, must be membrane-bound, i.e. the plasmalemma forms avesicle during this endocytotic event.

B. is correct.

Chromosomes are not individually membrane-bound structures. The fact that theyare contained within the nuclear membrane during the interphase of the cell cycledoes not classify them as membrane-bound structures.

2.140 True or false: the secondary lysosome is membranebound.

A. True

B. False

2.141 True or false: the lipid synthesized de novo in cytoplasm is membrane-bound.

A. True

B. False

Page 99: Review Questions for Human Histology

2.142 True or false: the peroxisome is membrane-bound.

A. True

B. False

2.143 True or false: glycogen inclusions are membranebound.

A. True

B. False

2.144 True or false: the secretory granule is membranebound.

A. True

B. False

2.145 True or false: microtubules are membrane-bound.

A. True

B. False

2.146 True or false: desmin containing intermediate filaments is membrane-bound.

A. True

B. False

2.147 True or false: the actin containing microfilaments are membrane-bound.

A. True

B. False

2.148 True or false: the microtubule organizing center (MTOC) is membrane-bound.

A. True

B. False

2.149 True or false: condensing vacuoles at the trans-face of the Golgi body aremembrane-bound.

A. True

Page 100: Review Questions for Human Histology

B. False

A. is correct.

A secondary lysosome is like a primary lysosome in that it is packaged by theGolgi apparatus and therefore is covered by a membrane. When a primarylysosome fuses with a phagosome or autophagic vacuole, the membranes of thelysosome fuse with those of the phagosome or autophagic vacuole, but the endresult is still a membrane-bound structure.

B. is correct.

Lipid synthesized on the spot or de novo in the cytoplasm is not membrane-bound.

A. is correct.

Peroxisomes are lysosome-like organelles which are packaged (given theirmembrane) as they exit the mature face of the Golgi apparatus.

B. is correct.

Intracellular cytoplasmic deposits of glycogen are not membrane-bound.

A. is correct.

Proteins, glycoproteins, etc. packaged by the Golgi apparatus are covered by aGolgi membrane as they leave the mature face of the Golgi body.

B. is correct.

Microtubules are components of the cytoskeleton and are not membrane-bound.

B. is correct.

Intermediate filaments are not membrane bound.

B. is correct.

These microfilaments are components of the cytoskeleton and are not membrane-bound.

B. is correct.

This structure is not membrane-bound, it exists "free" in the cytoplasm.

A. is correct.

Page 101: Review Questions for Human Histology

Since the trans-face is the mature face where vacuole/granule packaging iscomplete, these vacuoles would be membrane-bound.

2.150 True or false: the specific granules in cytoplasm of neutrophil are membrane-bound.

A. True

B. False

A. is correct.

The specific granules in a PMN are similar, if not identical, to lysosomes andtherefore, are packaged by the Golgi apparatus and are membrane bound.

Page 102: Review Questions for Human Histology

SECTION 3: EPITHELIAL TISSUE

3.001 All of the following are components of the most common epithelial junctionalcomplex EXCEPT:

A. zonula occludens

B. macula adherens

C. zonula adherens

D. fascia adherens

E. intermediate filaments of the cytokeratin type

3.002 As you look at a slide you see secretory units or groups of cells withsecretory granules in their cytoplasm and simple cuboidal to columnar cells lininga variety of different sized tubular structures. Identify this section.

A. endocrine gland

B. compound acinar-alveolar gland

C. straight tubular gland

D. unicellular gland

E. none of the above are correct

3.003 Which type of sheet-like epithelium helps form a serosa?

A. simple columnar

B. simple cuboidal

C. simple squamous

D. stratified cuboidal

E. transitional

3.004 Which of the following does NOT apply to microvilli?

A. associated with glycocalyx

Page 103: Review Questions for Human Histology

B. actin containing microfilaments

C. brush/striated border

D. tubulin

E. increased surface area

3.005 In which of the following sheet-like epithelia do all cells touch the basementmembrane but not all cells reach or border the lumen.

A. transitional

B. stratified squamous non-keratinizing

C. stratified squamous keratinizing

D. pseudostratified columnar

E. stratified cuboidal as in the duct of a sweat gland

D. is correct.

The classical epithelial junctional complex from the luminal or apical side of theepithelium toward basal lamina is ZO-ZA-MA. Cytokeratin intermediate filamentsare components of the MA. The fascia adherens is not a component of this mostcommon type of junction between epithelial cells.

B. is correct.

The sheet-like epithelium lining the tubes indicates a duct system. Since manyducts can be seen the duct system is branched or compound. Secretory units oracini/alveoli of the serous type are indicated by the presence of secretory granulesin the cytoplasm.

C. is correct.

A serosa such as that of the visceral and parietal pleura is composed of a sheet ofsimple squamous cells or mesothelium with some submesothelial connectivetissue. A neoplasm of this epithelium is called a mesothelioma.

D. is correct.

Tubulin is the protein component of microtubules which are found in cilia andflagella, not in microvilli.

D. is correct.

Page 104: Review Questions for Human Histology

The question actually is a description or definition of a pseudostratified epithelium.

3.006 When one adult sheet-like epithelium is replaced by another adult sheet-likeepithelium the process is called:

A. hyperplasia

B. hypertrophy

C. dysplasia

D. metaplasia

E. anaplasia

3.007 As a glandular epithelial cell secretes its product small pieces of the apicalcytoplasm of the cell are released as a component of the secretory product. Whichof the following correctly applies to this situation?

A. holocrine secretion

B. paracrine secretion

C. endocrine secretion

D. merocrine secretion

E. apocrine secretion

3.008 What would you expect to see lots of under the electron microscope in thecytoplasm of a cell that secretes a steroid hormone?

A. RER

B. SER

C. secretory vesicles

D. lysosomes

E. residual bodies

3.009 In a stratified, non-keratinized wet epithelium such as that which lines theesophagus or the vagina, the basal cells resting on the basement membrane livewhich type of mitotic or cell cycle life?

Page 105: Review Questions for Human Histology

A. vegetative intermitotic

B. differentiating intermitotic

C. reverting postmitotic

D. fixed postmitotic

3.010 Identify the epithelium: the superficial cells contain cytoplasmic areas offolded cell membrane forming vesicles which represent a reservoir of cellmembrane?

A. stratified squamous keratinizing

B. stratified squamous non-keratinizing

C. stratified columnar

D. transitional

E. stratified cuboidal

D. is correct.

Metaplasia is the process in which one adult epithelium is replaced by a differentadult epithelium. The most common case is the change from pseudostratifiedciliated columnar to stratified squamous non-keratinizing epithelium in the tracheaand bronchi as a result of exposure to cigarette smoke.

E. is correct.

Holocrine secretion is characterized by the death of the entire cell. Merocrine is theusual form of secretion in which the product is exocytosed. Apocrine is when a partof the apical region of the cytoplasm actually is a component of the secretoryproduct. Most hormones whether they are autocrine, paracrine or endocrine arereleased via the merocrine route of cell secretion.

B. is correct.

The enzymes necessary to synthesize steroids from precursor molecules arelocated in the smooth endoplasmic reticulum.

A. is correct.

These are the stems cells for this epithelium. These vegetative intermitotics divideand produce daughter cells which (1) can remain attached to the basementmembrane and continue to function as vegetative intermitotics or (2) can leave the

Page 106: Review Questions for Human Histology

basal layer and move into more superficial layers where they begin to undergosome cytodifferentiation or cell specialization but they still retain (remember?) thecapability to run the cell cycle and divide. In this case these cells can be classifiedas differentiating intermitotics. As their daughter cells continue to differentiate andmove toward the surface of the epithelium they eventually, progressively lose thecapability to divide and can then be classified as fixed postmitotics.

D. is correct.

This is a characteristic feature of transitional epithelium which allows theepithelium to expand and radically change shape as the urinary bladder fills.

3.011 The following description applies to which structure: an anastomosingsystem of ridges extends around the entire perimeter of the cells involved, theridges are composed of integral membrane proteins which interlock with integralmembrane proteins of opposing ridges.

A. macula adherens

B. zonula adherens

C. macula occludens

D. zonula occludens

E. nexus

3.012 Which of the following cell junctions or attachment sites has a marginal bandof microfilaments anchoring to the cell membrane?

A. macula adherens

B. zonula adherens

C. macula occludens

D. zonula occludens

E. nexus

3.013 Which of the following cell junctions or attachment sites has tonofilamentsanchored into disc-like plaques of electron-dense material.

A. macula adherens

B. zonula adherens

Page 107: Review Questions for Human Histology

C. macula occludens

D. zonula occludens

E. nexus

3.014 Which of the following cell junctions or attachment sites has tubularchannels constructed of transmembrane proteins from two adjacent cellmembranes?

A. macula adherens

B. zonula adherens

C. macula occludens

D. zonula occludens

E. nexus

3.015 In which of the following cell junctions are adjacent cells in ionic contact orcontinuity?

A. macula adherens

B. zonula adherens

C. macula occludens

D. zonula occludens

E. nexus

D. is correct.

When the integral membrane proteins on opposing ridges of two adjacent cellsinterlock the intercellular space is obliterated and thus molecules are preventedfrom passing into the narrow intercellular space beyond the occludens jct.

B. is correct.

The zonula adherens or the adhesion belt is the site where bundles ofmicrofilaments, not keratincontaining tonofilaments, are attached to the cellmembrane.

A. is correct.

Page 108: Review Questions for Human Histology

The macula adherens or desmosome is a spot attachment site in whichintermediate filaments are attached or hooked into an electron dense plaque on thecytoplasmic side of the cell membrane.

E. is correct.

The cap or nexus junction is an array of tubular channels that span the gap orintercellular space between two cells. The cylindrical walls of these tiny "straws"are called connexons.

E. is correct.

It is through the gap or nexus junction that adjacent cells are connectedphysiologically to form an ionic syncytium. Waves of excitation almostsimultaneously pass through gap junctions.

3.016 The following description correctly applies to which structure: 20-30 bundledactin filaments run parallel to the long axis and these filaments are bundled fimbrinand villin.

A. cilia

B. microvilli

C. flagella

D. desmosome

E. basal lamina substructure

3.016 For which of the following does the following statement apply: underneaththe plasma membrane there is a dense plaque into which tonofilaments areconnected.

A. desmosome

B. zonula

C. zonula occludens

D. gap junction

3.017 For which of the following does the following statement apply: ions passquickly through here.

A. desmosome

Page 109: Review Questions for Human Histology

B. zonula adherens

C. zonula occludens

D. gap junction

3.018 For which of the following does the following statement apply: outer bilayerhalves of adjacent two cell membranes are fused.

A. desmosome

B. zonula adherens

C. zonula occludens

D. gap junction

3.019 For which of the following does the following statement apply: theextracellular space is eliminated here.

A. desmosome

B. zonula adherens

C. zonula occludens

D. gap junction

3.020 For which of the following does the following statement apply: found in highconcentration in the stratum spinosum of epidermis.

A. desmosome

B. zonula adherens

C. zonula occludens

D. gap junction

B. is correct.

Bundles of actin filaments form the scaffolding or internal support structure ofmicrovilli.

A. is correct.

This describes the desmosome. It has the most dense plaque of all the other cell

Page 110: Review Questions for Human Histology

junctions listed and from this plaque radiate tonofilaments.

D. is correct.

This is the hallmark of the gap or nexus junction, i.e. the cells so connected form anionic and electrical syncytium. Gap junctions are found, for example, in theintercalated discs of cardiac muscle.

C. is correct.

This is a description of what happens when two adjacent cells form a tight junction.

C. is correct.

Since the outer leaflets of the two opposing cell membranes actually fuse in anoccludens junction, the extracellular space is obliterated here.

A. is correct.

The spiny layer of the epidermis, the stratum spinosum, is composed ofkeratinocytes which have large numbers of desmosomes on their cell surface.When this tissue is fixed in something like formalin, some shrinkage occurs and thecells shrink away from each other in the areas where they are not stuck together bydesmosomes. This gives this layer its spiny or prickle cell appearance.

3.021 For which of the following does the following statement apply: lateralmovement of integral membrane proteins completely blocked.

A. desmosome

B. zonula adherens

C. zonula occludens

D. gap junction

3.022 For which of the following does the following statement apply: actinmicrofilaments extend from a submembrane plaque.

A. desmosome

B. zonula adherens

C. zonula occludens

D. gap junction

Page 111: Review Questions for Human Histology

3.023 For which of the following does the following statement apply: 4 to 6 units ofconnexin form a connexon.

A. desmosome

B. zonula adherens

C. zonula occludens

D. gap junction

3.024 For which of the following does the following statement apply: thesubcellular plaque material has a high concentration of vinculin.

A. desmosome

B. zonula adherens

C. zonula occludens

D. gap junction

3.026 Which of the following lines the trachea?

A. simple squamous epithelium

B. simple cuboidal epithelium

C. simple columnar epithelium

D. stratified squamous keratinizing epithelium

E. stratified squamous non-keratinizing epithelium

F. stratified cuboidal epithelium

G. stratified columnar epithelium

H. pseudostratified columnar epithelium

I. transitional epithelium

C. is correct.

In the zonula occludens or tight junction, the outer layers of each of the twoopposing cell membranes are fused along ridges. This fusion site eliminates theextracellular space and blocks the movement of membrane proteins and lipids.

Page 112: Review Questions for Human Histology

B. is correct.

Although the region just under the plasmalemma in both the desmosome and thezonula adherens is thickened to form a plaque, the plaques associated withadherens junctions are not as dense as those of the desmosome. In thedesmosome, anchoring tonofilaments radiate from the plaque into the cytoplasm. Inthe adherens junction, fibers much thinner than tonofilaments radiate from the "notso dense" plaque. These thinner microfilaments are actin microfilaments.

D. is correct.

A gap junction is a small circle of 4-6 subunits of a protein called connexin.Connexins vary extensively from tissue to tissue.

B. is correct.

Adherens junctions contain significant amounts of the actin-binding protein vinculinwhich is involved in linking the plasmalemma to the actin microfilaments of thecytoskeleton.

H. is correct.

The "respiratory epithelium" is a pseudostratified columnar epithelium, with severaltypes of cells in it. One type of cell in this epithelium contains cilia.

3.027 Which of the following lines the small intestine?

A. simple squamous epithelium

B. simple cuboidal epithelium

C. simple columnar epithelium

D. stratified squamous keratinizing epithelium

E. stratified squamous non-keratinizing epithelium

F. stratified cuboidal epithelium

G. stratified columnar epithelium

H. pseudostratified columnar epithelium

1. transitional epithelium

3.028 Which of the following lines the duct of a sweat gland in the dermis of the

Page 113: Review Questions for Human Histology

integument?

A. simple squamous epithelium

B. simple cuboidal epithelium

C. simple columnar epithelium

D. stratified squamous keratinizing epithelium

E. stratified squamous non-keratinizing epithelium

F. stratified cuboidal epithelium

G. stratified columnar epithelium

H. pseudostratified columnar epithelium

I. transitional epithelium

3.029 Which of the following lines the thoracic duct?

A. simple squamous epithelium

B. simple cuboidal epithelium

C. simple columnar epithelium

D. stratified squamous keratinizing epithelium

E. stratified squamous non-keratinizing epithelium

F. stratified cuboidal epithelium

G. stratified columnar epithelium

H. pseudostratified columnar epithelium

I. transitional epithelium

3.030 Which of the following lines the minor calyx?

A. simple squamous epithelium

B. simple cuboidal epithelium

C. simple columnar epithelium

Page 114: Review Questions for Human Histology

D. stratified squamous keratinizing epithelium

E. stratified squamous non-keratinizing epithelium

F. stratified cuboidal epithelium

G. stratified columnar epithelium

H. pseudostratified columnar epithelium

I. transitional epithelium

C. is correct.

The entire lining of the small intestine is simple columnar epithelium.

F. is correct.

Stratified cuboidal epithelium is not very common. The ducts of the eccrine sweatglands of the integument, however, are lined by it.

A. is correct.

The thoracic duct, as well as all other lymphatic ducts, as well as all blood vessels,is lined by a simple squamous epithelium or endothelium.

I. is correct.

The minor and major calyces are lined by transitional epithelium, as are the ureterand the urinary bladder.

3.031 Which of the following lines the major collecting ducts in kidney?

A. simple squamous epithelium

B. simple cuboidal epithelium

C. simple columnar epithelium

D. stratified squamous keratinizing epithelium

E. stratified squamous non-keratinizing epithelium

F. stratified cuboidal epithelium

G. stratified columnar epithelium

Page 115: Review Questions for Human Histology

H. pseudostratified columnar epithelium

1. transitional epithelium

3.032 What tissue type covers the surface of the ovary?

A. simple squamous epithelium

B. simple cuboidal epithelium

C. simple columnar epithelium

D. stratified squamous keratinizing epithelium

E. stratified squamous non-keratinizing epithelium

F. stratified cuboidal epithelium

G. stratified columnar epithelium

H. pseudostratified columnar epithelium

1. transitional epithelium

3.033 What tissue type covers the surface of the parietal layer of Bowman'scapsule of the kidney?

A. simple squamous epithelium

B. simple cuboidal epithelium

C. simple columnar epithelium

D. stratified squamous keratinizing epithelium

E. stratified squamous non-keratinizing epithelium

F. stratified cuboidal epithelium

G. stratified columnar epithelium

H. pseudostratified columnar epithelium

I. transitional epithelium

3.034 What tissue type covers the surface of the true vocal fold of the larynx?

Page 116: Review Questions for Human Histology

A. simple squamous epithelium

B. simple cuboidal epithelium

C. simple columnar epithelium

D. stratified squamous keratinizing epithelium

E. stratified squamous non-keratinizing epithelium

F. stratified cuboidal epithelium

G. stratified columnar epithelium

H. pseudostratified columnar epithelium

1. transitional epithelium

C. is correct.

The large collecting ducts in the medullary regions of the kidney are lined bysimple columnar cells which require the presence of antidiuretic hormone (ADH)for the passage of water from the lumen of the collecting duct into the surroundinghypertonic interstitium.

B. is correct.

The ovary surface is covered by a simple cuboidal to fat squamous epithelium. Atone point in time it was thought that the oogonia were derived from this epitheliumand it still carries the misnomer "germinal epithelium".

A. is correct.

The visceral layer of Bowman's capsule is composed of the podocytes. Theparietal layer of Bowman's capsule is composed of simple squamous epithelium.

E. is correct.

The surface of the true vocal folds are covered by a stratified squamous epitheliumwhich is reflective of the function of these folds in the production of sounds.

3.035 Which of the following is found in patches in the female urethra?

A. simple squamous epithelium

B. simple cuboidal epithelium

Page 117: Review Questions for Human Histology

C. simple columnar epithelium

D. stratified squamous keratinizing epithelium

E. stratified squamous non-keratinizing epithelium

F. stratified cuboidal epithelium

G. stratified columnar epithelium

H. pseudostratified columnar epithelium

I. transitional epithelium

3.036 What tissue type lines the epididymis?

A. simple squamous epithelium

B. simple cuboidal epithelium

C. simple columnar epithelium

D. stratified squamous keratinizing epithelium

E. stratified squamous non-keratinizing epithelium

F. stratified cuboidal epithelium

G. stratified columnar epithelium

H. pseudostratified columnar epithelium

1. transitional epithelium

3.037 What tissue type covers the surface of the visceral pleura?

A. simple squamous epithelium

B. simple cuboidal epithelium

C. simple columnar epithelium

D. stratified squamous keratinizing epithelium

E. stratified squamous non-keratinizing epithelium

F. stratified cuboidal epithelium

Page 118: Review Questions for Human Histology

G. stratified columnar epithelium

H. pseudostratified columnar epithelium

I. transitional epithelium

3.038 What tissue type lines the fossa navicularis of the penis?

A. simple squamous epithelium

B. simple cuboidal epithelium

C. simple columnar epithelium

D. stratified squamous keratinizing epithelium

E. stratified squamous non-keratinizing epithelium

F. stratified cuboidal epithelium

G. stratified columnar epithelium

H. pseudostratified columnar epithelium

I. transitional epithelium

G. is correct.

Stratified columnar epithelium is rare in the human. However, patches of this typeof epithelium can be found in both the male and the female urethras.

H. is correct.

The epididymis is lined by a pseudostratified columnar epithelium. A specialfeature here is the tuft of microvilli on each columnar cell giving the namestereocilia to these structures.

A. is correct.

The visceral and parietal pleura are serous membranes, just like the comparablevisceral and parietal peritoneum. The surfaces of these membranes are covered bya simple squamous epithelium given the special name of mesothelium.Mesothelioma is a neoplasm of these cells.

E. is correct.

The epithelial lining of the penile urethra, proximal to the junction with the skin over

Page 119: Review Questions for Human Histology

the glans penis, changes to a stratified squamous wet epithelium.

3.039 What tissue type lines the thyroid follicles under TSH stimulation?

A. simple squamous epithelium

B. simple cuboidal epithelium

C. simple columnar epithelium

D. stratified squamous keratinizing epithelium

E. stratified squamous non-keratinizing epithelium

F. stratified cuboidal epithelium

G. stratified columnar epithelium

H. pseudostratified columnar epithelium

1. transitional epithelium

3.040 What tissue type lines the common bile duct?

A. simple squamous epithelium

B. simple cuboidal epithelium

C. simple columnar epithelium

D. stratified squamous keratinizing epithelium

E. stratified squamous non-keratinizing epithelium

F. stratified cuboidal epithelium

G. stratified columnar epithelium

H. pseudostratified columnar epithelium

1. transitional epithelium

3.041 What tissue type covers the external surface of the tympanic membrane?

A. simple squamous epithelium

B. simple cuboidal epithelium

Page 120: Review Questions for Human Histology

C. simple columnar epithelium

D. stratified squamous keratinizing epithelium

E. stratified squamous non-keratinizing epithelium

F. stratified cuboidal epithelium

G. stratified columnar epithelium

H. pseudostratified columnar epithelium

I. transitional epithelium

3.042 What tissue type covers the anterior surface of the lens?

A. simple squamous epithelium

B. simple cuboidal epithelium

C. simple columnar epithelium

D. stratified squamous keratinizing epithelium

E. stratified squamous non-keratinizing epithelium

F. stratified cuboidal epithelium

G. stratified columnar epithelium

H. pseudostratified columnar epithelium

I. transitional epithelium

C. is correct.

Without stimulation by thyroid stimulating hormone the epithelial lining of thethyroid follicles is of a cuboidal to fat squamous type. However, under TSHstimulation the normally cuboidal/fat squamous epithelium is stimulated to "newheights" for the production of colloid and/or the uptake of colloid with enzymaticprocessing to the thyroid hormones tri-and tetra-iodothyronine.

C. is correct.

The hepatic duct, cystic duct and common bile duct are lined by a simple columnarepithelium.

Page 121: Review Questions for Human Histology

D. is correct.

The external surface of the tympanic membrane is skin.

B. is correct.

The anterior surface of the lens is covered by a simple cuboidal epithelium. Thereis no such epithelium on the posterior surface of the lens. The exterior of the lens iscoated with a homogeneous layer of collagen and proteoglycans called thecapsule of the lens.

3.043 What tissue type lines the uterine cavity?

A. simple squamous epithelium

B. simple cuboidal epithelium

C. simple columnar epithelium

D. stratified squamous keratinizing epithelium

E. stratified squamous non-keratinizing epithelium

F. stratified cuboidal epithelium

G. stratified columnar epithelium

H. pseudostratified columnar epithelium

I. transitional epithelium

3.044 What tissue type lines the majority of the penile urethra?

A. simple squamous epithelium

B. simple cuboidal epithelium

C. simple columnar epithelium

D. stratified squamous keratinizing epithelium

E. stratified squamous non-keratinizing epithelium

F. stratified cuboidal epithelium

G. stratified columnar epithelium

Page 122: Review Questions for Human Histology

H. pseudostratified columnar epithelium

I. transitional epithelium

3.045 What tissue type lines the vagina?

A. simple squamous epithelium

B. simple cuboidal epithelium

C. simple columnar epithelium

D. stratified squamous keratinizing epithelium

E. stratified squamous non-keratinizing epithelium

F. stratified cuboidal epithelium

G. stratified columnar epithelium

H. pseudostratified columnar epithelium

I. transitional epithelium

3.046 Which of the following best describes the sublingual gland?

A. simple tubular gland

B. simple coiled tubular gland

C. simple branched tubular gland

D. simple branched alveolar gland

E. compound tubular or acinar gland

3.047 Which of the following best describes the sweat gland of skin?

A. simple tubular gland

B. simple coiled tubular gland

C. simple branched tubular gland

D. simple branched alveolar gland

E. compound tubular or acinar gland

Page 123: Review Questions for Human Histology

C. is correct.

The epithelium lining the uterine cavity is a simple columnar type of epitheliumwhich contains columnar cells which are ciliated and columnar cells which aresecretory (mucus).

H. is correct.

The penile urethra is lined mostly by pseudostratified columnar epithelium. At thedilated end of the penile urethra, the fossa navicularis, the epithelium changes to astratified squamous wet type of epithelium.

E. is correct.

The epithelial lining of the vagina is stratified squamous wet (non-keratinizing).Other organs where such a tough, wet epithelium (stratified squamous) is foundare: esophagus and lower anal canal.

E. is correct.

The sublingual gland is composed of nearly 100% mucous acini secreting to abranched duct system, hence the best description is a compound acinar gland.

B. is correct.

Since the duct of the sweat gland is not branched it can not be termed compoundor branched. Since both the duct and the secretory portion of the sweat gland arehighly coiled tubular structures, the term simple coiled tubular gland applies.

3.048 Which of the following best describes the submandibular gland?

A. simple tubular gland

B. simple coiled tubular gland

C. simple branched tubular gland

D. simple branched alveolar gland

E. compound tubular or acinar gland

3.049 Which of the following best describes the colonic crypt?

A. simple tubular gland

B. simple coiled tubular gland

Page 124: Review Questions for Human Histology

C. simple branched tubular gland

D. simple branched alveolar gland

E. compound tubular or acinar gland

3.050 Which of the following best describes the gastric pyloric gland?

A. simple tubular gland

B. simple coiled tubular gland

C. simple branched tubular gland

D. simple branched alveolar gland

E. compound tubular or acinar gland

3.051 Which of the following best describes the sebaceous gland in skin?

A. simple tubular gland

B. simple coiled tubular gland

C. simple branched tubular gland

D. simple branched alveolar gland

E. compound tubular or acinar gland

3.052 Which of the following best describes the parotid gland?

A. simple tubular gland

B. simple coiled tubular gland

C. simple branched tubular gland

D. simple branched alveolar gland

E. compound tubular or acinar gland

E. is correct.

This gland has both serous and mucous acini in it which are drained by a ductsystem which branches tremendously. Therefore, it is a compound acinar glandwith a mixed secretory production (mucus and watery fluid containing proteins).

Page 125: Review Questions for Human Histology

A. is correct.

The straight glands of the mucosa of the colon are examples of simple (nobranching of the duct system) tubular glands.

C. is correct.

The gastric pit/gastric gland arrangement is an example of a tubular gland whichhas several divisions or branches opening into a common lumen (the gastric pit).

D. is correct.

Sebaceous glands secrete by the holocrine method and this secretory product(entire dead cell) is delivered to the hair shaft through a simple, branched "duct".

E. is correct.

Since the main duct of the parotid gland branches frequently as it drains thesecretory acini, it is a compound gland.

Page 126: Review Questions for Human Histology

SECTION 4: CONNECTIVE TISSUE

4.001 Which component of the LM basement membrane at the interface betweenan epithelium and its underlying connective tissue is synthesized by theconnective tissue cells deep to the basement membrane?

A. lamina lucida

B. epithelial glycocalyx

C. lamina reticularis

D. basal lamina proper

E. hemidesmosome

4.002 All of the following are components of the extracellular matrix except:

A. tissue fluid

B. collagen fibers

C. glycosaminoglycans

D. antibodies

E. plasma cells

4.003 All of the following are glycosaminoglycans EXCEPT:

A. chondroitin sulfate

B. dermatan sulfate

C. hyaluronic acid

D. desmin

E. keratan sulfate

4.004 All of the following are structural glycoproteins found in connective tissueEXCEPT:

A. chondronectin

Page 127: Review Questions for Human Histology

B. laminin

C. hyaluronidase

D. fibronectin

4.005 During the formation of tissue fluid, some but not all, of the tissue fluid isdrawn back in to the venous side of the capillary because?

A. the hydrostatic pressure is higher than the osmotic pressure

B. the osmotic pressure is higher than the hydrostatic pressure

C. lymph capillaries are directly connected to the venous side of a capillary bed

D. lymph drainage of the area creates a negative pressure in the lymph capillariesdrawing some tissue fluid into their lumens.

E. endothelial cells of the post-capillary veins are impermeable to tissue fluid

C. is correct.

The lamina reticularis or reticular lamina is the only component synthesized byconnective tissue cells, all of the other components listed are derived from theepithelium.

E. is correct.

Cells found in connective tissue are embedded within the extracellular matrix butare not considered to be a part of it. All of the other choices are components of theextracellular matrix.

D. is correct.

Desmin is found in intermediate filaments of muscle cells, it is not aglycosaminoglycan.

C. is correct.

Hyaluronidase is an enzyme produced by some bacteria and cancer cells whichdigests the glycosaminoglycan hyaluronic acid and causes a loss of viscosity ofthe connective tissue. All other choices are structural proteins in connective tissue.

B. is correct.

The osmotic pressure exerted by the solutes and large molecules in the blood onthe venous side of a capillary bed is relatively higher than the hydrostatic pressure,

Page 128: Review Questions for Human Histology

thereby drawing fluid from the interstitial space into the veins.

4.006 All of the following can result in edema EXCEPT:

A. traumatized endothelium

B. longstanding malnutrition

C. increased venous hydrostatic pressure

D. obstruction of lymphatic drainage

E. decreased arterial hydrostatic pressure

4.007 Which of the following polymerizes to form collagen fibrils?

A. preprocollagen

B. procollagen

C. tropocollagen

D. hydroxyproline

E. registration peptides

4.008 All of the following are temporary or mobile residents of connective tissueEXCEPT:

A. eosinophil

B. neutrophil

C. fibrocyte

D. lymphocyte

E. monocyte

4.009 Classify the connective tissue found underlying a wet epithelial surface.

A. dense irregular

B. reticular

C. adipose

Page 129: Review Questions for Human Histology

D. loose areolar

E. mucoid

4.010 Which of the following predominates in the spleen, lymph nodes and redbone marrow?

A. elastic fibers

B. collagen type IV

C. reticular fibers

D. collagen type II

E. collagen type V

4.011 Which of the following connective tissue cells binds immunoglobulin E?

A. plasma cell

B. fibrocyte

C. mast cell

D. macrophage

E. lymphocyte

E. is correct.

A decrease in arterial hydrostatic pressure would cause less not more fluid to exitthe capillary lumen and become tissue fluid. All other choices are causes ofedema. In the case of malnutrition, tissue fluid accumulates in the peritoneal cavity(abdominal ascites) because the individual simply does not have enough largemolecules in his/her blood to exert enough of an osmotic draw to bring most of thefluid produced at the arterial end back into the venous structures.

C. is correct.

Procollagen molecules are secreted by the fibroblast. The ends of thesemolecules(the registration peptides) are clipped off by peptidases transformingprocollagen to tropocollagen. The tropocollagen molecules are fibrillar structureswhich aggregate into collagen fibrils.

C. is correct.

Page 130: Review Questions for Human Histology

The fibroblast/fibrocyte is a permanent non-mobile cellular component ofconnective tissue. All other choices are found in connective tissue areas but theyare free to come and go.

D. is correct.

All wet epithelia are "serviced" by an underlying loose areolar connective tissuewhich is also called the lamina propria.

C. is correct.

Reticular fibers are major stromal elements of the lymphoid organs.

C. is correct.

IgE is originally produced by the plasma cell but is bound to the surface of the mastcell. When a second exposure to the same antigen occurs the antigen binds to theIgE on the surface of the mast cell. This causes the mast cell to degranulatereleasing heparin, histamine, leukotrines and eosinophil chemotactic factor-anaphylaxis.

4.012 Which of the following acts outside of the cell:

A. catalase

B. lysyl oxidase

C. glycosidase

D. DNAase

E. hydroxyacid oxidase

4.013 Which of the following is not considered to be a microtubule organizingcenter or MTOC?

A. centromere region of a chromosome

B. secondary constriction of a chromosome

C. centriole

D. basal body

4.014 Which of the following is characterized by: PAS positivity and argyrophilia?

A. skeletal muscle fiber

Page 131: Review Questions for Human Histology

B. collagen type I fiber

C. elastic fiber

D. reticular fiber E. nerve fiber

4.015 The amino acids isodesmosine and desmosine are unique to which of thefollowing?

A. skeletal muscle fiber

B. collagen type I fiber

C. elastic fiber

D. reticular fiber

E. nerve fiber

4.016 Which of the following connective tissue cells would be the mostheterochromatinized.

A. fibroblast

B. fibrocyte

C. plasma cell

D. macrophage

E. multinuclear giant cells

4.017 Which of the following areas is comprised of dense, regularly arranged,white fibrous connective tissue?

A. fibrous layer of periosteum

B. tendon of biceps muscle

C. annulus fibrosus of an intervertebral disc

D. reticular layer of the dermis

E. lamina propria of intestinal villus

B. is correct.

Page 132: Review Questions for Human Histology

Lysyl oxidase cross-links tropocollagen molecules extracellularly. All otherenzymes mentioned "work" inside the cell as a result of lysosomal or peroxisomalactivity.

B. is correct.

All other choices are classified as MTOCs. The secondary constriction appears ononly 5 human chromosomes and is the location of the nucleolar organizing regionof the genome.

D. is correct.

Reticular fibers have a high content of glycoproteins associated with them and theyare composed mostly of type III collagen.

C. is correct.

These two amino acids are unusual and are formed by covalent reactions betweenlysine residues causing the rubber-like quality of elastin.

B. is correct.

The fibrocyte is the quiescent form of the more active cell the fibroblast. Thefibrocyte is "shut down" with respect to the synthesis of tropocollagen anddemonstrates a pyknotic or highly heterochromatinized nuclear chromatin pattern.

B. is correct.

Tendons, aponeuroses and ligaments are all comprised of dense, regularlyarranged connective tissue. Choices A and D represent dense, irregularlyarranged connective tissue. Choice C is an example of fibrocartilage. Choice E isan example of loose areolar connective tissue.

4.018 The dark banded regular periodicity of collagen types I, II & III seen with theelectron microscope in negatively stained preparations is directly related to all ofthe following EXCEPT:

A. stain has penetrated the gaps between collagen molecules

B. triple helices of collagen molecules line up in rows

C. collagen molecules overlap between rows by about one fourth of their length

D. collagen molecules do not overlap between rows leaving all gaps betweenmolecules in register

Page 133: Review Questions for Human Histology

4.019 All of the following diseases are diseases of collagen EXCEPT:

A. interstitial pulmonary fibrosis

B. rickets

C. scurvy

D. osteogenesis imperfecta

E. Ehlers-Danlos (rubber man) syndrome

4.020 The following description best fits which of the following: carbohydrate sidechains radiate laterally from a protein core similar to the bristles on a test tubebrush.

A. chromatin suprastructure

B. type V collagen

C. Ng-CAM (neuroglial cell adhesion molecule)

D. glycosaminoglycans (GAG)

E. hyaluronic acid

4.021 Which of the following are involved in linking cell surfaces with collagen-proteoglycan complexes?

A. fibronectin

B. laminin

C. hyaluronic acid

D. A & B are correct

E. A & B & C are correct

4.022 Which of the following best describes the primary composition of thepapillary layer of dermis?

A. dense regular connective tissue

B. dense irregular connective tissue

C. reticular connective tissue

Page 134: Review Questions for Human Histology

D. loose areolar connective tissue

E. adipose connective tissue

F. mesenchymal connective tissue

G. mucoid connective tissue

D. is correct.

The molecules do overlap between adjacent rows and wherever their is a gap itwill fill in with stain in a negatively stained preparation.

B. is correct.

Rickets is a disease related to a lack of vitamin D which is necessary for theintestinal absorption of calcium. Scurvy is the result of a lack of vitamin C which isnecessary for the synthesis of procollagen.

D. is correct.

The GAGs can be attached to hyaluronic acid molecules to form large complexeswhich can impede the movement of tissue fluid.

D. is correct.

The two most famous linker glycoproteins are fibronectin and laminin. Laminin isrestricted to basal laminae. Fibronectin is a large glycoprotein composed of fibrouspolypeptides.

D. is correct.

The papillary layer of the dermis is that layer closest to the overlying avascularepidermis which it services with a blood supply and lymphatic drainage. The typeof connective tissue found here is loose areolar.

4.023 Which of the following best describes the primary composition of the reticularlayer of dermis?

A. dense regular connective tissue

B. dense irregular connective tissue

C. reticular connective tissue

D. loose areolar connective tissue

Page 135: Review Questions for Human Histology

E. adipose connective tissue

F. mesenchymal connective tissue

G. mucoid connective tissue

4.024 Which of the following best describes the primary composition of thehypodermis?

A. dense regular connective tissue

B. dense irregular connective tissue

C. reticular connective tissue

D. loose areolar connective tissue

E. adipose connective tissue

F. mesenchymal connective tissue

G. mucoid connective tissue

4.025 Which of the following best describes the primary composition of theperineurium?

A. dense regular connective tissue

B. dense irregular connective tissue

C. reticular connective tissue

D. loose areolar connective tissue

E. adipose connective tissue

F. mesenchymal connective tissue

G. mucoid connective tissue

4.026 Which of the following best describes the primary composition of theligament?

A. dense regular connective tissue

B. dense irregular connective tissue

Page 136: Review Questions for Human Histology

C. reticular connective tissue

D. loose areolar connective tissue

E. adipose connective tissue

F. mesenchymal connective tissue

G. mucoid connective tissue

4.027 Which of the following best describes the primary composition of themesentery?

A. dense regular connective tissue

B. dense irregular connective tissue

C. reticular connective tissue

D. loose areolar connective tissue

E. adipose connective tissue

F. mesenchymal connective tissue

G. mucoid connective tissue

B. is correct.

The reticular layer of the dermis is the major supporting layer of the dermis and iscomposed of dense irregular connective tissue.

E. is correct.

The hypodermis is composed of adipose tissue. In this area the adipose tissueserves as a shock absorber as well as an insulator and calorie storehouse.

B. is correct.

The perineurium, perimysium and peritentinium are comprised of dense irregularlyarranged fibrous connective tissue.

A. is correct.

All ligaments are composed of regularly arranged fibrous connective tissue. Eventhe ligamentum nuchae, which is composed of elastic fibers, has the elastic fibersregularly arranged.

Page 137: Review Questions for Human Histology

D. is correct.

Mesenteries are examples of loose areolar connective tissue which can have asignificant amount of adipose tissue associated with them.

4.028 Which of the following best describes the primary composition of the capsuleof kidney?

A. dense regular connective tissue

B. dense irregular connective tissue

C. reticular connective tissue

D. loose areolar connective tissue

E. adipose connective tissue

F. mesenchymal connective tissue

G. mucoid connective tissue

4.029 Which of the following best describes the primary composition of the red pulpof spleen?

A. dense regular connective tissue

B. dense irregular connective tissue

C. reticular connective tissue

D. loose areolar connective tissue

E. adipose connective tissue

F. mesenchymal connective tissue

G. mucoid connective tissue

4.030 Which of the following best describes the primary composition of the core ofthe umbilical cord?

A. dense regular connective tissue

B. dense irregular connective tissue

Page 138: Review Questions for Human Histology

C. reticular connective tissue

D. loose areolar connective tissue

E. adipose connective tissue

F. mesenchymal connective tissue

G. mucoid connective tissue

4.031 Which of the following best describes the primary composition of laminapropria?

A. dense regular connective tissue

B. dense irregular connective tissue

C. reticular connective tissue

D. loose areolar connective tissue

E. adipose connective tissue

F. mesenchymal connective tissue

G. mucoid connective tissue

4.032 Which of the following best describes the primary composition of the packingoutside the developing brain?

A. dense regular connective tissue

B. dense irregular connective tissue

C. reticular connective tissue

D. loose areolar connective tissue

E. adipose connective tissue

F. mesenchymal connective tissue

G. mucoid connective tissue

B. is correct.

In general, the capsules of most organs and glands are composed of dense

Page 139: Review Questions for Human Histology

irregular fibrous connective tissue.

C. is correct.

The main connective tissue type of the parenchymal regions of the lymphoidorgans is reticular connective tissue.

G. is correct.

The core or connective tissue of the umbilical cord is one of the two types ofembryonic connective tissue or mucoid connective tissue (Wharton's jelly).Because of its watery, gelatinous nature it is also termed myxomatous connectivetissue. In the adult, undifferentiated mesodermally derived cells can form aneoplasm which demonstrates differentiation in the mucoid or myxomatousdirection (myxoma if benign, myxosarcoma if malignant).

D. is correct.

All lamina propriae, wherever they are found, are composed of loose areolarconnective tissue containing the smaller blood vessels such as arterioles,capillaries and venules, and lymph capillaries which are servicing the overlyingavascular epithelium.

F. is correct.

Mesenchyme is the embryonic connective tissue found in all regions of the embryo,especially in the head between the neural tube and the ectoderm.

4.033 Which of the following best describes the primary composition of anaponeurosis?

A. dense regular connective tissue

B. dense irregular connective tissue

C. reticular connective tissue

D. loose areolar connective tissue

E. adipose connective tissue

F. mesenchymal connective tissue

G. mucoid connective tissue

4.034 Which of the following best describes the primary composition of theepimysium?

Page 140: Review Questions for Human Histology

A. dense regular connective tissue

B. dense irregular connective tissue

C. reticular connective tissue

D. loose areolar connective tissue

E. adipose connective tissue

F. mesenchymal connective tissue

G. mucoid connective tissue

4.035 Which of the following best describes the primary composition of the core ofan intestinal villus?

A. dense regular connective tissue

B. dense irregular connective tissue

C. reticular connective tissue

D. loose areolar connective tissue

E. adipose connective tissue

F. mesenchymal connective tissue

G. mucoid connective tissue

4.036 Which of the following best describes the primary composition of thesubmucosa of the esophagus?

A. dense regular connective tissue

B. dense irregular connective tissue

C. reticular connective tissue

D. loose areolar connective tissue

E. adipose connective tissue

F. mesenchymal connective tissue

Page 141: Review Questions for Human Histology

G. mucoid connective tissue

4.037 Which of the following best describes the primary composition of theadventitia of the colon?

A. dense regular connective tissue

B. dense irregular connective tissue

C. reticular connective tissue

D. loose areolar connective tissue

E. adipose connective tissue

F. mesenchymal connective tissue

G. mucoid connective tissue

A. is correct.

Ligaments, tendons and aponeuroses are formed of regularly arranged fibrousconnective tissue.

B. is correct.

The epimysium, epitendinium and epineurium are examples of dense irregularlyarranged fibrous connective tissue.

D. is correct.

All of the connective tissue underlying and servicing an overlying epithelium is ofthe loose areolar type. Even in skin, the epithelium servicing the overlying stratifiedsquamous keratinized epithelium, i.e. the papillary layer of the dermis, is looseareolar connective tissue.

D. is correct.

Both the lamina propria and the submucosa of the GI tract are composed of looseareolar connective tissue.

D. is correct.

Adventitial regions, such as the tunica adventitia of vessels and the adventitia ofthe parts of the GI tract which are retroperitoneal, are good examples of looseareolar connective tissue.

Page 142: Review Questions for Human Histology

4.038 Which type of collagen is primarily found in bone?

A. type I collagen

B. type II collagen

C. type III collagen

D. type IV collagen

E. type V collagen

4.039 Which type of collagen is primarily found in cartilage?

A. type I collagen

B. type II collagen

C. type III collagen

D. type IV collagen

E. type V collagen

4.040 Which type of collagen is primarily found in skin?

A. type I collagen

B. type II collagen

C. type III collagen

D. type IV collagen

E. type V collagen

4.041 Which type of collagen is primarily found in tendons?

A. type I collagen

B. type II collagen

C. type III collagen

D. type IV collagen

E. type V collagen

Page 143: Review Questions for Human Histology

4.042 Which type of collagen is primarily found in basal laminae?

A. type I collagen

B. type II collagen

C. type III collagen

D. type IV collagen

E. type V collagen

4.043 Which type of collagen is primarily found in the stroma of the cornea?

A. type I collagen

B. type II collagen

C. type III collagen

D. type IV collagen

E. type V collagen

4.044 Which type of collagen is found in the walls of blood vessels?

A. type I collagen

B. type II collagen

C. type III collagen

D. type IV collagen

E. type V collagen

A. is correct.

The type of collagen found in bone is type I.

B. is correct.

Hyaline and elastic cartilage are primarily composed of type II collagen.

A. is correct.

In dermis the main type of collagen is type I.

Page 144: Review Questions for Human Histology

A. is correct.

Type I collagen is found in tendons, fascias, dermis and bone.

D. is correct. Type IV collagen is the type found in basal laminae.

A. is correct.

Type V is found in the cornea but secondary in concentration to type I

C. is correct.

Type III collagen is the type of collagen found in the walls of blood vessels.

4.045 Both ends of a precursor molecule are enzymatically clipped

A. ribosome

B. cisternae of RER

C. Golgi body

D. cell exterior

4.046 Where are glycine, hydroxylysine and hydroxyproline linked together to formcollagen?

A. ribosome

B. cisternae of RER

C. Golgi body

D. cell exterior

4.047 Where do pro-alpha chains wind together by threes to form procollagen?

A. ribosome

B. cisternae of RER

C. Golgi body

D. cell exterior

4.048 Where are highly insoluble collagen fibers formed?

Page 145: Review Questions for Human Histology

A. ribosome

B. cisternae of RER

C. Golgi body

D. cell exterior

4.049 Where is procollagen packaged into secretion vesicles?

A. ribosome

B. cisternae of RER

C. Golgi body

D. cell exterior

D. is correct.

Procollagen is trimmed extracellularly at the surface of the cell. The tropocollagenthus formed aggregates to form collagen.

A. is correct.

Collagen in somewhat unique in that it is composed of about 30% glycine and 30%hydroxyproline and proline.

B. is correct.

The individual chains are made on the ribosomes of the RER, but they coil aroundeach other in the cisternae of the RER before they head for the Golgi body.

D. is correct.

Tropocollagen molecules self-assemble into insoluble collagen fibers outside ofthe fibroblast.

C. is correct.

This is the general function of the Golgi apparatus, i.e. to package a protein in amembrane bound vesicle. Procollagen is packaged into secretory vesicles whichempty their contents onto the cell surface where a protease clips or trims theprocollagen to form tropocollagen.

Page 146: Review Questions for Human Histology

SECTION 5: MUSCLE TISSUE

5.001 In a skeletal muscle such as the deltoid which of the following would be thesmallest component?

A. perimysium

B. endomysium

C. epimysium

D. skeletal muscle fiber or myofiber

E. skeletal muscle myofibrils

5.002 In a cross section of a sarcomere in skeletal muscle as seen with theelectron microscope which of the following would show only thick filaments?

A. Z line

B. I band

C. A band

D. H band

5.003 Muscle contraction depends on the availability of calcium ions as well asATP. Where in the skeletal muscle cell is the calcium sequestered duringrelaxation and released from during contraction?

A. motor end plate

B. sarcolemma

C. transverse or T tubule

D. sarcoplasmic reticulum

E. mitochondria

5.004 In human skeletal muscle where does the T tubule intercept the sarcomere?

A. Z line

B. I band

Page 147: Review Questions for Human Histology

C. A band

D. A-I jct.

E. I-H jct.

5.005 During the contraction of a skeletal muscle fiber which of the followingdecreases in length?

A. A band

B. I band

C. Z band

D. M band

E. is correct.

The myofibrils are components of the myofiber. Individual myofibers or cells aresurrounded by a connective tissue covering, the endomysium. Groups or fasciclesof myofibers are surrounded by perimysium and the entire organ or muscle iscovered by the epimysium.

D. is correct.

The thick or myosin filaments are located in the H and A bands, but in the A bandsthe myosin interdigitates with the thin or actin filaments. The I band is composedonly of thin filaments. The H band is composed only of thick filaments.

D. is correct.

The sarcoplasmic reticulum is the cellular site for the storage and release ofcalcium. The wave of depolarization is initiated at the motor end plate and spreadsover the sarcolemma and into the cell via the transverse tubule system which formstriad structures with two adjacent terminal cisternae of the sarcoplasmic reticulum.In this way the wave of depolarization is transmitted to all internal regions of thecell simultaneously. When the sarcoplasmic reticulum releases its stored calciumin the presence of ATP contraction occurs.

D. is correct.

In cardiac muscle the T tubules surround the sarcomere at the Z line. In skeletalmuscle the T tubules surround the sarcomere at the A-I intercept.

B. is correct.

Page 148: Review Questions for Human Histology

As the thick and thin filaments slide over each other during contraction the I banddecreases in length because the thin actin filaments sort of slide along and therebyinto the thick filaments of the A band.

5.006 Which of the following is NOT a major component of the assembled thinfilament of skeletal muscle?

A. troponin

B. tropomyosin

C. actin

D. meromyosin

5.007 In which muscle in the body would you expect to find a motor unit comprisingone motor fiber per each muscle fiber instead of one motor fiber innervating about100 muscle fibers?

A. deltoid

B. quadriceps femoris

C. biceps

D. external abdominal oblique

E. superior oblique

5.008 All of the following describe white skeletal muscle fibers EXCEPT.

A. low level of myoglobin

B. low level of mitochondria

C. low level of cytochrome

D. capable of rapid contractions

E. capable of sustained contractions

5.009 In the heart the cardiac muscle cells are in ionic communication with eachother through which of the following:

A. zonula adherens

Page 149: Review Questions for Human Histology

B. macula adherens

C. nexus or gap junctions.

D. zonula occludens

E. macula occludens

5.010 Where is the best cellular source for natriuretic factor, a hormone which isantagonistic to aldosterone and ADH in its effects on the kidney?

A. ventricular cardiac muscle

B. atrial cardiac muscle

C. smooth muscle of the tunica media of the aorta

D. skeletal muscle of the intercostal space

E. myoneural junction

5.011 Which of the following can undergo both hypertrophy and hyperplasia undernormal physiological stress?

A. skeletal muscle

B. cardiac muscle

C. smooth muscle

D. is correct.

Heavy and light meromyosin are components of the myosin molecule. The otherchoices all are involved in polymerizing together to form the assembled thinfilament.

E. is correct.

The muscles which move the eye must be very finely controlled compared tocoarser movements of other muscles. The extraocular muscles have a relativelyhigh frequency of motor units comprised of one motor neuron innervating oneskeletal muscle cell.

E. is correct.

White muscles such as the breast muscles in a chicken or the extraocular musclesin the human are characterized by A-D. The red muscles are characterized by E.

Page 150: Review Questions for Human Histology

Most muscles in the body are combinations of white, red and intermediate types ofmyofibers.

C. is correct.

Within the intercalated disc on its lateral rather than transverse portions are gapjunctions. These junctions provide the means by which adjacent cardiac musclecells communicate with each other ionically.

B. is correct.

At each cytoplasmic pole of the centrally located nucleus in atrial cardiac muscle,membrane bound granules can be seen. These granules are the storage form ofatrial natriuretic factor. Thus the heart is now known to be an endocrine organ.

C. is correct.

All three types of muscle can undergo hypertrophy or an increase in individual cellsize, but only smooth muscle can also increase in number (hyperplasia) underphysiologic stress such as pregnancy.

5.012 Skeletal muscle can regenerate. When skeletal muscle regeneration occursthe source for new skeletal muscle cells is?

A. embryonic myotubes left over from fetal myogenesis

B. traumatized skeletal muscle nuclei

C. endomysial fibroblasts

D. satellite cells

E. neurilemmal cells

5.013 Which type of muscle has a single nucleus per cell?

A. cardiac muscle

B. skeletal muscle

C. smooth muscle

D. both cardiac and smooth muscle

E. both cardiac and skeletal muscle

F. all three types of muscle

Page 151: Review Questions for Human Histology

5.014 Which of the following can multiply in the adult?

A. cardiac muscle

B. skeletal muscle

C. smooth muscle

D. both cardiac and smooth muscle

E. both cardiac and skeletal muscle

F. all three types of muscle

5.015 Which of the following is/are fixed postmitotic?

A. cardiac muscle

B. skeletal muscle

C. smooth muscle

D. both cardiac and smooth muscle

E. both cardiac and skeletal muscle

F. all three types of muscle

5.016 Which of the following can undergo hypertrophy?

A. cardiac muscle

B. skeletal muscle

C. smooth muscle

D. both cardiac and smooth muscle

E. both cardiac and skeletal muscle

F. all three types of muscle

D. is correct.

Satellite cells are mesenchymal cells which lie in close contact with differentiatedskeletal muscle cells. In fact they are enclosed in the basal lamina of the skeletal

Page 152: Review Questions for Human Histology

muscle cell. Under the proper conditions, the undifferentiated satellite cells canundergo differentiation to myoblasts. The regeneration of skeletal muscle, whichcan be observed under experimental situations, is believed to be derived fromundifferentiated satellite cells found lying in the basal lamina of skeletal musclecells, not from the more differentiated skeletal muscle nuclei

themselves.

D. is correct.

The only type of muscle cell which is multinucleated is the skeletal muscle cell.

C. is correct.

Smooth muscle cells are the only ones which can increase in number in the adult.

E. is correct.

In general, neither cardiac nor skeletal muscle cell nuclei can undergo celldivision. However, smooth muscle cells can undergo cell division or hyperplasiaespecially during pregnancy in the myometrium.

F. is correct.

All three types of muscle cells can increase in size in response to physiologic workload. This is easy to appreciate when one thinks of the weight lifter or body builderfor the skeletal muscle cell, the pregnant myometrium for the smooth muscle celland the enlarged heart of the marathon runner.

5.017 Which of the following contains actin and myosin filaments?

A. cardiac muscle

B. skeletal muscle

C. smooth muscle

D. both cardiac and smooth muscle

E. both cardiac and skeletal muscle

F. all three types of muscle

5.018 Which of the following contain centrally located nuclei?

A. cardiac muscle

Page 153: Review Questions for Human Histology

B. skeletal muscle

C. smooth muscle

D. both cardiac and smooth muscle

E. both cardiac and skeletal muscle

F. all three types of muscle

5.019 Which of the following are multinucleated?

A. cardiac muscle

B. skeletal muscle

C. smooth muscle

D. both cardiac and smooth muscle

E. both cardiac and skeletal muscle

F. all three types of muscle

5.020 For which of the following does contraction involve Ca complexing withcalmodulin?

A. cardiac muscle

B. skeletal muscle

C. smooth muscle

D. both cardiac and smooth muscle

E. both cardiac and skeletal muscle

F. all three types of muscle

5.021 Which of the following are derived from embryonic mesoderm/mesenchyme?

A. cardiac muscle

B. skeletal muscle

C. smooth muscle

Page 154: Review Questions for Human Histology

D. both cardiac and smooth muscle

E. both cardiac and skeletal muscle

F. all three types of muscle

5.022 The length of the I band decreases in the event of

A. contraction of skeletal muscle

B. relaxation of skeletal muscle

C. both relaxation and contraction of skeletal muscle

F. is correct.

All three types contain both actin and myosin filaments.

D. is correct.

The nuclei in skeletal muscle are peripherally located in the cell. The nucleus ineach smooth or cardiac muscle cell is located in the center of the cell.

B. is correct.

Of the three types of muscle only the skeletal muscle cell is multinucleated.

C. is correct.

The thin filaments of smooth muscle contain actin and tropomosin but not troponin.In place of troponin, smooth muscle cells use a different calcium binding protein,calmodulin.

F. is correct.

In general all three types of muscle are derived from mesoderm or mesenchymewhich differentiates into myoblasts.

A. is correct.

During contraction the actin filaments slide into the stationary myosin filaments.This results in a decrease in the total length of the I band.

5.023 Calcium is released into the sarcoplasm in the event of

A. contraction of skeletal muscle

Page 155: Review Questions for Human Histology

B. relaxation of skeletal muscle

C. both relaxation and contraction of skeletal muscle

5.024 An action potential travels along T tubules in the event of

A. contraction of skeletal muscle

B. relaxation of skeletal muscle

C. both relaxation and contraction of skeletal muscle

5.025 The length of the I band increases in the event of

A. contraction of skeletal muscle

B. relaxation of skeletal muscle

C. both relaxation and contraction of skeletal muscle

5.026 CaATPase actively transports calcium to the lumen of the sarcoplasmicreticulum in the event of

A. contraction of skeletal muscle

B. relaxation of skeletal muscle

C. both relaxation and contraction of skeletal muscle

5.027 The length of the A band stays the same in the event of

A. contraction of skeletal muscle

B. relaxation of skeletal muscle

C. both relaxation and contraction of skeletal muscle

5.028 The length of the sarcomere increases in the event of

A. contraction of skeletal muscle

B. relaxation of skeletal muscle

C. both relaxation and contraction of skeletal muscle

5.029 What type of muscle is found in the pyloric sphincter?

Page 156: Review Questions for Human Histology

A. cardiac muscle

B. skeletal muscle

C. smooth muscle

D. both skeletal and smooth muscle

E. none of the above

5.030 What type of muscle is found in the lower third of the esophagus?

A. cardiac muscle

B. skeletal muscle

C. smooth muscle

D. both skeletal and smooth muscle

E. none of the above

A. is correct.

The release of calcium from its storage in the lumen of the sarcoplasmic reticuluminto the sarcoplasm where its concentration will increase around the myofibrils isassociated with contraction.

A. is correct.

As an action potential moves into the muscle cell along the T tubules it will causerelease of calcium from the sarcoplasmic reticulum and this will cause contraction.

B. is correct.

As the actin filaments slide or move out of the myosin filaments during relaxationthe length of the I band will be increased.

B. is correct.

During relaxation calcium ions are stored in the lumen of the sarcoplasmicreticulum.

C. is correct.

The myosin filaments in the A band stay put during both contraction and relaxation.The actin filaments slide into the myosin filaments during contraction and they slide

Page 157: Review Questions for Human Histology

out during relaxation.

B. is correct.

During relaxation of skeletal muscle the length of the sarcomere increases as theactin and myosin filaments slide over each other in a fashion which is opposite tothat found during contraction.

C. is correct.

This is composed of smooth muscle in the muscularis externa of the stomach.

C. is correct.

The voluntary nature of swallowing, i.e. the skeletal muscle of the pharyngealconstrictors and its continuation into the esophagus is lost by the time the loweresophagus is reached. From here to the external anal sphincter the GI tube hasonly smooth muscle in its muscularis mucosae and externae layers.

5.031 What type of muscle is found in the core of the false vocal fold?

A. cardiac muscle

B. skeletal muscle

C. smooth muscle

D. both skeletal and smooth muscle

E. none of the above

5.032 What type of muscle is found in the sphincter of the iris?

A. cardiac muscle

B. skeletal muscle

C. smooth muscle

D. both skeletal and smooth muscle

E. none of the above

5.033 What type of muscle is found in the taenia coli?

A. cardiac muscle

Page 158: Review Questions for Human Histology

B. skeletal muscle

C. smooth muscle

D. both skeletal and smooth muscle

E. none of the above

5.034 What type of muscle is found in the auricular appendage of the atrium?

A. cardiac muscle

B. skeletal muscle

C. smooth muscle

D. both skeletal and smooth muscle

E. none of the above

5.035 What type of muscle is found in the tunica media of the thoracic duct?

A. cardiac muscle

B. skeletal muscle

C. smooth muscle

D. both skeletal and smooth muscle

E. none of the above

5.036 What type of muscle is found in the tunica adventitia of the inferior venacava?

A. cardiac muscle

B. skeletal muscle

C. smooth muscle

D. both skeletal and smooth muscle

E. none of the above

E. is correct.

Page 159: Review Questions for Human Histology

The core of the false vocal fold contains loose areolar connective tissue andglands, but no skeletal muscle.

C. is correct.

The sphincter of the iris, like the dilator of the iris, is composed of smooth musclewhich is innervated by the autonomic nervous system.

C. is correct.

The taenia coli of the colon represent enlarged areas of longitudinally arrangedsmooth muscle belonging to the muscularis externa layer of the wall of the colon. Ifone is interested in locating myenteric (Auerbach's) plexuses in the colon, a goodplace to look is the interface between the outer longitudinal and inner circularlayers of smooth muscle.

A. is correct.

The auricular appendages of the cardiac atria would be composed of cardiacmuscle.

C. is correct.

All tunica media are composed of circularly arranged smooth muscle.

C. is correct.

In the usual sense, a tunica adventitia is a connective tissue area, but in the inferiorvena cava the large amount of longitudinally arranged smooth muscle which isimportant in moving blood superiorly, is officially placed in the tunica adventitia.

5.037 What type of muscle is found in the middle third of the esophagus?

A. cardiac muscle

B. skeletal muscle

C. smooth muscle

D. both skeletal and smooth muscle

E. none of the above

5.038 What type of muscle spans the open ends of cartilage rings in the trachea?

A. cardiac muscle

Page 160: Review Questions for Human Histology

B. skeletal muscle

C. smooth muscle

D. both skeletal and smooth muscle

E. none of the above

5.039 What type of muscle is found in the trabeculae of the lymph node?

A. cardiac muscle

B. skeletal muscle

C. smooth muscle

D. both skeletal and smooth muscle

E. none of the above

5.040 What type of muscle is found in the core of the true vocal fold?

A. cardiac muscle

B. skeletal muscle

C. smooth muscle

D. both skeletal and smooth muscle

E. none of the above

5.041 What type of muscle is found in the mediastinum of the testis?

A. cardiac muscle

B. skeletal muscle

C. smooth muscle

D. both skeletal and smooth muscle

E. none of the above

5.042 What type of muscle is found in the myometrium of the uterus?

A. cardiac muscle

Page 161: Review Questions for Human Histology

B. skeletal muscle

C. smooth muscle

D. both skeletal and smooth muscle

E. none of the above

D. is correct.

The middle third of the esophagus is the region in which the skeletal muscle fromthe upper part is giving way to the smooth muscle of the lower part.

C. is correct.

The trachealis muscle is the mass of smooth muscle which bridges the gapbetween the open ends of the C-shaped pieces of hyaline cartilage.

E. is correct.

Connective tissue trabeculae, in general, are extensions of the connective tissuecapsule of an organ into the deeper regions of the organ. The trabeculae of thespleen and the lymph node, for example, are examples of connective tissue areaswhich leave the capsule and enter the organ.

B. is correct.

The deep core of the true vocal fold is composed of skeletal muscle, the vocalismuscle. Between the vocalis muscle and the epithelial covering of this fold is amass of elastic connective tissue sometime called the vocal ligament.

E. is correct.

The mediastinum of the testis is a major connective tissue area which contains therete testis and from which connective tissue septa radiate outward to form partitionsor little "rooms" which house the seminiferous tubules.

C. is correct.

The myometrium of the uterus is a mass of smooth muscle which can undergo bothhypertrophy and hyperplasia during pregnancy. Benign tumors of the smoothmuscle cells mixed with the ever present fibroblasts in the myometrium are uterine"fibroids" (actually fibromyomas).

5.043 What type of muscle is found in the pars membranaceum of theinterventricular septum?

Page 162: Review Questions for Human Histology

A. cardiac muscle

B. skeletal muscle

C. smooth muscle

D. both skeletal and smooth muscle

E. none of the above

5.044 What type of muscle is found in the tunica adventitia of the descendingaorta?

A. cardiac muscle

B. skeletal muscle

C. smooth muscle

D. both skeletal and smooth muscle

E. none of the above

5.045 What type of muscle is found in the tunica albuginea of the corporacavernosa of the penis?

A. cardiac muscle

B. skeletal muscle

C. smooth muscle

D. both skeletal and smooth muscle

E. none of the above

5.046 What type of muscle is found in the upper third of the esophagus?

A. cardiac muscle

B. skeletal muscle

C. smooth muscle

D. both skeletal and smooth muscle

E. none of the above

Page 163: Review Questions for Human Histology

5.047 What type of muscle is found in the stapes of the middle ear?

A. cardiac muscle

B. skeletal muscle

C. smooth muscle

D. both skeletal and smooth muscle

E. none of the above

5.048 What type of muscle is responsible for "goosebumps" on cold skin?

A. cardiac muscle

B. skeletal muscle

C. smooth muscle

D. both skeletal and smooth muscle

E. none of the above

E. is correct.

This region of the interventricular septum of the heart is one of the components ofthe cardiac skeleton and it is composed of dense regular fibrous connective tissue.

E. is correct.

Tunica adventitiae are composed of loose areolar connective tissue mingled withsome adipose tissue. No muscle is present. There is one exception to this rule andthat is the inferior vena cava which has a large amount of longitudinally arrangedsmooth muscle in its tunica adventitia.

E. is correct.

The tunica albuginea of the body, be it of ovary, testis, or penis, are all composedof dense irregularly arranged fibrous connective tissue and not of any kind ofmuscle.

B. is correct.

The skeletal muscle of the pharyngeal constrictors continues inferiorly into themuscularis externa of the upper third of the esophagus and then begins to blend

Page 164: Review Questions for Human Histology

with smooth muscle fibers in the middle third of the esophagus. In the lower third ofthe esophagus the skeletal muscle fibers have been replaced by smooth musclefibers.

E. is correct.

The stapes of the inner ear is made of bone. The stapedius muscle of the inner earis a tiny skeletal muscle innervated by the 7th cranial nerve.

C. is correct.

The arrector pili muscles found in the reticular layer of the dermis are small smoothmuscles which, when contracted, pull the hair follicle up straight and in doing socause "bumps" to form on the surface of the skin.

5.049 What type of muscle is found in the internal anal sphincter?

A. cardiac muscle

B. skeletal muscle

C. smooth muscle

D. both skeletal and smooth muscle

E. none of the above

5.050 What type of muscle is found in the tunica media of the umbilical vein in theumbilical cord?

A. cardiac muscle

B. skeletal muscle

C. smooth muscle

D. both skeletal and smooth muscle

E. none of the above

C. is correct.

The internal anal sphincter, as opposed to the external anal sphincter, isinvoluntary and is composed of smooth muscle.

C. is correct.

Page 165: Review Questions for Human Histology

The tunica media of all vessels is a good place to fmd circularly arranged smoothmuscle cells.

Page 166: Review Questions for Human Histology

SECTION 6: NERVOUS TISSUE

6.001 Which of the following cells makes the myelin sheath found within the CNS?

A. microglia

B. protoplasmic astrocyte

C. satellite cell

D. Schwann cell

E. oligodendrocyte

6.002 Nissl substance or bodies actually is the light microscopic version of what?

A. Golgi apparatus

B. synaptic vesicles

C. RER and polysome areas

D. groups of neurofilaments

E. the axon hillock

6.003 The region of the perikaryon in which microtubules are concentrated andarranged into bundles or fascicles is the?

A. axolemma

B. axon hillock

C. Nissi body

D. node of Ranvier

E. dendrite

6.004 Which of the following would be the best place to find the highestconcentration of a neurotransmitter?

A. synaptic cleft

B. synaptic vesicle

Page 167: Review Questions for Human Histology

C. presynaptic membrane

D. postsynaptic membrane

E. neuromuscular junction

6.005 Which of the following cells is known to be phagocytic?

A. oligodendrocyte

B. microglia

C. fibrous astrocyte

D. protoplasmic astrocyte

E. ependyma

E. is correct.

The Schwann cell makes peripheral myelin and the oligodendrocyte makes centralmyelin.

C. is correct.

Nissl bodies are the basophilic granular areas seen with the light microscope inthe perikaryon of the neuron. Nissl bodies represent mixtures of RER andpolysomes at the EM level.

B. is correct.

The axon hillock is devoid of Nissl substance and is a place where themicrotubules are found in bundles.

B. is correct.

Although neurotransmitters can be found in all of the areas listed, it is in thesynaptic vesicle in which the neurotransmitter is stored before its release. After itsexocytosis from the presynaptic membrane its half life is very short.

B. is correct.

The microglial cells are phagocytes and probably antigen-presenting cells in theCNS.

6.006 In an electron micrograph of myelinated peripheral nerve the major dense

Page 168: Review Questions for Human Histology

lines seen in the myelin sheath actually represent ?

A. lines of fusion of the P surfaces of plasmalemma

B. lines of fusion of the E surfaces of plasmalemma

C. lines of fusion of the P faces of the plasmalemma

D. lines of fusion of the E faces of the plasmalemma

E. lines of fusion of the glycocalyxes of two apposing plasmalemmas

6.007 An axon residing in a simple cleft in the cytoplasm of a Schwann cell isprobably:

A. a myelinated peripheral nervous system axon

B. a myelinated central nervous system axon

C. an unmyelinated PNS axon

D. an unmyelinated CNS axon

6.008 With respect to the autonomic nervous system which of the following doesNOT have postganglionic nerve fibers?

A. Meissner's (submucosal) plexus neuron

B. Auerbach's (myenteric) plexus neuron

C. parenchyma of adrenal medulla

D. neuron in the celiac ganglion

E. neuron in the superior cervical ganglion

6.009 Where in the nervous system, in relation to the propagation of an actionpotential, are the sodium and potassium channels concentrated?

A. endoneurium

B. satellite cell

C. Schwann cell interperiod line

D. node of Ranvier

Page 169: Review Questions for Human Histology

E. perikaryon of a motor neuron

6.010 Which of the following is NOT transported by the slow component of theaxonal transport system?

A. synaptic vesicles

B. tubulin

C. neurofilament proteins

D. actin

E. metabolic enzymes

6.011 Chromatolysis following injury to a nerve includes all of the followingEXCEPT:

A. decreased cytoplasmic basophilia in perikaryon

B. disappearance of Nissl substance

C. increase in volume of perikaryon

D. peripheral movement of nucleus in perikaryon

E. complete axonal degeneration

A. is correct.

As the Schwann cell wraps around the axon it is myelinating, the cytoplasm is"squeezed out" and the major dense line in myelin results from a fusion of the Psurfaces of the same Schwann cell.

C. is correct.

Unmyelinated axons in the CNS often run free in the neuropile. Unmyelinatedaxons in the PNS sit in clefts in the cytoplasm of Schwann cells.

C. is correct.

The cells of the adrenal medulla are neural crest derivatives which instead ofdifferentiating into neurons, differentiate into endocrine glandular secretory cellsand therefore they receive a direct preganglionic input and are considered to beanalogous to postganglionic units.

D. is correct.

Page 170: Review Questions for Human Histology

In saltatory conduction the action potential jumps from node to node because it isat the nodes of Ranvier that the axon is exposed to the extracellular fluid. This isalso where the sodium and potassium voltage-gated channels are concentrated inthe cell membrane of the neuron.

A. is correct.

Synaptic vesicles are transported by the fast axonal transport system (hundreds ofmm/day), along with mitochondria, down the axon toward its termination. All of theother choices are transported by the slow transport system (few mm/day).

E. is correct.

Complete axonal degeneration occurs distal to the site of the injury. The myelinsheath also degenerates, but the Schwann cells proliferate in the endoneural tubeforming "guides" for the regenerating axons. All of the other choices arecharacteristics of chromatolysis in the nerve cell body.

6.012 In which of the following are the satellite cells complete in their enveloping ofperikarya?

A. dorsal root ganglion

B. myenteric ganglion

C. submucosal ganglion

D. inferior mesenteric ganglion

E. inferior cervical ganglion

6.013 Purkinje cells are found:

A. in the cerebrum

B. in the cerebellum

C. in the anterior horn of the spinal gray matter

D. in the white matter of the spinal cord

E. in the choroid plexus

6.014 Which of the following is the main component of the blood-brain barrier?

A. gap jct. or nexus

Page 171: Review Questions for Human Histology

B. macula adherens

C. zonula occludens

D. fascia adherens

E. interdigitation

6.015 The Na/K-ATPase pump causes which of the following to happen?

A. Na moved to outside/K moved to inside the cell

B. voltage difference between inside and outside the cell is maintained by thepump

C. voltage difference is measured in millivolts

D. A & B are correct

E. A & B & C are correct

6.016 Nerve and muscle cells, unlike other body cells, are excitable, i.e. theirmembrane voltage can change instantly in response to the correct stimulus. Whenthis happens, all of the following correctly apply at the cellular level EXCEPT?

A. voltage gated channels open

B. Na flows inward

C. K flows outward

D. the inside of the cell becomes momentarily positive

E. the outside of the cell becomes momentarily positive

A. is correct.

Autonomic ganglia are characterized by an incomplete envelope of satellite cells.The dorsal root or spinal ganglia have complete envelopes of satellite cellssurrounding the perikarya.

B. is correct.

Purkinje cells are located in the cerebellum between its outer molecular layer andthe inner granular layer.

Page 172: Review Questions for Human Histology

C. is correct.

The capillary endothelial cells in the CNS have no fenestrations and very littlepinocytotic activity. Unlike endothelial cells outside of the CNS, these cells haveoccludens type junctions between them.

E. is correct.

Although this difference occurs in all animal cells, in nerve and muscle cells thiselectrical difference is called the resting potential.

D. is correct.

In this action potential situation the outside of the cell becomes negative and theinside becomes positive. The change in inside electrical potential occurs in lessthan one millisecond. The action potential passes along (propagated) the cellsurface.

6.017 What causes the action potential in a myelinated nerve to jump from onenode of Ranvier to the next node of Ranvier as it is propagated?

A. myelin acts as an electrical insulator

B. Na channels are restricted almost entirely to the nodal plasmalemma.

C. Na channels are evenly concentrated throughout the entire neurilemma

D. A & B are correct

E. A & B & C are correct

6.018 When an action potential reaches the end of an axon terminal, all of thefollowing happen EXCEPT:

A. it jumps across the synaptic cleft and depolarizes the postsynaptic membrane ofthe next neuron

B. it fails to jump across the synaptic cleft

C. synaptic vesicles exocytose their contents into the synaptic cleft

D. neurotransmitter molecules cross the synaptic cleft

E. neurotransmitter molecules are bound by receptors or ligand-gated channels ina plasmalemma

6.019 All of the following are known to be neurotransmitter substances contained

Page 173: Review Questions for Human Histology

and released by synaptic vesicles EXCEPT:

A. acetylcholine

B. serotonin

C. dopamine

D. norepinephrine

E. Ca ion flux

6.020 Arrange the following layers in proper sequence as a pin passes from piamater to cerebral spinal fluid through the cerebral cortex: 1=pia mater, 2=molecularlayer; 3=outer granular layer; 4=pyramidal cell layer, 5=inner granular layer;6=pyramidal cells of Betz.

A. 1-2-3-4-5-6

B. 1-3-2-5-4-6

C. 1-4-2-3-5-6

D. 1-5-4-2-3-6

E. 1-5-4-3-2-6

6.021 Why do myelinated nerve fibers transmit an impulse much faster than non-myelinated fibers?

A. almost all of the membrane channels are confined at the nodes of Ranvier

B. the myelin coat significantly reduces current leakage from the axon

C. the myelin coat decreases the capacitance of the axonal membrane

D. A & B are correct

E. A & B & C are correct

D. is correct.

This type of jumpy propagation of the action potential in a myelinated nerve fiber isreferred to as saltatory conduction. It is much faster (100 meters/second) than theburning fuse method found in unmyelinated nerve fibers (25 meters/second).

A. is correct.

Page 174: Review Questions for Human Histology

Action potentials stimulate the next neuron in the sequence, not by electricallyjumping across the synaptic cleft, but by causing the release of neurotransmitter.

E. is correct.

Ca ion influx into the cytoplasm of the axon terminal immediately precedes thefusion of the synaptic vesicles with the presynaptic membrane which is howneurotransmitter substances are released into the synaptic cleft.

A. is correct.

D. is correct.

The myelin coat actually increases the capacitance of the axonal membranethereby preventing the buildup of opposite charges on either side of the axolemmaand sort of confining this buildup to the nodes of Ranvier.

6.022 As a pin enters the substance of the brain it traverses the pia mater, then abasement membrane and then which of the following?

A. oligodendrocyte cytoplasm

B. central myelin

C. interneurons

D. ependymal cells

E. astrocyte cytoplasm

6.023 Which of the following forms myelin?

A. oligodendrocyte

B. fibrous astrocyte

C. protoplasmic astrocyte

D. microglia

E. ependymal cell

6.024 Which of the following lines the lumen of the neural tube?

A. oligodendrocyte

Page 175: Review Questions for Human Histology

B. fibrous astrocyte

C. protoplasmic astrocyte

D. microglia

E. ependymal cell

6.025 Which of the following covers the capillaries of the choroid plexus?

A. oligodendrocyte

B. fibrous astrocyte

C. protoplasmic astrocyte

D. microglia

E. ependymal cell

6.026 Which of the following can be phagocytic under certain situations such astrauma or inflammation?

A. oligodendrocyte

B. fibrous astrocyte

C. protoplasmic astrocyte

D. microglia

E. ependymal cell

6.027 Which of the following have end-feet that ensheath capillaries in graymatter?

A. oligodendrocyte

B. fibrous astrocyte

C. protoplasmic astrocyte

D. microglia

E. ependymal cell

E. is correct.

Page 176: Review Questions for Human Histology

Immediately under the basement membrane of the pia mater are found the subpialprocesses of astrocytes, which collectively form the glia limitans which invests theentire CNS.

A. is correct.

The oligodendrocyte forms myelin in the central nervous system.

E. is correct.

Ependymal cells line the ventricles of the brain and the central canal of the spinalcord.

E. is correct.

The ciliated cuboidal to columnar cells comprising the ependyma also cover thecapillary tufts of the choroid plexuses where they probably play a role in theformation of cerebral spinal fluid.

D. is correct.

Microglia are can become phagocytic under certain conditions such as injury to apart of the CNS.

C. is correct.

This is a description of what the protoplasmic astrocyte does in gray matter. Sincealmost all of the external surface of the capillaries in the brain can be covered bythe end-feet of the astrocytes, it is thought that the astrocytes play an important rolein regulating the composition of the tissue fluid compartment of the brain.

6.028 What structure are you touching when you roll the radial nerve of grossanatomy between your index finger and thumb?

A. epineurium

B. perineurium

C. endoneurium

D. all of the above

E. none of the above

6.029 What structure is found immediately peripheral to the plasmalemma of aSchwann cell?

Page 177: Review Questions for Human Histology

A. epineurium

B. perineurium

C. endoneurium

D. all of the above

E. none of the above

6.030 What structure is continuous with the same investment found in a tendon?

A. epineurium

B. perineurium

C. endoneurium

D. all of the above

E. none of the above

6.031 What structure contains collagen fibers and fibroblasts?

A. epineurium

B. perineurium

C. endoneurium

D. all of the above

E. none of the above

6.032 What structure divides a peripheral nerve up into fascicles?

A. epineurium

B. perineurium

C. endoneurium

D. all of the above

E. none of the above

6.033 What structure would contain almost exclusively capillaries?

Page 178: Review Questions for Human Histology

A. epineurium

B. perineurium

C. endoneurium

D. all of the above

E. none of the above

6.034 Schwann cells are associated with:

A. central nervous system

B. peripheral nervous system

A. is correct.

The outermost connective tissue covering of a peripheral nerve is the epineurium.

C. is correct.

The connective tissue supporting individual nerve fibers is the endoneurium.

E. is correct.

Peripheral nerves do not continue into tendons. However, the endo-, periand epi-mysium layers of a skeletal muscle such as the biceps do run into the same layersof a tendon such as the endo-, periand epitendinium.

D. is correct.

All of the connective tissue coverings found in a peripheral nerve containfibroblasts and collagen fibers.

B. is correct.

Perineurium forms the connective tissue sheaths for groups of nerve cellprocesses. Such a grouping is a fascicle. All of the connective tissue coveringswithin a peripheral nerve between the endoneurium and the epineurium aredivisions and examples of

perineurium.

C. is correct.

Page 179: Review Questions for Human Histology

Since the endoneurium is the connective tissue covering for each individual nervefiber in a peripheral nerve, it would be expected to contain nothing but capillaries,whereas the larger and thicker connective tissue coverings (peri-and epi-neurium)would be expected to contain vessels larger than capillaries, in addition to somecapillaries.

B. is correct.

Schwann cells are the myelin forming cells associated with the peripheral nervoussystem.

6.035 Satellite cells are associated with:

A. central nervous system

B. peripheral nervous system

6.036 Oligodendrocytes are associated with:

A. central nervous system

B. peripheral nervous system

6.037 Endoneurium is associated with:

A. central nervous system

B. peripheral nervous system

6.038 Ependymal cells are associated with:

A. central nervous system

B. peripheral nervous system

6.039 Motor end plates are associated with:

A. central nervous system

B. peripheral nervous system

6.040 The choroid plexus is associated with:

A. central nervous system

B. peripheral nervous system

Page 180: Review Questions for Human Histology

B. is correct.

Satellite cells are special cells which completely (dorsal root ganglia) orincompletely (autonomic ganglia) surround nerve cell bodies in ganglia of theperipheral nervous system.

A. is correct.

These are the cells which myelinate nerve fibers in the central nervous system.They are equivalent to the Schwann cell of the peripheral nervous system, exceptthat central myelin is chemically different from peripheral myelin.

B. is correct.

Endoneurium is the delicate connective sheath surrounding each nerve fiber in aperipheral nerve. The endoneurium is immediately outside or peripheral to theSchwann cell which is immediately peripheral to the axolemma.

A. is correct.

They are cuboidal to columnar ciliated cells which line the ventricles of the brainand the central canal of the spinal cord. They also cover the choroid plexuses.

B. is correct.

Motor end plates are specialized endings of motor neurons on skeletal musclecells and therefore can only be found in the peripheral nervous system. They areexcitatory synapses which release acetylcholine which induces depolarization inthe sarcolemma.

A. is correct.

The choroid plexus is a tuft of fenestrated capillaries covered by a cuboidalepithelium. The choroid plexuses are found in the ventricles of the brain wherethey are instrumental in producing cerebral spinal fluid which is a dialysate ofblood with its ionic content modified by the cuboidal epithelium covering of theplexus.

Page 181: Review Questions for Human Histology

SECTION 7: INTEGUMENT

7.001 Where in the epidermis are keratohyaline granules found?

A. stratum corneum

B. stratum lucidum

C. stratum granulosum

D. stratum spinosum

E. stratum basale

7.002 When a sebaceous gland secretes its product what type of secretorymechanism is utilized?

A. apocrine

B. merocrine

C. holocrine

D. endocrine

E. autocrine

7.003 Where in the integument would you expect to find Meissner's corpuscles?

A. stratum spinosum

B. papillary layer of dermis

C. reticular layer of dermis

D. hypodermis

E. superficial fascia

7.004 Arrange the following structures in the correct sequence from birth to finaldesquamation of an epidermal cell: 1=stratum spinosum, 2=stratum granulosum,3=stratum basale, 4=stratum lucidum, 5=stratum comeum.

A. 1-2-34-5

Page 182: Review Questions for Human Histology

B. 3-1-2-4-5

C. 2-1-4-3-5

D. 4-3-2-1-5

E. 3-2-1-4-5

7.005 When the police fingerprint a suspect the whorled pattern comprising thefingerprint, the dermatoglyphic pattern, actually represents?

A. primary epidermal ridges

B. primary dermal ridges

C. secondary dermal ridges

D. interpapillary pegs

E. dermal papillae

C. is correct.

Keratohyaline granules are characteristic of the stratum granulosum.

C. is correct.

The secretory product of the sebaceous gland is the entire cell itself. When theentire cell is the secretory product the type of secretion is termed holocrine.

B. is correct.

Meissner's corpuscles are found in the dermal papillae as they project into theoverlying epidermis as extensions from the papillary layer of the dermis.

B. is correct.

Epidermal cells are born in the stratum basale and move up through the s.spinosum, s. granulosum, s. lucidum to s. corneum.

A. is correct.

The friction ridges on the surface of the fingertips are formed from primaryepidermal ridges which overlie primary dermal ridges. The interpapillary peg is adowngrowth of the deep side of a primary ridge. The interpapillary peg divides theprimary dermal ridges into two secondary dermal ridges, each of which can haveseveral smaller dermal papillae which project toward the epidermis.

Page 183: Review Questions for Human Histology

7.006 The following description applies to which cell: epidermal cells which arestellate, contain tyrosinase and have their specific product phagocytized byadjacent cells.

A. keratinocyte

B. Langerhans cell

C. Merkel cell

D. melanocyte

E. fibroblast

7.007 Which of the following cells of the integument is derived from a bone marrowprecursor cell and can trap and present antigen to immunologically competentcells?

A. keratinocyte

B. Langerhans cell

C. Merkel cell

D. melanocyte

E. sebaceous cell

7.008 Which of the following cells of the integument is most closely associated withbare nerve endings?

A. keratinocyte

B. Langerhans cell

C. Merkel cell

D. melanocyte

E. cuboidal cell in duct of sweat gland

7.009 Arrange the following structures in proper sequence for the structure of a hairfollicle from centermost in the hair itself outward: 1=cortex, 2=cuticle, 3=medulla, 4=internal root sheath, 5=external root sheath.

A. 1-2-3-0-5

Page 184: Review Questions for Human Histology

B. 4-2-1-3-5

C. 2-3-1-4-5

D. 3-2-1-4-5

E. 3-1-213-5

7.010 Which of the following is the germinal region of a finger nail?

A. nail plate

B. nail matrix

C. lunula

D. nail bed

E. eponychium

D. is correct.

The melanocyte is derived from neural crest. It can synthesize melanin bysynthesizing tyrosinase which converts tyrosine into an melanin precursor andeventually melanin granules. The melanin granules are dispersed from themelanocyte cell processes and are "injected" (cytocrine secretion) into adjacentkeratinocytes and phagocytized by macrophages in the dermis (chromatophores).

B. is correct.

The Langerhans cell is a stellate cell found mostly in the stratum spinosum. It is animportant component of the defense system.

C. is correct.

Free nerve endings as opposed to encapsulated nerve endings are "free" of theirSchwann cell investment. In the skin of the palms and soles some free nerveendings have a disk-like ending in close association with a cell which has somedense-cored granules in its cytoplasm. This arrangement serves as amechanoreceptor as opposed to the pain and temperature receptor function ofother bare nerve endings.

E. is correct.

B. is correct.

Page 185: Review Questions for Human Histology

The nail matrix is the germinating center or area for the nail plate or nail itself. Thenail matrix is associated with the deep or proximal part of the nail plate. The nailbed is the epidermis underlying the nail plate, but not contributing cells to it. Theeponychium or cuticle is formed from the stratum comeum of the epidermis as itfolds to cover the nail root.

7.011 Which of the following apply to the condition of cold fingers and toes?

A. arteriovenous anastomoses in the rete cutaneum or cutaneous plexus are open

B. blood flow in the rete subpapillare or subpapillary vascular plexus is normal

C. blood flow in the subcutaneous vessels is diminished

D. A & B are correct

E. A & B & C are correct

7.012 The secretory product of this gland is mainly composed of water, salt,ammonia and urea; identify the gland.

A. sebaceous gland

B. eccrine sweat gland

C. apocrine sweat gland

D. ceruminous gland

E. gland of Zeiss

7.013 Which of the following cells would be detected by the dopa reaction?

A. Langerhans cells

B. keratinocyte in stratum granulosum

C. keratinocyte in stratum spinosum

D. chromatophore in dermis

E. melanocyte

7.014 All of the following are functions of the integument EXCEPT.

A. protection

Page 186: Review Questions for Human Histology

B. temperature regulation

C. synthesis of vitamin C

D. synthesis of vitamin D

E. excretion

7.015 A split-thickness skin graft is removed from a donor site (unburned) andgrafted to a burned site. The donor site will eventually be covered with a "new"epidermis. From what source(s) do the cells come from to form this "new"epidermis on the donor site?

A. external root sheath of hair follicles

B. ducts of sweat glands

C. sebaceous glands

D. A & B are correct

E. A & B & C are correct

A. is correct.

The AV anastomoses in the cutaneous plexus open when the environment is cold.This prevents much blood from perfusing the subpapillary plexus and losing itsheat out there. Blood flow in the subcutaneous vessels would be normal. When theenvironmental conditions are too hot, the AV anastomoses in the cutaneous plexusclose and by doing so, force blood into the small vessels of the subpapillary plexuswhere it will be close to the surface of the skin where it can benefit from the coolingeffect of sweat evaporation.

B. is correct.

All of the other glands listed secrete a more viscous product. The glands of Zeisare modified sebaceous glands associated with the hair follicles of the eyelids.

E. is correct.

When dope (dihydrophenylalanine) is exposed to a suitable preparation ofepidermis-dermis it is converted into melanin by the cells in the epidermis whichcontain tyrosinase, the melanocytes.

C. is correct.

Vitamin C is a dietary requirement.

Page 187: Review Questions for Human Histology

D. is correct.

Sebaceous glands secrete by the holocrine type in which an entire dead cell is thesecretory product. These cells do not participate in the formation of new epidermis.Cells from choices A & B do proliferate and migrate over the wound site to form anew epidermis at the donor site.

7.016 Which of the following sensory receptors of the integument is usually locatedin the epidermis, not in the dermis or hypodermis?

A. Pacinian corpuscle

B. free or bare nerve ending

C. Ruffin corpuscle

D. Meissner's corpuscle

E. Kraus end bulb

7.017 Anchoring proteins are found in a lamina lucida in which of the following?

A. epidermis

B. epidermo-dermal junction

C. papillary layer of dermis

D. reticular layer of dermis

E. hypodermis

7.018 Prickle cells are found in which of the following?

A. epidermis

B. epidermo-dermal junction

C. papillary layer of dermis

D. reticular layer of dermis

E. hypodermis

7.019 Birbeck granules are found in which of the following?

Page 188: Review Questions for Human Histology

A. epidermis

B. epidermo-dermal junction

C. papillary layer of dermis

D. reticular layer of dermis

E. hypodermis

7.020 A large amount of adipose tissue is found in which of the following?

A. epidermis

B. epidermo-dermal junction

C. papillary layer of dermis

D. reticular layer of dermis

E. hypodermic

7.021 Meissner's corpuscles are found in which of the following?

A. epidermis

B. epidermo-dermal junction

C. papillary layer of dermis

D. reticular layer of dermis

E. hypodermis

B. is correct.

All other choices are found in the dermis from its most superficial region just underthe epidermis to deep into the hypodermis.

B. is correct.

The lamina lucida (rara) is not the stratum lucidum. The lamina lucida is acomponent of the basement membrane (basal lamina) which is closest to the cellmembrane of the keratinocytes in the stratum basale. The denser lamina densa isclosest to the adjacent connective tissue.

A. is correct.

Page 189: Review Questions for Human Histology

This is another name for the spiny cells of the stratum spinosum. The prickle orspiny look is derived from the presence of many desmosomes on thesekeratinocytes which are accentuated when the tissue is fixed and some shrinkageartifact occurs.

A. is correct.

These are the dense granules found in the Langerhans cells which reside in theepidermis. These granules are rod shaped and have been called vermiformgranules and Langerhans granules.

E. is correct.

The hypodermis contains a significant amount of adipose tissue. It is the samelayer as the superficial fascia or tela subcutanea of gross anatomy.

C. is correct.

Meissner's corpuscles are mechanoreceptors for touch. They are found in thepapillary layer of the dermis, especially in the projection of this layer into theoverlying epidermis.

7.022 Glands secreting by the holocrine method of secretion are found in which ofthe following?

A. epidermis

B. epidermo-dermal junction

C. papillary layer of dermis

D. reticular layer of dermis

E. hypodermis

7.023 Keratinocytes are found in which of the following?

A. epidermis

B. epidenno-dermal junction

C. papillary layer of dermis

D. reticular layer of dermis

E. hypodermis

Page 190: Review Questions for Human Histology

7.024 Melanin synthesis is found in which of the following?

A. epidermis

B. epidermo-dermal junction

C. papillary layer of dermis

D. reticular layer of derrnis

E. hypodermis

7.025 Which of the following is avascular?

A. epidermis

B. epidermo-dermal junction

C. papillary layer of dermis

D. reticular layer of dermis

E. hypodermis

7.026 Dense irregular collagenous connective tissue is found in which of thefollowing?

A. epidermis

B. epidermo-dermal junction

C. papillary layer of dermis

D. reticular layer of dennis

E. hypodermis

7.027 Keratohyaline granules are found in which of the following?

A. epidermis

B. epidenno-dermal junction

C. papillary layer of dermis

D. reticular layer of dermis

Page 191: Review Questions for Human Histology

E. hypodermis

7.028 The eponychium is found in which of the following?

A. epidermis

B. epidenno-dermal junction

C. papillary layer of dermis

D. reticular layer of dermis

E. hypodermis

D. is correct.

The sebaceous glands associated with the hair follicles in the reticular layer of thedermis secrete via the holocrine method. One way to remember this is that the"whole" (holo-of holocrine) cell is the secretory product.

A. is correct.

Keratinocytes are the epithelial cells of the epidermis.

A. is correct.

Melanocytes are found in the epidermis, actually its stratum basale. Macrophagesof the papillary layer of the dermis can ingest melanin granules. When they do thisthey are called chromatophores.

A. is correct.

The epidermis actually is a stratified squamous dry or keratinized or cornifiedepithelium. All epithelia are avascular, receiving nutrients from the underlyingloose areolar connective tissue.

D. is correct.

The reticular layer of the dermis is composed of a type of connective tissue calleddense irregular collagenous CT.

A. is correct.

Keratohyaline granules are basophilic granules characteristic of the stratumgranulosum of the epidermis. They represent the source of the protein of softkeratin or the keratin of the epidermis as opposed to the hard keratin of the nails.

Page 192: Review Questions for Human Histology

A. is correct.

The eponychium as well as all other parts of the nail are all specializations of theepidermis. The eponychium is the proximal cuticle of the nail.

7.029 Vegetative intermitotic cells of cuboidal shape are found in which of thefollowing?

A. epidermis

B. epidenno-dermal junction

C. papillary layer of dermis

D. reticular layer of dermis

E. hypodennis

7.030 Arrector pili muscles are found in which of the following?

A. epidermis

B. epidenno-dermal junction

C. papillary layer of dermis

D. reticular layer of dermis

E. hypodennis

A. is correct.

These would be the stem cells of the stratum basale of the epidermis.

D. is correct.

These little smooth muscles run from the hair follicle deep in the reticular layer ofthe dermis obliquely to the outer most reaches of the reticular layer of the dermisand perhaps as far as the papillary layer of the dermis. They occur on the side ofthe follicle the hair angles away from. When they contract they actually pull thebasal region of the hair follicle obliquely toward the surface. The net effect is tostraighten the hair.

Page 193: Review Questions for Human Histology

SECTION 8: CARTILAGE AND BONE

8.001 Which of the following is the best place to find isogenous groups of cells?

A. chondrogenic layer of perichondrium

B. fibrous layer of perichondrium

C. interstices of tracheal ring of hyaline cartilage

D. fibrous layer of periosteum

E. osteogenic layer of periosteum

8.002 All of the following are components of hyaline cartilage EXCEPT:

A. collagen

B. elastin

C. hyaluronic acid

D. capillaries

E. tissue fluid

8.003 All of the following are found in or pertain to the chondrogenic layer of theperichondrium EXCEPT:

A. collagen

B. chondroblasts

C. appositional growth

D. interstitial growth

8.004 Where in the human body would one find fibrocartilage?

A. tracheal rings

B. pinna of the ear

C. intervertebral disc

Page 194: Review Questions for Human Histology

D. epiglottis

E. epiphyseal plate

8.005 Canaliculi contain all of the following EXCEPT:

A. tissue fluid

B. oxygen

C. cytoplasmic extensions of osteocytes

D. hydroxyapatite crystals

E. nutrients

8.006 All of the following pertain to or are involved in intramembranous ossificationEXCEPT:

A. mesenchymal cells

B. osteoblasts

C. chondrocytes

D. canaliculi

E. lacunae

C. is correct.

All other choices represent the appositional type of growth. Deep in a piece ofcartilage is where evidence of chondrocyte division or interstitial growth is found.

D. is correct.

Cartilage is avascular, and contains all other components in varying amounts.Cartilage contains an antiangiogenesis factor which prevents blood vesselinvasion into the cartilage under most circumstances.

D. is correct.

Chondroblasts differentiate in this inner region of the perichondrium and formmatrix which is added to the surface of matrix which had already been formed. Thisis known as appositional growth.

C. is correct.

Page 195: Review Questions for Human Histology

Fibrocartilage has few chondrocytes and large amounts of parallel bundles ofcollagen. It is characteristic of the intervertebral disc, pubic symphysis and intra-articular menisci.

D. is correct.

Hydroxyapatite crystals are found in mineralized bone, not within canaliculi.Canaliculi are small tunnels in bone through which tissue fluid, oxygen andnutrients are delivered to the osteocytes living in the lacunae. Osteocytes haveslender cytoplasmic extensions or processes which course within the canaliculiand actually contact processes of adjacent osteocytes.

C. is correct.

Intramembranous ossification is the formation of mineralized bone matrix frommesenchyme, as in the formation of some of the flat bones of the skull, without anyinvolvement of a cartilaginous model.

8.007 When chondrocytes hypertrophy during the process of endochondralossification, all of the following events eventually occur EXCEPT:

A. hypertrophied chondrocytes die

B. adjacent matrix calcifies

C. osteogenic cells invade the area

D. unmineralized osteoid is deposited on calcified cartilage surfaces

E. hypertrophied chondrocytes undergo interstitial growth

8.008 As a pin passes into an epiphyseal plate from the primary or diaphysealcenter of ossification to the secondary or epiphyseal center of ossification thefollowing regions or zones will be encountered in which sequence: 1=zone ofresting cartilage; 2=zone of hypertrophied cartilage; 3=zone of calcified cartilage;4=zone of proliferating cartilage; 5=zone of osteoid deposition.

A. 1-2-3-4-5

B. 5-4-3-2-1

C. 5-3-2-4-1

D. 4-5-1-3-2

E. 3-2-4-5-1

Page 196: Review Questions for Human Histology

8.009 As a pin passes from bone marrow into the center of a spicule of developingendochondral bone the following layers will be encountered in which sequence:1=calcified cartilage, 2=fresh, unmineralized osteoid; 3=mineralized osteoid;4=osteoblast.

A. 4-2-3-1

B. 1-3-2-4

C. 4-3-2-1

D. 1-2-4-3

E. 2-3-1-4

8.010 Which of the following structures in a fully grown adult bone would beclosest to the endosteum?

A. Haversian system

B. interstitial lamellae

C. inner circumferential lamellae

D. outer circumferential lamellae

E. osteogenic layer of periosteum

E. is correct.

The events listed in A - D are characteristic of endochondral ossification, E iscompletely false. These are the named zones one finds in the epiphyseal plate asone proceeds from the diaphysis toward the epiphyseal center of ossification.

C. is correct.

A. is correct.

A is the correct sequence from bone marrow to the surface of the spicule where anosteoblast would be busy laying down some fresh, unmineralized osteoidunderneath itself. Deep to the fresh osteoid the pin would encounter older osteoidwhich, because of its earlier deposition, would be mineralized. In the center of thisspicule of endochondral ossification would be a core of calcified cartilage.

C. is correct.

Page 197: Review Questions for Human Histology

As a bone reaches its adult size both the periosteum and endosteum lay downlamellae of bone which are not components of either the Haversian systems or theinterstitial lamellae. The osteogenic layer of the periosteum produces the outercircumferential lamellae. The osteogenic cells and the osteoblasts comprising theendosteum are involved in producing the inner circumferential lamellae.

8.011 Proliferation of chondrocytes in the epiphyseal plate can be stimulated by:

A. somatotrophin

B. somatostatin

C. parathyroid hormone

D. calcitonin

E. erythropoietin

8.012 Which of the following is not covered with a perichondrium?

A. elastic core of the epiglottis

B. articular cartilage in a synovial joint

C. cartilage ring in the trachea

D. cartilage model of the humerus in a fetus

E. fibrocartilage of pubic symphysis

8.013 A pellet containing a protein is placed under the skin. Within a few daysmesenchymal cells migrate into the area and differentiate into osteoblasts whichbegin to form bone. Which of the follow best describes this situation?

A. the protein in the pellet was collagen

B. the protein in the pellet was elastin

C. the protein in the pellet was osteogenic protein

D. the protein in the pellet was erythropoietin

E. the protein in the pellet was synovial protein

8.014 Where in mature, compact bone are the oldest lamellae located?

A. adjacent to the Haversian canal

Page 198: Review Questions for Human Histology

B. adjacent to the cement line

C. half way between the Haversian canal and the cement line

D. in interstitial lamellae between Haversian systems

A. is correct.

Somatotrophin or growth hormone stimulates the proliferation and maturation ofchrondrocytes in the epiphyseal plate. If there is an excess of somatotrophin, ascould be the case with a benign, functioning adenoma of the hypophysis,excessive growth of the epiphyseal plate following by endochondral ossificationcan result in abnormally tall individuals (giantism). Conversely, too littlesomatotrophin will result in stunted individuals.

B. is correct.

The hyaline cartilage facing the synovial fluid in a synovial joint is not covered by aperichondrium. The articular cartilage derives its nourishment from the synovialfluid. It is the articular cartilage, which, with age becomes inflamed anddegenerative in the disease osteoarthritis.

C. is correct.

Human osteogenic protein, also known as "bone morphogenic protein", has beenisolated, purified and manufactured by recombinant DNA techniques. Likeerythropoietin its site of manufacture is the kidney, where the mRNA for BMP hasbeen localized. BMP has been used clinically to get tooth dentine to grow backwhich normally does not happen once bacteria have created a hole in the dentinewhich eventually leads to bacterial invasion of the pulp and thus the need for a rootcanal procedure. BMP has also been successfully used in healing bone fractureswhich have large gaps between the ends of the broken bones.

D. is correct.

As compact bone is remodeled by osteoclastic activity and the formation ofresorption tunnels, not all of the pre-existing Haversian systems get resorbed. As anew Haversian system is formed from the outer walls of the resorption tunnelinward, some lamellae from the pre-existing Haversian system remain outside theconfines of the newly formed Haversian system. These lamellae are the interstitiallamellae and are the oldest bone.

8.015 Which sequence is the correct sequence for the healing of a simple fractureof a long bone: 1=bone breaks; 2=cartilage is replaced by cancellous bone;3=osteocytes in Haversian systems die; 4=some osteogenic cells begin to

Page 199: Review Questions for Human Histology

differentiate into chondroblasts; 5=a callus forms a bridge across the break.

A. 1-2-3-4-5

B. 1-3-4-5-2

C. 1-4-5-2-3

D. 1-511-3-2

E. 1-2-5-3-4

8.016 Which of the following is/are used in the formation of a long bone?

A. endochondral ossification

B. intramembranous ossification

C. both of the above

D. none of the above

8.017 Which of the following is/are used in the formation of a flat bone of the skull?

A. endochondral ossification

B. intramembranous ossification

C. both of the above

D. none of the above

8.018 Osteocytes get trapped in lacunae during which of the following?

A. endochondral ossification

B. intramembranous ossification

C. both of the above

D. none of the above

8.019 Unmineralized osteoid is found adjacent to an osteoblast in which of thefollowing?

A. endochondral ossification

Page 200: Review Questions for Human Histology

B. intramembranous ossification

C. both of the above

D. none of the above

8.020 Interstitial growth of cells in lacunae occurs in which of the following?

A. endochondral ossification

B. intramembranous ossification

C. both of the above

D. none of the above

B. is correct.

A. is correct.

Long bones begin as models of hyaline cartilage in the fetus. These cartilaginousbones are "turned into" bones via endochondral ossification.

B. is correct.

The flat bones of the skull are formed via the intramembranous route.

C. is correct.

Osteoblasts can secrete enough fresh or unmineralized osteoid that they gettrapped in what they produce. A trapped osteoblast shuts down most of itssynthetic machinery and becomes an osteocyte.

C. is correct.

Osteoblasts "do their thing", i.e. secrete fresh or unmineralized osteoid during bothendochrondral and intramembranous ossification.

A. is correct.

Bone can only grow by appositional growth. But in endochrondral ossification thereis a lot of growth of chondrocytes, be it in the epiphyseal plate or in the model of along bone in the fetus. Mitotic growth of chondrocytes is called interstitial growth asopposed to appositional growth, which is the addition of cells and fibers to theoutside of a model or piece of cartilage instead of mitotic events occurring in thechrondrocytes inside the cartilage itself, i.e. interstitial growth.

Page 201: Review Questions for Human Histology

8.021 Hyaline cartilage matrix is used as a scaffold in which of the following?

A. endochondral ossification

B. intramembranous ossification

C. both of the above

D. none of the above

8.022 Fresh osteoid eventually becomes mineralized osteoid in which of thefollowing?

A. endochondral ossification

B. intramembranous ossification

C. both of the above

D. none of the above

8.023 Spicules are present sometimes during which of the following?

A. endochondral ossification

B. intramembranous ossification

C. both of the above

D. none of the above

8.024 Which of the following is used in growth in length of a long bone?

A. endochondral ossification

B. intramembranous ossification

C. both of the above

D. none of the above

8.025 Myoblasts play a significant role in which of the following/

A. endochondral ossification

B. intramembranous ossification

Page 202: Review Questions for Human Histology

C. both of the above

D. none of the above

8.026 Chondrocytes hypertrophy and die in which of the following?

A. endochondral ossification

B. intramembranous ossification

C. both of the above

D. none of the above

8.027 Which of the following has a ruffled border?

A. osteoblast

B. osteocyte

C. osteoclast

A. is correct.

Endochondral ossification is exactly what it says, making bone from a priorcartilaginous model. The models of the long bones in the fetus are made of hyalinecartilage.

C. is correct.

This process is found in both intramembranous and endochondral ossification.

C. is correct.

Spicules of calcified cartilage and cancellous bone are found in endochrondralossification. Spicules of bone (cancellous bone) are found in intramembranousossification.

A. is correct.

Endochondral ossification is the process used by the epiphyseal plate to changecartilage into bone and that bone eventually into marrow cavity. In this way thelength of a long bone increases.

D. is correct.

Myoblasts are stem cells for myogenesis or the formation of muscle and they have

Page 203: Review Questions for Human Histology

nothing to do with either endochrondral or intramembranous ossification.

A. is correct.

Endochondral ossification is the type of ossification which takes a cartilaginousmodel of a bone and turns it into bone using a calcified cartilaginous substrate.Chondrocytes must die in this process in order to make way for the osteoblasts.Chondrocytes die by committing suicide by secreting alkaline phosphatase whichcauses the cartilaginous matrix to calcify, thereby shutting off the diffusion ofnutrients to the chondrocytes.

C. is correct.

This is a characteristic of the osteoclast. The ruffled border faces the bony surfacewhich is being resorbed.

8.028 Which of the following has extensive RER in the cytoplasm?

A. osteoblast

B. osteocyte

C. osteoclast

8.029 Which of the following is normally multinucleated?

A. osteoblast

B. osteocyte

C. osteoclast

8.030 Gap junctions occur on cytoplasmic processes of which of the following?

A. osteoblast

B. osteocyte

C. osteoclast

8.031 The activity of which of the following is greatly increased by PTH?

A. osteoblast

B. osteocyte

C. osteoclast

Page 204: Review Questions for Human Histology

8.032 Which of the following needs vitamin C for successful fibrogenesis?

A. osteoblast

B. osteocyte

C. osteoclast

8.033 The cytoplasm of which of the following contains many membrane boundvesicles and vacuoles?

A. osteoblast

B. osteocyte

C. osteoclast

8.034 A well developed Golgi apparatus would be found in which of the following?

A. osteoblast

B. osteocyte

C. osteoclast

8.035 The product of which of the following is unmineralized osteoid?

A osteoblast

B. osteocyte

C. osteoclast

A. is correct.

RER is required by the osteoblast in its synthesis of procollagen.

C. is correct.

Osteoclasts are multinucleated giant cells. They are believed to be formed by thefusion of many macrophages derived from the blood monocyte pool.

B. is correct.

Osteocytes have cytoplasmic "arms" which extend into the canaliculi. Theseprocesses touch and gap junctions can be found here.

Page 205: Review Questions for Human Histology

C. is correct.

The osteoclast is the primary target for parathyroid hormone which significantlyincreases the number and activity of osteoclasts.

A. is correct.

Like the fibroblast, the osteoblast requires the presence of vitamin C for normalsynthesis of collagen fibers.

C. is correct.

This is characteristic of the osteoclast as it "mines" mineralized bone.

A. is correct.

This applies to the osteoblast as it synthesizes and packages for secretion theprocollagen molecules.

A. is correct.

This applies to the osteoblast. It is similar in ultrastructure to the fibroblast becauseit makes collagen fibers which are a major component of bone. Like fibroblasts,osteoblasts require vitamin C to produce collagen. Fresh osteoid is unmineralized.If there is no vitamin D deficiency, the fresh osteoid will become mineralized soon.

8.036 Which of the following would NOT be found in the endosteum?

A. osteoblast

B. osteocyte

C. osteoclast

8.037 Which of the following is only found in complete lacunae?

A. osteoblast

B. osteocyte

C. osteoclast

B. is correct.

Both the osteoblast and the osteoclast reside in the endosteal layer, osteocytes aretrapped in lacunae in compact and cancellous bone and are not normalcomponents of the endosteum.

Page 206: Review Questions for Human Histology

B. is correct.

Osteocytes are trapped in bony matrix and thus are completely surrounded by thematrix which forms little "rooms" or lacunae. Osteoclasts sit in a depression in thebony surface called a Howship's lacuna, but osteoclasts are not completelysurrounded by matrix in the sense that osteocytes are.

Page 207: Review Questions for Human Histology

SECTION 9: BLOOD AND BONE MARROW

9.001 What number represents a normal hematocrit?

A. 25

B. 35

C. 45

D. 55

E. 65

9.002 The following description applies to which blood cell type: major job is to eatbacteria in the blood stream and/or exit the blood stream through vessel walls andeat bacteria in connective tissue areas.

A. eosinophil

B. neutrophil

C. lymphocyte

D. monocyte

E. basophil

9.003 The following description applies to which blood cell type: is known tophagocytize antigen-antibody complexes and thereby plays a role in the function ofthe immune system.

A. eosinophil

B. neutrophil

C. lymphocyte

D. monocyte

E. basophil

C. is correct.

When whole blood is centrifuged the formed elements are packed at the bottom of

Page 208: Review Questions for Human Histology

the tube and the plasma or fluid component is at the top. The percent of the totalvolume in the tube taken up by the plasma is about 55% and the packed, formedelements represent 45%. A hematocrit of 25% would indicate a significantdecrease in the quantity of formed elements. This could be due to an anemia inwhich fewer than normal cells are produced in the marrow or it could be due achronic hemorrhage into, for example, the GI tract or through a "bleeder" aftersurgery. A hematocrit of 55% could represent an abnormal increase in the quantityof formed elements such as can be seen in some leukemias or in people who arepolycythemic in which case they have abnormally high numbers of red blood cells.

B. is correct.

This is the job of the neutrophil.

A. is correct.

Eosinophils also contain major basic protein in their granules. In fact the densecore or "internum" of the eosinophilic granule, as seen with the electronmicroscope, is where this arginine rich protein is located. The major basic proteinis thought to be involved in the killing of parasites phagocytized by the eosinophil.

9.004 The following description applies to which blood cell type: exits the bloodstream and lives and functions in connective tissue spaces where it is recognizedand termed a histiocyte or tissue macrophage.

A. eosinophil

B. neutrophil

C. lymphocyte

D. monocyte

E. basophil

9.005 Which of the following cell types can differentiate into and function as aplasma cell?

A. eosinophil

B. neutrophil

C. lymphocyte

D. monocyte

Page 209: Review Questions for Human Histology

E. basophil

9.006 The following description applies to which cell type: terminally differentiatedcell, clock-faced heterochromatin pattern in nucleus, prominent Golgi apparatus,extensive amounts of rough endoplasmic reticulum.

A. tissue macrophage

B. T lymphocyte

C. B lymphocyte

D. mast cell

E. plasma cell

D. is correct.

Monocytes are sort of "tissue macrophages" taking a free ride in the blood streamto their workplace. They exit the blood through vessel walls and wander around inconnective tissue areas eating many different things from bacteria to cell/tissuenormal debris to foreign objects. Indeed macrophages may fuse together and formmultinucleated giant cells which will eat or attempt to eat large objects such assplinter fibers, sutures, etc. In this case these giant cells are called "foreign bodygiant cells". Macrophage/monocyte fusion also is believed to be the process inwhich osteoclasts are formed.

C. is correct.

Lymphocytes in peripheral blood come in many different functional types: B cellsderived from bone marrow, T cells derived from the thymus, Nk or natural killercells, and null cells or lymphocytes which can not be identified by specialtechniques to be of the B, T, or Nk classes. B lymphocytes can respond to a foreignantigen by differentiating into a morphologically very different type of cell theplasma cell. Plasma cells synthesize and release one kind (monoclonal) ofantibody. This antibody circulates in the blood stream and is called humoralantibody.

E. is correct.

This is a classic morphological description of the plasma cell. Terminallydifferentiated means that as it changes from lymphocyte morphology to plasma cellmorphology (plasmablast-proplasmacyte- plasmacyte stages) in response to anantigenic stimulus, it loses the capability to divide and becomes a non-replicator orfixed post-mitotic cell which can not produce daughter cells. Fully differentiatedplasma cells live and function for about 2 weeks.

Page 210: Review Questions for Human Histology

9.007 The following description best describes which formed element of blood:microtubular system, canaliculi connecting to the surface, dense granulescontaining serotonin.

A. neutrophil

B. platelet

C. T lymphocyte

D. B lymphocyte

E. monocyte

9.008 If some red blood cells are placed in 0.9% sodium chloride what will happento them?

A. nothing

B. they will take up water, swell and burst

C. they will lose water, wrinkle and shrink

9.009 Which of the following blood cell types has had its peripheral membraneproteins, which are on the P surface of its plasmalemma, well studied?

A. neutrophil

B. platelet

C. RBC

D. T lymphocyte

E. Nk lymphocyte

9.010 In histological preparations what is the diameter of a RBC in microns?

A. 6.7

B. 7.0

C. 7.3

D. 7.6

Page 211: Review Questions for Human Histology

E. 7.9

B. is correct.

The platelet uses these components in its role in clot formation. When a vesselwall is damaged and the endothelial lining is interrupted, components of the vesselwall such as collagen become exposed to the blood. When circulating plateletscome in contact with the exposed collagen they stick to it and to each other forminga primary aggregation. They also are stimulated by this interaction to release thecontents of their granules. The serotonin will locally cause the smooth muscle inthe vessel wall to contract thereby slowing the blood flow. The other granules inthe platelets release a variety of platelet specific substances which participate withsome plasma proteins to eventually form fibrin strands. This is sort of likemanufacturing a fishing net in the stream. The fibrin strands (net) trap the formedelements of blood. Together all of this is called a thrombus.

A. is correct.

0.9% NaCl is saline which is the salt concentration which is isotonic with red bloodcells. 1.5% NaCI would be hypertonic to the RBCs and they would lose their waterto the salt and shrink and wrinkle. These RBCs are called crenated RBCs. 0.4%NaCl would be hypotonic to the RBCs and they would take up water or waterwould move into the RBC causing it to swell and burst.

C. is correct.

Because RBCs are so numerous and accessible and they are anucleate they havebeen an excellent cell in which to study the detailed structure and biochemistry ofthe plasmalemma. One of the proteins associated with the P surface which hasbeen studied extensively is spectrin.

C. is correct.

In the living state in an isotonic medium the average diameter of a RBC is 7.5microns, but in fixed and stained preparations it shrinks down to a mean of 7.3microns. Since RBCs are found in vessels in almost all histological slides, findinga good profile view of a RBC is the same as finding a tiny "histological ruler".

9.011 The plasmalemma of the RBC is like a bag containing proteins and water.What percent of the total contents of a RBC is hemoglobin?

A. 15

B. 25

C. 35

Page 212: Review Questions for Human Histology

D. 45

E. 55

9.012 In a patient with sickle cell anemia what fundamental change has occurredin the hemoglobin?

A. glutamic acid substituted for valine

B. valine substituted for glutamic acid

C. alanine substituted for phenylalanine

D. phenylalanine substituted for alanine

E. uridine substituted for thymidine

9.013 Antigen-IgE antibody coupling occurs on the surface of this cell resulting in adegranulation and release of histamine and other active substances. Identify thecell.

A. eosinophil

B. basophil

C. neutrophil

D. B lymphocyte

E. platelet

9.014 Myeloid tissue is the term used to describe which of the following?

A. yellow bone marrow

B. red bone marrow

C. all formed elements in peripheral blood

D. all classes of lymphocytes

E. lymphatic tissue

9.015 Myeloid tissue is composed of parenchymal and stromal elements. All of thefollowing are nonparenchymal elements in red bone marrow EXCEPT:

Page 213: Review Questions for Human Histology

A. fibroblast

B. endothelial cell

C. reticular cell

D. adipose cell

E. myeloblast

C. is correct.

A RBC contains about a 35% solution of the normal adult hemoglobin orhemoglobin A. In the fetal RBC the hemoglobin is not A but hemoglobin F.

B. is correct.

In this case the genetic code of GAA for glutamic acid is changed to GUA whichcodes for valine instead of glutamic acid. The resulting hemoglobin is not HbA butHbS. When HbS loses its oxygen it undergoes profound structural changes whichrender the normally deformable RBC rigid or inflexible and the shape of the cellchanges to a sickle.

B. is correct.

Basophils, like mast cells, respond to an antigen-IgE antibody coupling at their cellsurface with release of the contents of their granules. The release of biologicallyactive compounds by these cells can cause systemic or local hypersensitivityreactions.

B. is correct.

The terms red bone marrow and myeloid tissue can be used interchangeably. Twofamous sites for the clinical aspiration of myeloid tissue are the sternum and theiliac crest.

E. is correct.

The myeloblast is a cell in the granulocyte line of hematopoiesis and therefore isone of the identifiable parenchymal cells of red bone marrow. All other choices areexamples of the variety of stromal cells found in myeloid tissue. The stromal cellsare the source of the hematopoietic growth factors which impact on the growth andcytodifferentiation of the parenchymal cells.

9.016 During hematopoiesis cells move from one type of cell kinetic compartmentinto a different cell kinetic compartment (functional not morphological

Page 214: Review Questions for Human Histology

compartments). Place the following cell kinetic compartments in correct order withrespect to hematopoiesis: 1=mature or near mature cells, 2=cells whichcontinuously divide but only some of which differentiate while others do notdifferentiate, 3=cells which divide and differentiate.

A. 1-2-3

B. 3-2-1

C. 2-1-3

D. 2-3-1

E. 3-1-2

9.017 Which cell in the granulocyte line of maturation has large numbers of onlyuncommitted, azurophilic granules in its cytoplasm?

A. myeloblast

B. neutrophil

C. myelocyte

D. metamyelocyte

E. promyelocyte

9.018 Which of the following bone marrow cells has the highest concentration offree polysomes in its cytoplasm?

A. promyelocyte

B. pronormoblast

C. basophilic metamyelocyte

D. basophilic normoblast

E. orthochromatophilic normoblast

9.019 Which of the following bone marrow cells normally has repeated rounds ofDNA synthesis without accompanying cytokinesis or cell division?

A. pronormoblast

B. polychromatophilic normoblast

Page 215: Review Questions for Human Histology

C. monocytoblast

D. lymphoblast

E. megakaryoblast

D. is correct.

This is basically the same hierarchy found in stratified epithelia, i.e. first is the stemcell or vegetative intermitotic compartment, second is the differentiating intermitoticgroup and third is the functional, mature group. Only the stem cell compartment is"self-sustaining".

E. is correct.

The myeloblast is characterized by no or just a few azurophilic granules whereasthe promyelocyte (progranulocyte) has large numbers of these granules in itscytoplasm. It is at the myelocyte stage of differentiation when the specific nature ofthe granules is apparent enough to classify the myelocyte as either neutrophilic,basophilic or eosinophilic. Some authors refer to the "dawn of neutrophilia" asoccurring at the myelocyte stage in the cytodifferentiation of the neutrophil.

D. is correct.

The first cell in the erythroid line is the pronormoblast and it does have a diffuselybasophilic cytoplasm due to the presence of polysomes. However, theconcentration of these polysomes significantly increases as the cell continues todifferentiate to reach the maximal concentration in the basophilic normoblast stage.The polysomes have mRNA on them and this is where the genetic code is beingtranslated into the protein hemoglobin.

E. is correct.

The megakaryoblast undergoes endoreduplication as it matures. Thus the ploidyincreases from 2N-4N- 8N-16N-32N and even 64N in the most maturemegakaryocyte. The 64N nucleus is huge and multilobated and can "govern" alarger amount of cytoplasm. More platelets are produced from a 64Nmegakaryocyte than from one with 4N.

9.020 A patient in your care has a peripheral blood workup with the following facts:WBC count of 38,000/cubic mm and most of the circulating WBCs are large cellswith large nuclei containing 2-3 prominent nucleoli, the cytoplasm contains manyazurophilic granules. What is the diagnosis?

A. severe bacterial infection

Page 216: Review Questions for Human Histology

B. chronic hemorrhage

C. myeloblastic leukemia

D. promyelocytic leukemia

E. sickle cell anemia

9.021 A patient in your care has severe neutropenia. Treatment with which of thefollowing growth factors would be beneficial for this person?

A. NGF

B. EGF

C. M-CSF

D. G-CSF

E. EPO

9.022 At which of the following steps does nuclear extrusion occur?

A. megakaryocyte - platelet

B. polychromatophilic normoblast - orthochromatophilic normoblast

C. orthochromatophilic normoblast - reticulocyte

D. monocytoblast - monocyte

E. reticulocyte - mature RBC

9.023 In a normal bone marrow smear, what is the ratio of the number of cells in themyeloid line to those in the erythroid line?

A. 1-1

B. 2-1

C. 3-1

D. 1-2

E. 1-3

9.024 A patient in your care has a reticulocyte count of 13%. What could be going

Page 217: Review Questions for Human Histology

on?

A. severe bacterial infection

B. chronic, significant hemorrhage

C. myeloblastic leukemia

D. thrombocytopenia

E. sickle cell anemia

D. is correct.

In this case differentiation in the myeloid line has stopped between thepromyelocyte and metamyelocyte stages. In addition to the bone marrow filling upwith promyelocytes, many of these cells gain access to the circulation.

D. is correct.

The acronyms were used because these growth factors are becoming as wellknown as other substances which are recognized by their acronyms such as TSH,ACTH, etc. Nerve growth factor and epidermal growth factor stimulate theirrespective target cells. Erythropoietin stimulates entry into and/or progress througherythrocytopoiesis. Monocyte colony stimulating factor would enhancemonocytopoiesis. A recombinant, human G-CSF or rhG-CSF has been usedclinically to stimulate granulocytopoiesis in patients with severe neutropenia due tochemotherapy.

C. is correct.

When the cytoplasmic concentration of hemoglobin reaches a certain thresholdlevel the orthochromatophilic normoblast is stimulated to extrude or exocytose itsentire nucleus.

C. is correct.

A normal M/E or myeloid/erythroid ratio is about 3-1. This means that in normalmarrow one should see 3 cells in granulocytopoiesis for every one cell inerythrocytopoiesis. This is a reflection of the fact that mature granulocytes only livefor hours to a few days whereas RBCs live for an average of 120 days.

B. is correct.

Significant loss of RBCs as in the case of a bleeding duodenal ulcer not only canresult in a low hematocrit but also in an elevated reticulocyte count. In this case,

Page 218: Review Questions for Human Histology

peripheral loss of large numbers of RBCs feeds back to the erythroid line with theresult that erythrocytopoiesis is enhanced and immature RBCs or reticulocytes arereleased from the marrow.

9.025 In which of the following would you expect to see smooth cytoplasm which isabout 50/50 basophiliceosinophilic?

A. pronormoblast or myeloblast

B. basophilic normoblast or promyelocyte

C. polychromatophilic normoblast or myelocyte

D. orthochromatophilic normoblast or metamyelocyte

E. reticulocyte or band/stab form

9.026 After nuclear extrusion during erythrocytopoiesis the anucleate cell producedis very slightly basophilic. What causes this hint of basophilia?

A. granules containing heparin and histamine

B. polysomes

C. oxyhemoglobin A

D. spectrin

E. lysosomes

9.027 Which of the following blood cells has 3-5 nuclear lobes and two differentsized cytoplasmic granules?

A. eosinophil

B. neutrophil

C. lymphocyte

D. monocyte

E. basophil

9.028 Which of the following has 2 nuclear lobes and large cytoplasmic granulescontaining major basic protein?

A. eosinophil

Page 219: Review Questions for Human Histology

B. neutrophil

C. lymphocyte

D. monocyte

E. basophil

9.029 Which of the following has a bean-shaped nucleus and azurophiliccytoplasmic granules?

A. eosinophil

B. neutrophil

C. lymphocyte

D. monocyte

E. basophil

9.030 Which of the following has 2 nuclear lobes and large granules containingheparin and histamine?

A. eosinophil

B. neutrophil

C. lymphocyte

D. monocyte

E. basophil

C. is correct.

The polychromatophilic normoblast has non-granular cytoplasm which has twothings going on in it. One is the presence of polysomes and these areas stainbasophilic. The other is the presence of hemoglobin which is manufactured on thepolysomes. The hemoglobin stains eosinophilic. Therefore a polychromatophilicnormoblast would have a mixture of basophilic and eosinophilic areas in itscytoplasm.

B. is correct.

The diffusely basophilic erythrocyte or DBE is diffusely basophilic because it still

Page 220: Review Questions for Human Histology

contains a few polysomes. If this cell is stained with a special staining technique(brilliant cresyl blue) the ribonucleoprotein is precipitated into a web-like orreticulated structure and the cell is called a reticulocyte.

B. is correct.

The more lobes the more mature the PMN. PMNs have two different size granulesin their cytoplasm; large and small granules. The large granules average 700nmwhereas the smaller granules average 400nm. The smaller granules containphagocytins. The large granules contain many hydrolytic enzymes.

A. is correct.

The major basic protein in the lysosomes of the eosinophil, being strongly basic,attracts the acidic dye eosin.

D. is correct.

The typical monocyte has a bean-shaped nucleus and a clumpy-stringy (spaghetti+ meatballs) chromatin pattern and a cytoplasm containing lyssomes (azurophilicgranles).

E. is correct.

This describes the basophil.

9.031 Which of the following has a heterochromatinized nucleus and a thin rim ofcytoplasm?

A. eosinophil

B. neutrophil

C. lymphocyte

D. monocyte

E. basophil

9.032 What is the average percentage of eosinophils in a normal differential count?

A. 70%

B. 20%

C. 6%

Page 221: Review Questions for Human Histology

D. 3%

E. 1%

9.033 What is the average percentage of neutrophils in a normal differential count?

A. 70%

B. 20%

C. 6%

D. 3%

E. 1%

9.034 What is the average percentage of basophils in a normal differential count?

A. 70%

B. 20%

C. 6%

D. 3%

E. 1%

9.035 What is the average percentage of monocytes in a normal differential count?

A. 70%

B. 20%

C. 6%

D. 3%

E. 1%

9.036 What is the average percentage of lymphocytes in a normal differentialcount?

A. 70%

B. 20%

Page 222: Review Questions for Human Histology

C. 6%

D. 3%

E. 1%

C. is correct.

This is a pretty accurate description of a lymphocyte.

D. is correct.

For every 100 WBCs counted there should be about 3 eosinophils seen.

A. is correct.

If a differential count comes back as 67% PMNs it is considered normal.

E. is correct.

The normal range for basophils in peripheral blood is from 0.5% to 1.5%.

C. is correct.

This is the normal percentage for monocytes.

B. is correct.

Changes in these approximate normal numbers (the normal ranges extend frombelow to above the numbers given) tell clinicians a lot about what might be goingon in a patient. Thus peripheral blood is a "window" into the body. For example, adifferential count of 12% eosinophils indicates a hay fever episode or a parasiteinfection.

9.037 What is the normal number of leukocytes in an aliquot of peripheral blood?

A. 250,000/microliter

B. 5 million/microliter

C. 7000/microliter

9.038 What is the normal number of platelets in an aliquot of peripheral blood?

A. 250,000/microliter

B. 5 million/microliter

Page 223: Review Questions for Human Histology

C. 7000/microliter

9.039 What is the normal number of erythrocytes in an aliquot of peripheral blood?

A. 250,000/microliter

B. 5 million/microliter

C. 7000/microliter

9.040 A 16N megakaryocyte leads what kind of mitotic cell life?

A. vegetative intermitotic

B. differentiating intermitotic

C. reverting postmitotic

D. fixed postmitotic

9.041 The neutrophilic metamyelocyte leads what kind of mitotic cell life?

A. vegetative intermitotic

B. differentiating intermitotic

C. reverting postmitotic

D. fixed postmitotic

9.042 The reticulocyte leads what kind of mitotic cell life?

A. vegetative intermitotic

B. differentiating intermitotic

C. reverting postmitotic

D. fixed postmitotic

C. is correct.

This is the approximate number of white blood cells in a microliter of peripheralblood.

A. is correct.

Page 224: Review Questions for Human Histology

This is approximately the normal number of platelets in a microliter of blood.

B. is correct.

The numbers used above are not the ranges but a single number from within thenormal range. These approximate concentrations for normal blood are veryimportant to the clinician. Increases or decreases in these concentrations canprovide useful diagnostic data. For example, if the platelet count drops to50,000/microliter either too many platelets are dying early or the bone marrow isnot producing enough. Respectively, these situations would indicate athrombocytopenia vs. a thrombocytosis. The same is true for leukocytes;leukocytopenia vs. leukocytosis. For the red cells the terms anemia vs.erythrocytosis or polycythemia would apply.

B. is correct.

The standard classification of cell kinetic lives falls apart when we deal with themegakaryocyte line of cytodifferentiation, because megakaryocytes undergoendoreduplication of their DNA without accompanying cytokinetic events. The 16Ncell in the question could go through one or two more rounds of endoreduplicationand become a 32N or 64N megakaryocyte before "shedding" platelets.

D. is correct.

This cell is an example of a FPM.

D. is correct.

This cell has no nucleus. It is in the same kinetic place in the erythroid line as theband/stab is in the myeloid line. This cell kinetic place is that of the FPM.

9.043 The megakaryocytoblast leads what kind of mitotic cell life?

A. vegetative intermitotic

B. differentiating intermitotic

C. reverting postmitotic

D. fixed postmitotic

9.044 The pronormoblast leads what kind of mitotic cell life?

A. vegetative intermitotic

B. differentiating intermitotic

Page 225: Review Questions for Human Histology

C. reverting postmitotic

D. fixed postmitotic

9.045 The pluripotential cell found in bone marrow leads what kind of mitotic celllife?

A. vegetative intermitotic

B. differentiating intermitotic

C. reverting postmitotic

D. fixed postmitotic

9.046 The promyelocyte leads which kind of mitotic cell life?

A. vegetative intermitotic

B. differentiating intermitotic

C. reverting postmitotic

D. fixed postmitotic

9.047 The orthochromatophilic normoblast leads which kind of mitotic cell life?

A. vegetative intermitotic

B. differentiating intennitotic

C. reverting postmitotic

D. fixed postmitotic

9.048 The polychromatophilic normoblast leads which kind of mitotic cell life?

A. vegetative intermitotic

B. differentiating intennitotic

C. reverting postmitotic

D. fixed postmitotic

9.049 The neutrophilic myelocyte leads which kind of mitotic cell life?

Page 226: Review Questions for Human Histology

A. vegetative intermitotic

B. differentiating intermitotic

C. reverting postmitotic

D. fixed postmitotic

B. is correct.

The megakaryocytoblast is not a stem cell and therefore is not a VIM, and it is not afixed postmitotic either. It is a DIM.

B. is correct.

The prononnoblast is a differentiating intermitotic because it undergoescytodifferentiation and retains the capability to divide.

A. is correct.

These are true stem cells, therefore classified as VIMs.

B. is correct.

The promyelocyte is an example of a cell which can undergo cytodifferentiationand cell division, thereby being classified as a differentiating intermitotic.

D. is correct.

In the erythroid line of cytodifferentiation this is the first cell which is classified as afixed postmitotic.

B. is correct.

This is the last cell in the erythroid line which can still undergo cell division,therefore it is an example of a differentiating intermitotic.

B. is correct.

Stem cells or VIMs are those cells less differentiated than the first cells in the DIMcompartment which are the myeloblast, erythroblast or pronormoblast,monocytoblast, and the megakaryocytoblast. The members of the FPM group arethe most mature cells in each line of cytodifferentiation and the next few cells backup in the line, for example, the orthochromatophilic and metamyelocyte stages.

9.050 The first clear indication of characteristic cytoplasmic granules (neutrophilic,basophilic or eosinophilic) appears in which of the following?

Page 227: Review Questions for Human Histology

A. prononnoblast or myeloblast

B. basophilic normoblast or promyelocyte

C. polychromatophilic normoblast or myelocyte

D. orthochromatophilic normoblast or metamyelocyte

E. reticulocyte or band/stab form

9.051 The cytoplasm of which of the following is loaded with "free" polysomes?

A. pronormoblast or myeloblast

B. basophilic normoblast or promyelocyte

C. polychromatophilic normoblast or myelocyte

D. orthochromatophilic normoblast or metamyelocyte

E. reticulocyte or band/stab form

9.052 Nuclear indentation first begins to be noticed in which of the following?

A. pronormoblast or myeloblast

B. basophilic normoblast or promyelocyte

C. polychromatophilic normoblast or myelocyte

D. orthochromatophilic normoblast or metamyelocyte

E. reticulocyte or band/stab form

9.053 Nuclear extrusion occurs in which of the following?

A. pronormoblast or myeloblast

B. basophilic normoblast or promyelocyte

C. polychromatophilic normoblast or myelocyte

D. orthochromatophilic normoblast or metamyelocyte

E. reticulocyte or band/stab form

Page 228: Review Questions for Human Histology

9.054 Immature, azurophilic granules fill cytoplasm in which of the following?

A. pronormoblast or myeloblast

B. basophilic normoblast or promyelocyte

C. polychromatophilic normoblast or myelocyte

D. orthochromatophilic normoblast or metamyelocyte

E. reticulocyte or band/stab form

9.055 For which of the following is about 1% is considered normal in peripheralblood?

A. pronormoblast or myeloblast

B. basophilic normoblast or promyelocyte

C. polychromatophilic normoblast or myelocyte

D. orthochromatophilic normoblast or metamyelocyte

E. reticulocyte or band/stab form

C. is correct.

The dawn of neutrophilia, or basophilia, or eosinophilia occurs at the myelocytestage. Prior to this stage one can not tell which way the promeylocyte will go; N orB or E.

B. is correct.

The mother (pronormoblast) of this particular cell has some polysomes in itscytoplasm, but at the basophilic normoblast stage the concentration of cytoplasmicpolysomes is greatly increased. Now the stage is set for the translation of mRNA,on these ribosomes, into the protein hemoglobin.

D. is correct.

The first signs of nuclear indentation force a name change from myelocyte (roundnucleus) to metamyelocyte (indented nucleus). When the indentation moves pastthe 50% point it is time to call this cell a band/stab.

D. is correct.

When the amount of hemoglobin reaches a threshold level in the cell it triggers

Page 229: Review Questions for Human Histology

exocytosis of the cell's pyknotic nucleus. The entire nucleus is shed, leavingbehind an anucleate cell, the reticulocyte or diffusely basophilic erythrocyte whichstill contains a fair amount of polysomes.

B. is correct.

This is the hallmark of the promyelocyte. It is as if the cell has not yet decided tomake either mature neutrophilic, basophilic or eosinophilic granules.

E. is correct.

Normal peripheral blood is allowed to contain about 1% bands or reticulocytes.Increases in this normal percentage can indicate a variety of clinical conditions.

9.056 In peripheral blood, 13% of which of the following would signal septicemia (abacterial infection in the blood stream)?

A. pronormoblast or myeloblast

B. basophilic normoblast or promyelocyte

C. polychromatophilic normoblast or myelocyte

D. orthochromatophilic normoblast or metamyelocyte

E. reticulocyte or band/stab form

9.057 Which of the following would contain a very delicate chromatin pattern andprominent nucleoli?

A. pronormoblast or myeloblast

B. basophilic normoblast or promyelocyte

C. polychromatophilic normoblast or myelocyte

D. orthochromatophilic normoblast or metamyelocyte

E. reticulocyte or band/stab form

E. is correct.

In a septicemia so many mature PMNs are being lost in the battle with the bugsthat the bone marrow allows some immature band/stab forms to circulate or to "gointo battle". The normal percent of band/stabs or reticulocytes is 1%.

A. is correct.

Page 230: Review Questions for Human Histology

The younger or less differentiated cells in each line, erythroid or granulocyte, havethe most delicate or vesicular chromatin pattern. Plus, these cells have veryprominent nucleoli. Daughters of these cells would have nucleoli, but theirchromatin patterns would not be as delicate or vesicular or "lacy".

Page 231: Review Questions for Human Histology

SECTION 10: DEFENSE SYSTEM

10.001 As you look at a lymphocyte in a Wright's stained smear of peripheral bloodyou realize that the cell you are looking at could be any one of the followingEXCEPT:

A. natural killer cell

B. T virgin cell

C. T effector cell

D. B virgin cell

E. B effector cell

10.002 T lymphocytes are found in certain tissues and organs in the body such asthe lymph node and even in peripheral blood. Where do the T lymphocytes thatpopulate these areas arise from?

A. tonsil

B. tongue

C. tela subcutanea

D. thymus

E. tunica propria

10.003 A cytotoxic cell physically contacts a target cell an initiates destruction ofthat foreign cell. Identify the cytotoxic cell.

A. B lymphocyte

B. plasma cell

C. T lymphocyte

D. plasmablast

E. monocytoblast

10.004 During antigen recognition by a B lymphocyte and its subsequentcytodifferentiation into a mature plasma cell, what other interactions are necessary

Page 232: Review Questions for Human Histology

before antibody to the insulting antigen can be detected?

A. nothing else is necessary

B. the B cell must contact another B cell

C. the B cell must receive "help" from a subset of T cells.

D. the B cell must contact antibody to the foreign cell

E. the B cell must be stimulated by plasminogen activators released from vesselendothelium

E. is correct.

The B effector cell or the cell of the B lineage which actually functions as the mostdifferentiated cell in that lineage is the terminally differentiated, fixed postmitotic ornon-replicating plasma cell. The lymphocyte you were looking at in the smearcould belong to any of the other types listed.

D. is correct.

All T lymphocytes which populate lymphoid and other tissues in the body arise inthe thymus and that is where the "T" comes from.

C. is correct.

The cell-mediated immune response is the hallmark of the T cell system. This isthe primary mechanism by which the immune system attacks viruses and foreigncells other than bacteria. The humoral response is the primary defense againstbacteria. In this response antibodies are made by B cells which have undergoneantigen recognition and differentiated into plasma cells.

C. is correct.

Most successful B cell activity requires not only antigen recognition but the help ofa subset of T lymphocytes termed T helper cells. T helper cells releasesubstances, called lymphokines, which promote B cells to proliferate(clonalexpansion) and differentiate into plasma cells.

10.005 Once clonal expansion of a specific type of B lymphocytes is under waywith the help of T helper cells, the activated B lymphocytes become ?

A. plasma cells

B. memory B cells

Page 233: Review Questions for Human Histology

C. virgin B cells

D. A & B are correct

E. A & B & C are correct

10.006 Which of the following represents an allograft situation with the host being ahuman?

A. human cadaver kidney transplant

B. baboon heart transplant

C. liver transplant between identical twins

D. skin transplant from abdomen to back on same body

10.007 Which of the following cells is most probably not a parenchymal element ofthe defense system?

A. T lymphocyte

B. B lymphocyte

C. dendritic reticular cell

D. plasma cell

E. fibroblast

10.008 When a T cell or a B cell is activated all of the following cytologicalchanges occur EXCEPT:

A. DNA synthesis

B. increased heterochromatinization of the nucleus C. enhanced RNA synthesis

D. cell division

E. increase in cell size

10.009 Arrange the following in proper order for lymph flow into and out of a lymphnode: 1=efferent lymphatic, 2=subcapsular sinus, 3=afferent lymphatic,4=paratrabecular sinus, 5=medullary sinus.

A. 5-4-3-2-1

Page 234: Review Questions for Human Histology

B. 3-2-4-5-1

C. 1-5-4-2-3

D. 2-3-5-1-4

E. 4-2-5-3-1

D. is correct.

Out of the clonal expansion of the activated B lymphocyte(s) two different types ofcells differentiate. One is our friend the plasma cell which is busy synthesizing andreleasing one type of specific antibody to the original antigenic insult (monoclonalantibody). The other cell produced from this clonal expansion is a memory B cell.Memory B cells can be activated by a second exposure to the original antigenicinsult much faster than virgin B cells. Such a second set reaction will also producea group of plasma cells and memory B cells identical to those produced in the "firstset" reaction.

A. is correct.

Allografts are tissue/organ transplants between members of the same species aslong as they are not identical twins. Isografting occurs when the transplantation isbetween genetically identical individuals. Xenografting is when speciesboundaries are crossed. Autografts occur from self to self.

E. is correct.

The fibroblasts found in the capsules and connective tissue septa in lymphoidorgans are the least likely candidate for lymphoid parenchyma, whereas, all otherchoices are functional components of the defense system.

B. is correct.

Instead of heterochromatinization the nucleus becomes more euchromatinizedwhich is morphological evidence for the enhanced RNA synthesis. All of the othercharacteristics listed do occur as an immunologically competent cell undergoesblast transformation and clonal expansion.

B. is correct.

This is the path that lymph takes as it flows through the lymph node and is "filtered"by phagocytic cells lining and bridging the various sinuses.

10.010 CD4 antigens are a group or cluster of differentiation antigens found on thesurface of which of the following cells?

Page 235: Review Questions for Human Histology

A. T killer/cytotoxic

B. T suppressor

C. T delayed type hypersensitivity

D. T helper

E. B memory

10.011 Which of the following cell types manufactures and releases interleukins?

A. T killer/cytotoxic

B. T suppressor

C. T delayed type hypersensitivity

D. T helper

E. B memory

10.012 Which of the following cell types manufactures and releases thelymphokines MIF (migration inhibitory factor) and MAF (macrophage activationfactor)?

A. T killer/cytotoxic

B. T suppressor

C. T delayed type hypersensitivity

D. T helper

E. B memory

10.013 T and B lymphocytes occupy different areas or regions in different lymphoidorgans. Where, in general, are the B cells primarily located?

A. mid to deep cortex of lymph nodes

B. periarterial sheath of splenic artery of white pulp

C. internodal areas in Peyer's patches

D. thymus

Page 236: Review Questions for Human Histology

E. germinal centers of lymph nodules

10.014 Which of the following immunoglobulins is found in tears, saliva, andmucus of the respiratory tract?

A. IgG

B. IgA

C. IgE

D. IgM

E. IgD

10.015 Which of the following cell types use a unique internal cell signaling systemwhich employs calcineurin?

A. T lymphocyte

B. B lymphocyte

D. monocyte/macrophage

D. dendritic reticular cell in lymph nodule

E. epithelial reticular cell in thymus

D. is correct.

CD4 antigens in general are found on the T helper cell whereas CD8 antigens arefound on the T suppressor cell.

D. is correct.

The CD4 or Th lymphocyte is the conductor of the immune symphony. A majorportion of its successful and coordinated "conducting" is mediated through a groupof substances called the interleukins. There are at least 10 interleukins (IL-1, IL-2,etc.).

C. is correct.

This subset of T cells release MIF and MAF. MIF and MAF serve to activate nearbymacrophages and prevent macrophages from migrating or exiting from the areawhere a delayed type of hypersensitivity reaction is underway.

E. is correct.

Page 237: Review Questions for Human Histology

All other choices are locations for T cells. In general, all germinal centers of alllymph nodules are primarily composed of B cells. In fact, in the germinal center iswhere B cell blast transformation, clonal expansion and differentiation into plasmacells is occurring.

B. is correct.

IgG is the major immunoglobulin comprising about 75% of the serumimmunoglobulins, IgM and IgD serve as the receptors on the surface of B cells andIgE is found on the surface of mast cells and basophils.

A. is correct.

The T lymphocyte and some cells of the kidney use an internal signaling systemwhich involves calcineurin. This has been called the "Achilles heel" of the T cellbecause cyclosporin, the immunosuppressive drug used to thwart immunerejection of allografts, works by blocking the activation of calcineurin. This alsoexplains why cyclosporin is toxic to the kidney, but not to other cell types.

10.016 Which of the following lymphoid organs is characterized by a very high rateof programmed cell death or apoptosis?

A. spleen

B. thymus

C. tonsil

D. lymph node

E. diffuse lymphatic tissue

10.017 Which of the following cell types has the job of surveillance for, detection ofand the removal of transformed or neoplastic cells which may be formed daily?

A. T cell

B. B cell

C. dendritic reticular cell

D. epithelial reticular cell

E. Null lymphocyte

10.018 The following description best fits which organ: presence of lymph nodules

Page 238: Review Questions for Human Histology

with germinal centers, surfaced by an epithelium with many intraepitheliallymphocytes, epithelial surface forms groves into underlying lymphatic tissue?

A. thymus

B. appendix

C. lymph node

D. tonsil

E. spleen

10.019 As a pin passes from blood into the parenchyma of the thymus through theblood-thymus barrier arrange the following structures in proper order after RBC:1=thymocyte, 2 =endothelial cell, 3=epithelial reticular cell, 4=pericyte in a space,5=basal lamina of endothelium, and 6=basal lamina of epithelial reticular cell.

A. 4-5-2-3-6-1

B. 6-3-4-5-2-1

C. 2-5-3-6-4-1

D. 5-213-3-6-1

E. 2-5-4-6-3-1

10.020 Where do you think a worn-out, 142 day old RBC will meet its phagocyticdeath?

A. thymus

B. spleen

C. Peyer's patch

D. lymph node

E. tonsil

B. is correct.

As thymocytes are being individually programmed during their differentiation in thefetal thymus some of them become programmed against normal selfantigens.When contacted or exposed to self-antigens these thymocytes die in largenumbers, effectively ablating or inactivating clones of cells which are designed to

Page 239: Review Questions for Human Histology

destroy self-antigens. The result is passively acquired immunological tolerance toone's own antigen.

A. is correct.

One of the functions of the T cell system is immune surveillance for tumor cells.Evidence in support of this concept is derived from the fact that allografted patientson immunosuppressive therapy have an enhanced incidence of cancer whencompared to the normal population.

D. is correct.

This is a description of a tonsil. In fact so many lymphocytes may be migratingthrough the epithelial surface that it is difficult to identify the epithelium. Many ofthese lymphocytes pass through the epithelium and become components of thesaliva where they can be seen on a smear of saliva and have been called "salivarycorpuscles".

E. is correct.

Antigens in the blood entering the thymus during the fetal period to causeapoptosis or programmed cell death of T cells arising to self antigen cross thebarrier in this anatomical sequence.

B. is correct.

The spleen is a lymphoid organ placed in the blood circulation whereas a lymphnode is a lymphoid organ placed in the lymph circulation. Lymph nodes filterlymph, the spleen filters blood and thereby removes foreign materials from theblood as well as worn-out, damaged, defective self materials.

10.021 What is the name for the region of red pulp of the spleen which is foundbetween two adjacent venous sinusoids?

A. paratrabecular sinus

B. Malphigian corpuscle

C. cord of Billroth

D. Hassall's corpuscle

E. periarterial cuff of lymphocytes

10.022 If a child is born into and kept in a germ free environment (boy in thebubble), which of the following structures would be missing?

Page 240: Review Questions for Human Histology

A. cortex of thymus

B. cortex of lymph node

C. medulla of thymus

D. germinal centers in lymph nodules

E. periarterial collar of lymphocytes in spleen

10.023 If a child is born with DiGeorgi's syndrome or agenesis of the thymus, whatwill characterize this situation?

A. very low numbers to no T lymphocytes

B. depressed function of B cells

C. poor to absent allograft rejection capabilities

D. A & B are correct

E. A & B & C are correct

10.024 In which of the following would you expect to find T helper and Tsuppressor interaction with B cells?

A. thymic medulla

B. germinal center

C. cord of Billroth

D. marginal zone

E. subcapsular sinus

10.025 Arrange the following structures in correct sequence for blood entering andthen exiting the spleen using the "open" circulatory route: 1=venous sinusoidlumen, 2=venous sinusoid wall, 3=cord of Billroth, 4=trabecular artery, 5=penicillarartery, 6=artery ensheathed with T lymphocytes.

A. 1-2-34-5-6

B. 6-5-4-1-2-3

C. 4-6-5-3-2-1

Page 241: Review Questions for Human Histology

D. 6-5-4-3-2-1

E. none of the above are correct

C. is correct.

The cords of Billroth are the areas of the red pulp of the spleen which have venoussinusoids at their peripheral edges. Capillaries terminate into the cords of Billroth,dumping blood into the cords where it passes around phagocytic cells beforeregaining access to the circulation by passing through the walls of the venoussinusoids. These walls have slit-shaped gaps between the lateral margins of theendothelial cells. Blood plasma and its formed elements which have not beenphagocytized by the macrophages located in a cord of Billroth pass through thegaps between the sinusoidal endothelial cells to gain access to the circulatorysystem again.

D. is correct.

Germinal centers are the areas where B cells have responded to an antigen, theantigen they were individually programmed to react to, and are undergoing clonalexpansion and differentiation into plasma cells and memory B cells. If there is noexposure to any foreign antigen then no germinal centers would form.

E. is correct.

All choices are functions of the T cell system and if there is no thymus there wouldbe basically no functional T cell system. Since most B cells require T cellinteraction for full-fledged normal production, B cell function would not becompletely normal.

D. is correct.

The marginal zone in the spleen is a term used for the broad region of interfacebetween white pulp and red pulp. This is where B cells produced in the lymphnodules of the spleen meet T cells arriving in the blood, exiting the blood vesseland moving through the T cell dependent areas toward B cell dependent areas.

C. is correct.

10.026 Which of the following events happens during the development of Blymphocytes into cells capable of undergoing clonal selection and expansion?

A. IgM is synthesized and inserted into the plasmalemma as a receptor typemolecule

B. IgD is synthesized and inserted into the plasmalemma as a receptor type

Page 242: Review Questions for Human Histology

molecule

C. IgM and IgD are synthesized and released to circulate as Immoral antibodies

D. A & B are correct

E. A & B & C are correct

10.027 Cells engaged in the immune response recognize each other and cells ofthe same body or a different body by what mechanism?

A. differences in DNA coding

B. differences in RNA coding

C. differences in nuclear/cytoplasmic ratio

D. differences in surface proteins

E. differences in surface lipids

10.028 Which class of genes code for MHC molecules found on lymphocytes ofdifferent kinds, but not on all body cells of an individual?

A. class I

B. class II

C. class III

D. class IV

E. class V

10.029 Which of the following events occur when a cell capable of antigenpresentation processes a foreign antigen for presentation to an immunologicallycompetent cell?

A. foreign proteins and the surface receptor "trapping" those proteins enter thecytoplasm by endocytosis

B. within the endocytotic vesicles the foreign antigens are processed into smallerfragments and newly synthesized class II MHC proteins are added to the vesicles

C. the fragments of foreign antigen which bind to the class II MHC proteins arebrought to the surface for display when the vesicles fuse with the plasmalemma

Page 243: Review Questions for Human Histology

D. A & B are correct

E. A & B & C are correct

D. is correct.

Developing B cells use IgM and IgD antibodies to establish their surface receptors.If they never come into contact with an antigen that can combine with either the IgMor IgD surface recognition molecules the B cells will remain inactive, i.e. they willnever undergo clonal selection, expansion or differentiation into plasma cells.

D. is correct.

The major group of proteins which cells carry on their surface which are used asimmunological recognition molecules are a group of proteins encoded in a group ofgenes known as the major histocompatibility complex (MHC).

B. is correct.

Major histocompatibility complex (MHC) proteins are coded for by classes ofgenes, i.e. class I genes code for class I MHC proteins which are found on allnonimmune system cells in an individual; class II genes code for class II MHCproteins which are found only on those cells involved in the immune reaction; classIII genes code for the proteins of the complement system.

E. is correct.

Choices A, B & C are the events in proper sequence which happen when antigenprocessing-presenting cells function normally.

10.030 Most T cells developing in the thymus are destroyed. What causes thisdestruction?

A. recognition and binding by the T cell of a selfprotein fragment attached to aclass I MHC protein

B. recognition and binding by the T cell of a selfprotein fragment attached to aclass II MHC protein

C. attack by competent B cells

D. A & B are correct

E. A & B & C are correct

10.031 All of the following lymphoid organs normally contain lymph follicles

Page 244: Review Questions for Human Histology

(nodules) EXCEPT:

A. lymph node

B. thymus

C. tonsil

D. spleen

E. Peyer's patches

10.032 All of the following are autoimmune diseases EXCEPT:

A. diGeorge's syndrome

B. multiple sclerosis

C. rheumatoid arthritis

D. lupus erythematosus

E. insulin-dependent juvenile onset diabetes

10.033 Which of the following lymphoid organs has connective tissue septainvading the organ from the capsule and thereby dividing it up into recognizablelobules of parenchyma?

A. thymus

B. tonsil

C. lymph node

D. spleen

10.034 Which of the following is/are "born" in the thymus after birth?

A. killer T cell

B. helper T cell

C. both of the above

D. none of the above

10.035 Phagocytosis, processing and presentation of a foreign antigen, is a

Page 245: Review Questions for Human Histology

function of which of the following?

A. killer T cell

B. helper T cell

C. both of the above

D. none of the above

D. is correct.

The developing T cells which do not bind self-protein fragments attached to class IMHC proteins are preserved, they will lose their CD4 receptor and become killer Tcells. The developing T cells which do not bind self-protein fragments attached toclass II MHC proteins are preserved, they will lose their CD8 receptors andbecome helper T cells.

B. is correct.

Although the thymus has a light area, the medulla, surrounded by darker stainingcortex, there are no germinal centers in the thymus.

A. is correct.

diGeorge's syndrome or thymic agenesis is not an autoimmune disease.

A. is correct.

Although the spleen and the lymph node do have connective septa arising from thecapsule and forming trabeculae, only the thymus has a lobulated appearance.

C. is correct.

T cells are born in the thymus. They leave the thymus and populate the T celldependent areas of the body wherever these areas are located. For example, themid-deep cortex of lymph nodes, the periarterial collar of lymphocytes around theartery of the white pulp in the spleen, etc.

D. is correct.

This is the function of any number of macrophages or dendritic reticular cells, not Tkiller or T helper cells.

10.036 Which of the following differentiate(s) into a cell with a prominent Golgibody and lots of RER?

Page 246: Review Questions for Human Histology

A. killer T cell

B. helper T cell

C. both of the above

D. none of the above

10.037 Perform is released, causing holes to form in the plasmalemma of a targetcell, after activation of which of the following:

A. killer T cell

B. helper T cell

C. both of the above

D. none of the above

10.038 Interleukins are released after activation of this cell type:

A. killer T cell

B. helper T cell

C. both of the above

D. none of the above

10.039 Which of the following is/are "born" in the bone marrow of the adult?

A. killer T cell

B. helper T cell

C. both of the above

D. none of the above

10.040 Which of the following must contact another cell to complete its effectorfunction?

A. killer T cell

B. helper T cell

C. both of the above

Page 247: Review Questions for Human Histology

D. none of the above

D. is correct.

This a description of what happens when a B lymphocyte, either a virgin or amemory cell, is exposed to the antigen it is preprogrammed against. It differentiatesinto a plasma cell.

A. is correct.

This is how a T killer cells destroys its target. It is something like a specific searchand find-nonspecific destroy the target mission.

B. is correct.

The T helper cell is the conductor of the immune concert. T helper cells elaboratean ever increasing number of lymphokines such as interlukins 1-10 and increasing.The various interleukins have profound effects on other types of T cells as well ason B cells. For example, T helper cells "help" B cells make and release antibody.

D. is correct.

T cells are "born" in the thymus from the time the thymus is differentiated as anorgan in the embryo/fetus until after puberty and into the early 20's. However, asthe thymus differentiates in the embryo/fetus it receives lymphocyte precursor cellsfrom the bone marrow. The immune cell type which is born in the bone marrow inthe adult is the B lymphocyte.

A. is correct.

This is a hallmark of the killer cells of the T cell system. They physically search fortheir specific target cell, physically contact the target and elaborate perforin whichis a cytotoxin which "bores" holes in the plasmalemma of the target cell therebykilling the target cell. T helper cells can contact other immune cells, but they alsocan and do communicate with other immune cells by the release of lymphokines.

Page 248: Review Questions for Human Histology

SECTION 11: CARDIOVASCULAR SYSTEM

11.001 Arrange the following structures or layers in the proper sequence for a pinpassing from the lumen of a muscular artery outward through its wall: 1=smoothmuscle, 2=internal elastic membrane, 3=simple squamous epithelium, 4=adiposetissue, 5=external elastic membrane.

A. 1-3-2-4-5

B. 3-2-4-5-1

C. 4-5-1-3-2

D. 3-2-1-5-4

E. 2-1-5-4-3

11.002 Which of the following would have the largest amount of elasticfibers/sheets in its wall?

A. inferior vena cava

B. pulmonary vein

C. thoracic duct

D. descending aorta

E. radial artery

11.003 Which of the following vessels contains a significant amount oflongitudinally arranged smooth muscle in its tunica adventitia?

A. aorta

B. inferior vena cava

C. superior vena cava

D. basilar artery

E. renal vein

11.004 When the number of circularly arranged smooth muscle cells in the tunicamedia of an arterial vessel is reduced to about 2-3, what is this vessel called?

Page 249: Review Questions for Human Histology

A. muscular artery

B. arteriole

C. small vein

D. metarteriole

E. continuous capillary

11.005 With respect to the capillaries of the brain, which of the following is acharacteristic feature separating these vessels from other vessels of similarcaliber?

A. a lumen of 8 micrometers

B. cell to cell junctions of the fascia occludens type

C. cell to cell junctions of the zonula occludens type

D. cell to cell junctions of the macula occludens type

E. an intact basal lamina outside the endothelium

D. is correct.

D. is convect.

The ascending aorta, its arch and the descending aorta and their immediatebranches are classified as elastic arteries because of the many elastic fibers orsheets in their walls. This high amount of elasticity provides for elastic recoil of thevessel wall after distention by the systolic ejection of blood into them. This keepsthe systemic blood pressure from falling to zero during diastole.

B. is correct.

The inferior vena cava and some of its immediate branches have circular smoothmuscle fibers in their tunics media, but since these veins are bringing bloodagainst gravity toward the heart they have a well developed layer of longitudinallyarranged smooth muscle fibers which officially are classified as being in thedeeper aspect of the tunica adventitia.

B. is correct.

Arterioles have from 2 to perhaps 5 layers of circular smooth muscle, and a smalllumen relative to the thickness of the wall. Metarterioles have one layer of smooth

Page 250: Review Questions for Human Histology

muscle cells and these are intermittent or "disappearing" as the vessel approachesits capillary morphology.

C. is correct.

Most endothelial cells outside of the central nervous system have fascial typeoccludens junctions. Since fascia occludens do not completely surround the entireperiphery of the cell, there are areas between adjacent endothelial cells whichpermit the passage of tissue fluid and perhaps white blood cells. The presence ofzonula occludens in the capillaries serving the CNS is what is known as the blood-brain barrier.

11.006 Which of the following organs contains capillaries of the fenestrated type?

A. kidney

B. liver

C. small intestine

D. A & B are correct

E. A & B & C are correct

11.007 What causes fluid to be extruded at the arterial end of a capillary bed?

A. higher hydrostatic pressure relative to osmotic p.

B. lower hydrostatic pressure relative to osmotic p.

C. hydrostatic and osmotic pressures are equal, contractility of vessel wallsqueezes fluid out

11.008 What causes at least some of the tissue fluid to be returned to the venousside of the capillary bed?

A. higher hydrostatic pressure relative to osmotic p.

B. lower hydrostatic pressure relative to osmotic p.

C. hydrostatic and osmotic pressures are equal, contractility of vessel wallsqueezes fluid in

11.009 Identify the cell which is embedded in the basal lamina of the endothelialcell of a capillary?

A. smooth muscle cell

Page 251: Review Questions for Human Histology

B. fibroblast

C. adipocyte

D. lymphocyte

E. undifferentiated mesenchymal cell or pericyte

11.010 The following description best describes which vessel: unusually widelumen relative to thinness of the wall, macrophages usually found adjacent to wall,wall is composed of endothelial cells and an intermittent basal lamina.

A. fenestrated capillary

B. continuous capillary

C. venule

D. metarteriole

E. sinusoid

E. is correct.

Continuous capillaries are capillaries which do not have little windows in them.Fenestrated capillaries have small windows or fenestrations in their cytoplasm.This makes this type of capillary very permeable. Sometimes fenestratedcapillaries are referred to as visceral capillaries whereas those supplying muscle,nerve, connective tissue and other somatic tissue are of the continuous type.

A. is correct.

Although the osmotic pressure or "draw" is high due to the multitude of largemolecules carried in the blood plasma, the hydrostatic pressure exceeds this at thearterial end and fluid is forced out of the capillary into the surrounding connectivetissue area where it is now called tissue fluid.

B. is correct.

On the venous side of a capillary bed the hydrostatic pressure has diminishedgreatly relative to what it was on the arterial side of the bed. The hydrostaticpressure on the venous side of a capillary bed is lower than the osmotic pressureand this causes much of the tissue fluid produced at the arterial end to be returnedto the blood at the venous end. The excess tissue fluid or the tissue fluid whichdoes not return to the blood at the venous end of the capillary bed, finds its way

Page 252: Review Questions for Human Histology

eventually into blind ended lymph capillaries where it is now called lymph.

E. is correct.

Pericytes are mesenchymal cells with long cytoplasmic processes which hang onto and partially surround capillary endothelium. Pericytes may be capable ofdifferentiating into smooth muscle cells, fibroblasts and even chondroblasts andosteoblasts. Since pericytes contain actin, myosin and tropomyosin they may becontractile.

E. is correct.

This is a description of a sinusoid. Some of the best places to find sinusoids are inthe liver and spleen.

11.011 Which cell type would have the best example of pinocytotic vesicles?

A. pericyte

B. endomysial fibroblast

C. capillary endothelium

D. endothelium of renal artery

E. macrophage associated with sinusoidal wall

A. sinusoids

B. perivascular space

C. arteriovenous anastomoses

D. perineural vessels

E. between small venules and medium venules

11.013 Which of the following vessels would contain opposing, semilunar folds ofintima?

A. axillary artery

B. superior vena cava

C. superficial veins in the extremities

Page 253: Review Questions for Human Histology

D. renal vein

E. splenic trabecular artery

11.014 The following description best applies to which vessel: endotheliumsurrounded by an incomplete basal lamina, no fenestrations, no pericytes,presence of anchoring filaments.

A. sinusoid

B. A-V anastomosis

C. thoracic duct

D. lymph capillary

E. right lymphatic duct

11.015 Capillary endothelial cells can be induced to migrate and proliferate in aspecific direction by the application of which of the following?

A. NGF

B. EGF

C. TAF

D. Factor VIII

E. Factor X

C. is correct.

The capillary is the exchange vessel or the site where a tremendous amount ofphysiological exchange occurs between blood and the tissues outside of thevessel wall. Some molecules move across by diffusion, such as oxygen, carbondioxide. Other larger molecules are moved across endothelial cytoplasm to andfrom both surfaces of the endothelial cell by pinocytosis.

C. is correct.

Capillary networks can be by-passed by direct connections between smallarteries/arterioles to small veins/venules.

C. is correct.

Valves in veins, especially those in the extremities are folds of intima with a core of

Page 254: Review Questions for Human Histology

subendothelial connective tissue. These valves prevent backward flow of blood.The great saphenous vein in the inferior extremity has such valves. Since the greatsaphenous vein is used as the bridge in coronary bypass surgery, it is important tokeep the proximaldistal orientation of the harvested vein in mind when performingsuch grafts.

D. is correct.

This is a description of a lymph capillary. Note that it differs quite a bit from a bloodcapillary especially in regard to the anchoring filaments which connect the lymphcapillary wall to surrounding larger connective tissue fibers.

C. is correct.

Nerve growth factor and epidermal growth factor have nothing to do withendothelium. Factors VIII and X are blood clotting factors. TAF or tumorangiogenesis factor stimulates endothelial cells to migrate/divide resulting inneoangiogenesis. Hyaline cartilage is avascular and it contains an anti-angiogenesis factor which apparently prevents angiogenesis in cartilage.

11.016 This description best fits what layer: presence of a large amount of fat cells,large blood vessels, lymphatics and nerve fibers; surfaced by a mesothelium.

A. endocardium

B. epicardium

C. myocardium

D. serous pericardium

E. parietal pericardium

11.017 Arrange the following structures in the correct sequence for impulseconduction in the heart: 1=AV node, 2=AV bundle, 3=SA node, 4=left or rightbundle branch.

A. 1-2-34

B. 4-3-2-1

C. 3-1-2-4

D. 3-2-1-4

E. 2-3-1-4

Page 255: Review Questions for Human Histology

11.018 The following description best describes which of the following: centrallylocated nucleus, perinuclear cytoplasm contains stored glycogen, peripherallyarranged myofibrils.

A. skeletal muscle myoblast B. cardiac muscle cell

C. subendocardial smooth muscle

D. Purkinje cell

E. Purkinje fiber

11.019 Where in the intercalated disc are the gap junctions located through whichcardiomyocytes communicate ionically?

A. transverse portion

B. lateral portion

11.020 What type of mitotic cell life characterizes the adult cardiac muscle cell?

A. vegetative intermitotic

B. differentiating intermitotic

C. reverting postmitotic

D. fixed postmitotic

B. is correct.

Although the serous pericardium is also surfaced by a medothelium and somedelicate submesothelial connective tissue, it does not contain large amounts of fat,obvious blood vessels such as the coronary arteries and veins. The endocardiumis surfaced by an endothelium and it has little or no fat in it.

C. is correct.

The impulse conducting system of the heart begins in the SA node, moves overatrial myocytes to the AV node, leaves the AV node in the AV bundle whichcrosses the pars membranaceum and then divides into the right and left bundlebranches or bundles of His.

E. is correct.

Purkinje cells are found in the cerebellum. Purkinje fibers are cardiomyocytesspecialized for impulse conduction.

Page 256: Review Questions for Human Histology

B. is correct.

In the transverse portion of the intercalated disc one finds myofibrillar junctions,desmosomes and maybe a few gap junctions. The longitudinal or lateral portion ofthe intercalated disc is loaded with gap junctions.

D. is correct.

The adult cardiac myocyte is a fixed postmitotic or non-replicating cell. When aportion of the heart loses its blood supply due to a thrombotic event in a branch of acoronary vessel, the cardiac myocytes distal to the thrombus die (myocardialinfarction). Since, the adjacent cardiac myocytes are incapable of cell divisionthere is no regeneration. The necrotic or dead tissue is eventually replaced byfibroblasts and collagen fibers which form a scar (if the person survives the heartattack).

A. Z band

B. A band

C. I band

D. A-I junction

E. A-H junction

11.022 Where are the membrane bound granules which contain natriuretic factorfound?

A. both poles of the nucleus in the Purkinje fiber

B. both poles of the nucleus in the atrial cardiomyocyte

C. lateral aspects of the intercalated disc

D. diffusely arranged in the endothelial cells lining the pulmonary capillaries

E. both poles of the nuclei of the juxtaglomerular apparatus

11.023 Wherein the layers of the heart are the Purkinje fibers located?

A. serous layer of parietal pericardium

B. fibrous layer of parietal pericardium

Page 257: Review Questions for Human Histology

C. myocardium

D. epicardium

E. endocardium

11.024 All of the following are composed of dense, almost regularly arrangedcollagenous connective tissue EXCEPT.

A. pars membranaceum of interventricular septum

B. parietal pericardium

C. visceral pericardium

D. central fibrous body of heart

E. cardiac valve rings

11.025 This region contains small cardiac muscle cells which do not haveintercalated discs or a prominent striated appearance; these cells are held togetherby desmosomes.

A. papillary muscle

B. sinoatrial node

C. trabeculae carneae

D. Purkinje fiber tract

E. membranous part of interventricular septum

11.026 Subendothelial fibroblasts are characteristic of

A. tunica intima

B. tunica media

C. tunica adventitia

A. is correct.

In human skeletal muscle T tubules are found at the A-I junction. In human cardiacmuscle the T tubules are found at the Z line.

B. is correct.

Page 258: Review Questions for Human Histology

Atrial natriuretic factor or atriopeptin is stored in membrane bound granules at eachpole of the elongated atrial myocyte. ANP is released in response to excessstretching of the myocyte which would happen if the blood volume is larger thannormal. ANP acts on the distal convoluted tubule (DCT) of the nephron stimulatingthe loss of sodium, potassium and water; thereby attempting to lower the bloodvolume.

E. is correct.

Purkinje fibers are located in the deeper aspects of the subendothelial connectivetissue of the endocardium or in that part of the endocardium which borders theinternal most aspect of the myocardium.

C. is correct.

All of the other structures are components of the cardiac skeleton. The visceralpericardium is a mesothelium under which there is some loose areolar connectivetissue with a significant amount of adipose tissue associated with it, especiallysurrounding the coronary vessels.

B. is correct.

In addition to the components of the description of the SA node mentioned, thisregion is embedded in dense irregular connective tissue with many nerve fiberspresent.

A. is correct.

11.0027 Adipose tissue is characteristic of

A. tunica intima

B. tunica media

C. tunica adventitia

11.0028 Smooth muscle is characteristic of

A. tunica intima

B. tunica media

C. tunics adventitia

11.0029 Vasa vasora are characteristic of

Page 259: Review Questions for Human Histology

A. tunics intima

B. tunica media

C. tunica adventitia

11.030 Simple squamous epithelium is characteristic of

A. tunica intima

B. tunica media

C. tunica adventitia

11.031 The epicardium of the heart is equivalent to what in a vessel wall:

A. tunica adventitia

B. tunica intima

C. tunica media

11.032 The myocardium of the heart is equivalent to what in a vessel wall:

A. tunica adventitia

B. tunica intima

C. tunica media

11.033 The endocardium of the heart is equivalent to what in a vessel wall:

A. tunics adventitia

B. tunica intima

C. tunica media

11.034 The pulmonary trunk is a/an:

A. elastic artery

B. muscular artery

C. arteriole

D. metarteriole

Page 260: Review Questions for Human Histology

E. capillary

11.035 A vessel with from 30 to 4-5 smooth muscle cells in the wall:

A. elastic artery

B. muscular artery

C. arteriole

D. metarteriole

E. capillary

C. is correct.

B. is correct.

C. is correct.

A. is correct.

A. is correct.

C. is correct.

B. is correct.

A. is correct.

B. is correct.

11.036 The radial artery is a/an:

A. elastic artery

B. muscular artery

C. arteriole

D. metarteriole

E. capillary

11.037 The arch of the aorta is a/an:

A. elastic artery

Page 261: Review Questions for Human Histology

B. muscular artery

C. arteriole

D. metarteriole

E. capillary

11.038 A vessel with a 5-8 micron lumen is a/an:

A. elastic artery

B. muscular artery

C. arteriole

D. metarteriole

E. capillary

11.039 A vessel with only I smooth muscle cell in tunica media is a/an:

A. elastic artery

B. muscular artery

C. arteriole

D. metarteriole

E. capillary

11.040 A vessel with a pericyte contained in its wall is a/an:

A. elastic artery

B. muscular artery

C. arteriole

D. metarteriole

E. capillary

11.041 Which of the following is an area of extensive pinocytosis?

A. elastic artery

Page 262: Review Questions for Human Histology

B. muscular artery

C. arteriole

D. metarteriole

E. capillary

11.042 Up to 70 fenestrated sheets of extracellular material:

A. elastic artery

B. muscular artery

C. arteriole

D. metarteriole

E. capillary

B. is correct.

A. is correct.

E. is correct.

D. is correct.

E. is correct.

E. is correct.

A. is correct.

11.043 Two prominent elastic lamina, one "internal" and the other "external":

A. elastic artery

B. muscular artery

C. arteriole

D. metarteriole

E. capillary

Page 263: Review Questions for Human Histology

11.044 Lined by endothelium with fenestrations:

A. elastic artery

B. muscular artery

C. arteriole

D. metarteriole

E. capillary

11.045 A vessel with 2-4 smooth muscle cells in media:

A. elastic artery

B. muscular artery

C. arteriole

D. metarteriole

E. capillary

11.046 Subendothelial fibroblasts are found in which of the following?

A. tunica intima

B. tunica media

C. tunics adventitia

11.047 Adipose tissue is found in which of the following?

A. tunics intima

B. tunica media

C. tunica adventitia

11.048 Smooth muscle is found in which of the following?

A. tunica intima

B. tunics media

C. tunica adventitia

Page 264: Review Questions for Human Histology

11.049 Vasa vasora are found in which of the following?

A. tunica intima

B. tunica media

C. tunica adventitia

B. is correct.

E. is correct.

C. is correct.

A. is correct.

This is one of the three sublayers of the tunica intima. The tunica intima consists offrom inside to outside: endothelium, some subendothelial fibroblasts and theinternal elastic membrane.

C. is correct.

Almost all of the fat cells in normal vessels can be found only in the tunicaadventitia.

B. is correct.

Most of the smooth muscle in the general plan of a vessel is confined to the tunicamedia. One major exception to this rule is the inferior vena cava which has somecircular smooth muscle in its Tunica media, but a large amount of longitudinallyarranged smooth muscle in its tunica adventitia.

C. is correct.

Vasa vasora are vessels for vessels. In the largest vessels the wall is so thick thatit can not be nourished from the blood in the lumen and therefore needs its ownblood supply. In general, the outer 50% of the wall of the very large vessels, botharteries and veins, receives a blood supply via vasa vasora. Vasa vasora can befound in the outer reaches of the tunica media, but they are most numerous andeasiest to see in the tunica adventitia.

11.050 Simple squamous epithelium is a component of which of the following?

A. tunics intima

B. tunica media

Page 265: Review Questions for Human Histology

C. tunica adventitia

11.051 The epicardium of the heart is equivalent to in a vessel wall.

A. tunica adventitia

B. tunica intima

C. tunica media

11.052 The myocardium of the heart is equivalent to in a vessel wall.

A. tunica adventitia

B. tunica intima

C. tunica media

11.053 The endocardium of the heart is equivalent to in a vessel wall.

A. tunica adventitia

B. tunica intima

C. tunica media

11.054 The pulmonary trunk is classified as which of the following?

A. elastic artery

B. muscular artery

C. arteriole

D. metarteriole

E. capillary

11.055 Which of the following has ranges from about 30 to 3 smooth muscle cellsin its tunica media and contains two major elastic membranes?

A. elastic artery

B. muscular artery

C. arteriole

Page 266: Review Questions for Human Histology

D. metarteriole

E. capillary

11.056 The radial artery is classified as which of the following?

A. elastic artery

B. muscular artery

C. arteriole

D. metarteriole

E. capillary

A. is correct.

Simple squamous epithelium in a vessel is known as endothelium and it is theinnermost layer or lining of the tunica intima.

A. is correct.

The epicardium of the heart is its outermost layer and it is equivalent to theoutermost layer of the generalized vessel wall, the tunica adventitia.

C. is correct.

The myocardium is the middle layer of the heart and it is equivalent to the middlelayer of the general plan of the vessel wall which is the tunica media.

B. is correct.

The innermost layer of the three layers of the heart is the same general layer as thetunica intima in the general plan of the three layers found in the generalized vesselwall.

A. is correct.

The pulmonary trunk is an example of an elastic or conducting artery.

B. is correct.

This is a description of a muscular or distributing artery. There are large musculararteries, which are most of the named arteries of gross anatomy in addition to thenamed elastic or conducting arteries (e.g. aorta). As these branch and branch theyget smaller and smaller and become too small to be named by gross anatomists. A

Page 267: Review Questions for Human Histology

small muscular artery by histological methods would have about 3-8 smoothmuscle cells in its tunica media.

B. is correct.

This named artery in the superior extremity is an example of a muscular ordistributing artery, not an elastic artery.

11.057 The arch of the aorta is classified as which of the following?

A. elastic artery

B. muscular artery

C. arteriole

D. metarteriole

E. capillary

11.058 Which of the following would have a 7-10 micron lumen?

A. elastic artery

B. muscular artery

C. arteriole

D. metarteriole

E. capillary

11.059 Its tunica media is composed of one, discontinuous layer of smoothmuscle?

A. elastic artery

B. muscular artery

C. arteriole

D. metarteriole

E. capillary

11.060 Which of the following would contain a pericyte in its wall?

Page 268: Review Questions for Human Histology

A. elastic artery

B. muscular artery

C. arteriole

D. metarteriole

E. capillary

11.061 In which of the following would one find areas of extensive pinocytosis?

A. elastic artery

B. muscular artery

C. arteriole

D. metarteriole

E. capillary

A. is correct.

The pulmonary trunk, the ascending aorta, the arch of the aorta, the proximal partsof the branches of the arch of the aorta and probably most of the descending aortaare examples of elastic or conducting arteries.

E. is correct.

Since this is only as big as a RBC or a little larger than a RBC (the RBC being the"histological ruler" at 7.2 microns in diameter), this small lumen best describes thecapillary.

D. is correct.

This is a description of the metarteriole or the last smooth muscle containing vesselon the arterial side of the capillary bed. The metarteriole has a single layer ofsmooth muscle cells in its tiny tunica media but these smooth muscle cells do notmake a continuous ring around the vessel wall.

E. is correct.

Pericytes represent undifferentiated, mesenchymaltype cells which may or may nothave cytoplasmic processes which wrap around the capillary. Pericytes "sit" in orare enclosed in their own basal lamina which is continuous with the basal laminaof the endothelial cell. Pericytes may be contractile. Since they are undifferentiated

Page 269: Review Questions for Human Histology

mesenchymal cells they are believed to be a cell source for other mesenchymalcells such as fibroblasts, fat cells, smooth muscle cells, etc.

E. is correct.

The endothelial cells of the capillaries show extensive pinocytotic activity, exceptfor the capillaries in the brain. The pinocytotic vesicles represent transendothelialtransport. In fenestrated endothelium, although pinocytotic vesicles are present,much material is transported transendothelially through the fenestrations. The lackof evidence for pinocytotic activity in the continuous capillaries of the brain is aspecial situation which is a contributing factor of the existence of a blood-brainbarrier. In addition to minimal pinocytotic activity here, the endothelial cells in thecapillaries of the brain are connected to each other by very tight zonula occludensjunctions.

11.062 In which of the following would one find up to 70 fenestrated sheets ofextracellular material?

A. elastic artery

B. muscular artery

C. arteriole

D. metarteriole

E. capillary

11.063 In which of the following would one find two prominent elastic lamina, one"internal" and the other "external"?

A. elastic artery

B. muscular artery

C. arteriole

D. metarteriole

E. capillary

11.064 This vessel is lined by endothelium with fenestrations:

A. elastic artery

B. muscular artery

Page 270: Review Questions for Human Histology

C. arteriole

D. metarteriole

E. capillary

11.065 1-2 smooth muscle cells in a continuous layer in the tunica media arecharacteristic of which of the following?

A. elastic artery

B. muscular artery

C. arteriole

D. metarteriole

E. capillary

A. is correct.

The main arteries exiting the heart (pulmonary trunk and aorta) are elastic orconducting arteries. They contain large numbers of fenestrated elastic sheets orlaminae between which there are smooth muscle cells. The elasticity of these greatvessels allows them to expand when the appropriate ventricle delivers blood intothem during systole. During cardiac diastole, the stretched elastic laminae of thegreat arteries recoil and thereby squeeze or put pressure on the blood in theirlumens. This would tend to force blood back into the heart but since the semilunarvalves are closed, blood is forced away from the heart and at the same time thehydrostatic pressure in the great arteries is maintained.

B. is correct.

The outermost layer of the tunica intima of muscular or distributing arteries is theinternal elastic membrane. The boundary between the outermost region of thetunica media and the tunica adventitia is where the less obvious external elasticmembrane is located.

E. is correct.

The endothelium of capillaries comes in two varieties: continuous and fenestrated.The fenestrated type is found in visceral organs whereas the continuous type ismore "somatic" in nature being found, for example, in skeletal muscle.

C. is correct.

Page 271: Review Questions for Human Histology

Arterioles have 1-2 continuous layers of smooth muscle cells in their tunica media.Further down the arterial tree from the smallest arteriole is the metarteriole whichhas a discontinuous single layer of smooth muscle cells. In general the wall of anarteriole is quite thick relative to the size of its lumen. Since the surface area of thearterioles is tremendous they play a significant role in maintaining, raising andlowering systemic blood pressure by their narrowing or widening, i.e., contractionor relaxation of their smooth muscle cells. The endothelial cells lining arteriolescontain a special feature and that is some cytoplasmic granules which containblood clotting factor VIII.

Page 272: Review Questions for Human Histology

SECTION 12: RESPIRATORY SYSTEM

12.001 Which of the following are not part of the conducting system of therespiratory tract?

A. trachea

B. terminal bronchioles

C. respiratory bronchioles

D. secondary bronchi

E. primary bronchi

12.002 The following description best fits which of the following cell types: bipolar,axon extends into a lamina propria, dendritic processes lie flat on the surface ofmicrovilli of a different cell type.

A. Clara cell

B. pneumocyte type I

C. pneumocyte type II (great alveolar cell)

D. cell in outer nuclear layer of the retina

E. olfactory receptor cell

12.003 Which epithelium is characteristic of most of the conducting passages ofthe respiratory system?

A. stratified squamous, non-keratinizing

B. stratified squamous, keratinizing

C. stratified columnar with cilia

D. pseudostratified columnar with cilia

E. pseudostratified cuboidal with cilia

12.004 Which of the following structures has stratified squamous epitheliumsomewhere on one surface and pseudostratified ciliated columnar epitheliumsomewhere on the other surface?

Page 273: Review Questions for Human Histology

A. hard palate

B. epiglottis

C. false vocal fold

D. A & B are correct

E. A & B & C are correct

C. is correct.

All other structures listed are components of the conducting or non-gaseousexchange portion of the respiratory system. Terminal bronchioles terminate into thefirst division of the respiratory or gaseous exchange division of the respiratorysystem: the respiratory bronchiole. The respiratory bronchiole is similar in structureto the conducting terminal bronchiole with the exception that it contains somealveoli in its wall.

E. is correct.

The olfactory neuron is bipolar with its dendritic bulb giving way to many olfactorycilia which are found lying flat along the surface of the olfactory epithelium over themicrovilli of the supporting or sustentacular cells.

D. is correct.

PSC ciliated with goblet cells is the epithelium found on most of the conductingpassages of the respiratory system. This epithelium is so characteristic of therespiratory system that it is often referred to as "respiratory epithelium".

D. is correct.

The false vocal fold is surfaced by respiratory epithelium. Both the hard palate andthe epiglottis have areas on their opposite sides which contain respiratoryepithelium and stratified squamous epithelium.

12.005 What can the daughter cells of the basal cells found in pseudostratifiedciliated columnar epithelium differentiate into?

A. basal cells

B. columnar cells with cilia

C. goblet cells

Page 274: Review Questions for Human Histology

D. A & B are correct

E. A & B & C are correct

12.006 The cilia of the respiratory epithelium all beat toward which structure?

A. external nares

B. oropharynx

C. alveolar duct

D. laryngeal ventricle

E. terminal bronchiole

12.007 The following description best fits which structure: covered on bothsurfaces mostly by stratified squamous non-keratinized epithelium, core of elasticfibers near its epithelium and skeletal muscle fibers deep to the elastic fibers.

A. uvula

B. epiglottis

C. false vocal fold

D. true vocal fold

E. soft palate

12.008 What tissue is found between or bridging the open ends of the trachealrings?

A. perichondrium

B. skeletal muscle

C. smooth muscle

D. loose areolar connective tissue

E. dense regular elastic connective tissue

12.009 The following description best fits which respiratory structure: absence ofhyaline cartilage, ciliated columnar to cuboidal epithelium, decreasing numbers ofgoblet cells, some smooth muscle.

Page 275: Review Questions for Human Histology

A. extrapulmonary bronchi

B. intrapulmonary bronchi

C. conducting bronchioles

D. respiratory bronchioles

E. alveolar ducts

E. is correct.

The small basal cells are incompletely differentiated cells which are mitoticallyactive. Their daughter cells 1) can remain in a relatively undifferentiated state asbasal cells, 2) differentiate into goblet cells and 3) differentiate into ciliatedcolumnar cells. This is the normal process of turnover in the respiratory epithelium.If you cough hard several times and then spit on a slide and stain the smear youwill see normally desquamated goblet cells and ciliated columnar cells from theconducting passages of your respiratory system, which must be continuouslyreplaced by daughter cells of the basal cells.

B. is correct.

The cilia in the nasal cavities beat toward the choanae. The cilia from the larynxand deeper into the respiratory system beat toward the oropharynx. This ciliarybeat constantly moves the mucus film on the surface of the respiratory epithelium tothe oropharynx where it is swallowed. The mucus film traps bacteria, inhaledforeign particles, etc. and the cilia move this conglomeration to the oropharynx foreventual swallowing.

D. is correct.

The true vocal fold is surfaced by mother nature's toughest wet epithelium,stratified squamous nonkeratinizing. The true cord has an elastic vocal ligamentunder the epithelium and deep to that the vocalis muscle.

C. is correct.

The smooth muscle occupying the space between the open ends of the trachealrings is called the trachealis muscle.

C. is correct.

Bronchi have plates or islands of hyaline cartilage in them. Respiratory bronchiolesand alveolar ducts have alveoli associated with them.

Page 276: Review Questions for Human Histology

12.010 Clara cells are found in which respiratory structure?

A. respiratory bronchioles

B. terminal bronchioles

C. secondary or intrapulmonary bronchi

D. A & B are correct

E. A & B & C are correct

12.011 Between what two adjacent structures in the respiratory tree is the alveolarduct placed?

A. terminal to respiratory bronchiole

B. alveolar sac to alveoli

C. alveolus from one pulmonary lobule to an adjacent pulmonary lobule

D. respiratory bronchiole to alveolar sac

E. alveolar sac to adjacent alveolar sac

12.012 Arrange the following structures in proper sequence for the passage ofoxygen from air in an alveolus to blood in an alveolar wall capillary: 1=bloodplasma, 2=basal lamina of endothelial cell, 3=surfactant, 4=endothelial cellcytoplasm, 5=basal lamina of pneumocyte type I, 6=cytoplasm of pneumocyte typeI.

A. 3-2-5-6-4-1

B. 3-2-6-4-5-1

C. 3-6-5-2-4-1

D. 3-5-6-4-2-1

E. 3-4-2-6-5-1

12.013 Lamellar bodies are characteristic of which of the following cells?

A. pneumocyte type I

B. pneumocyte type II

Page 277: Review Questions for Human Histology

C. alveolar macrophage

D. Clara cell

E. alveolar endothelium

D. is correct.

Clara cells first show up as one goes down the respiratory tree in the terminalbronchioles. They are taller than the ciliated cuboidal cells adjacent to them. Theydo not have cilia, but they do contain apical secretory granules which containglycosaminoglycans which may function as a protective lining for the terminal andrespiratory bronchioles. By the time Clara cells have appeared in the respiratorytree, goblet cells have disappeared but ciliated cells can be found deeper into thetree than goblet cells. This puts ciliary beat deeper into the tree than the productionof the mucus film which guarantees that all of the mucus film will creep toward theoropharynx.

D. is correct.

The alveolar duct connects the respiratory bronchiole with the lobbies (alveolarsacs) which have the individual rooms (alveoli) off of them.

C. is correct.

This is the ultrastructure of the air-blood interface. Carbon dioxide would bepassed in exactly the opposite direction from blood to air: 1-4-2-5-6-3.

B. is correct.

Lamellar bodies are electron-dense, layered or whorled looking structures rich inphospholipids, glycosaminoglycans and proteins. When the content of the lamellarbodies is released to the alveolar surface it forms a surface film which significantlyreduces the surface tension in the alveolus. The surface tension reducingsubstance is surfactant. Lamellar bodies do not appear in the great alveolarcells(pneumocyte type II) until the last several weeks of gestation. Prematureinfants do not have enough surfactant in their lungs to reduce the surface tensionfor normal "effortless" inspiration. This condition is called hyaline membranedisease.

12.014 In addition to the type I & II pneumocytes what other cell types are found inthe alveolar septum?

A. mast cells

B. fibroblasts

Page 278: Review Questions for Human Histology

C. macrophages

D. A & B are correct

E. A & B & C are correct

12.015 What structure can be found in the interlobular connective tissue septumwhich sweeps into the lung from the visceral pleura?

A. bronchiole

B. branch of pulmonary artery

C. branch of pulmonary vein

D. bronchus

E. mesothelial cells

12.016 The following description best fits which structure: mesothelial surface,under which lies a layer of fairly loose irregular connective tissue which gives wayto a very dense, almost regularly arranged collagenous connective tissue.

A. parietal pleura

B. visceral pleura

C. parietal pericardium

D. visceral pericardium

E. endocardium

12.017 Exocytosed lamellar bodies swell in the alveolar lining forming what?

A. alveolar macrophages

B. tubular myelin

C. blood lipids

D. mucus

E. serous secretion

12.018 Tritiated choline would be best suited for studying which process in thetype II pneumocyte?

Page 279: Review Questions for Human Histology

A. DNA synthesis

B. RNA synthesis

C. protein synthesis

D. lipid synthesis

E. carbohydrate synthesis

12.019 What percentage of surfactant consists of lipid?

A. 90

B. 70

C. 50

D. 30

E. 10

D. is correct.

All of these cell types are found in the interalveolar wall. In addition to residing inthe alveolar septum some of the macrophages wander over the surface of thepneumocyte I ingesting inhaled particles. These macrophages are often calleddust cells.

C. is correct.

In general the pulmonary arteries and their terminal branches carryingdeoxygenated blood coarse along side the bronchioles until branching into thealveolar capillary network. The oxygenated blood exiting the alveolar capillarynetwork is collected in pulmonary venules and veins which are found at theperiphery of each pulmonary lobule in the interlobular connective septum.

C. is correct.

The parietal pericardium is composed of two subdivisions. The innermostcomponent is the serous pericardium and it has a mesothelium on its surfacewhich lines the pericardial sac. Outside of the serous pericardium is a denseconnective tissue layer called the fibrous pericardium. The visceral and parietalpleura have only a mesothelial surface and some submesothelial connectivetissue.

Page 280: Review Questions for Human Histology

B. is correct.

The lamellar bodies secreted from the type II pneumocyte swell to form a latticestructure called tubular myelin. The surfactant film forms from this structure byinsertion of surfactant components into the surface film.

D. is correct.

This precursor was used to trace the morphological route of surfactant lipidsynthesis in the great alveolar cell.

A. is correct.

About 90 percent of surfactant consists of lipids. The main one isdipalmitoylphosphatidylcholine.

12.020 Pulmonary surfactant contains some proteins. These proteins are:

A. all hydrophobic

B. all hydrophilic

C. a mixture of hydrophobic and hydrophilic

12.021 All of the following cells are found in the olfactory mucosa EXCEPT:

A. basal cells which function as stem cells

B. ciliated columnar cells

C. sustentacular cells

D. lymphocytes

E. bipolar neurons

12.022 What kind of epithelium would you probably find lining the bronchi of a 70year old smoker (2 packs/day for 55 years)?

A. pseudostratified ciliated columnar

B. stratified squamous, non-keratinizing

C. olfactory

D. pure Clara cells

Page 281: Review Questions for Human Histology

E. stratified cuboidal

12.023 Which of the following is functionally important in the interalveolar wall.

A. collagen fiber

B. elastic fiber

C. reticular fiber

12.024 This organ has one side covered entirely by stratified squamous, non-keratinizing epithelium and its other side covered in one area by stratifiedsquamous, nonkeratinizing epithelium AND pseudostratified ciliated columnarepithelium.

A. hard palate

B. soft palate-uvula

C. true vocal fold

D. false vocal fold

E. epiglottis

12.025 This cell has cilia on its apical end. However, at the distal most end ofthese cilia the 9 doublet structure is absent but the two central fibers continue oninto a relatively long distal component of the cilium.

A. Clara cell

B. Type I pneumocyte

C. Type II pneumocyte

D. goblet cell

E. olfactory receptor cell

C. is correct.

The major surfactant proteins are four in number: two hydrophilic glycoproteins SP-A & SP-D and two hydrophobic proteins SP-B & SP-C. SP-A and SP-D act toenhance the phagocytosis of some viruses and bacteria by alveolar macrophages.

B. is correct.

Page 282: Review Questions for Human Histology

Unlike "regular" pseudostratified ciliated columnar epithelium or "respiratory"epithelium the olfactory epithelium does not contain ciliated cells. Lymphocytes arecommonly found migrating through epithelial surfaces, especially those of thedigestive and respiratory systems.

B. is correct.

After that much smoking the original "respiratory" epithelium most probably hadbeen replaced with a "tougher", stratified squamous, non-keratinizing epithelium.This change from one type to a different type of epithelium is termed metaplasia.Indeed the metaplastic, stratified squamous wet epithelium in this individualprobably showed some abnormal changes or dysplasia indicating its evolution intosquamous cell carcinoma.

B. is correct.

These fibers allow the expansion and contraction of the alveoli during respiration.

E. is correct.

The posterior side of the epiglottis is surfaced by stratified squamous, non-keratinizing epithelium in its superior half and by "respiratory" epithelium in itsinferior half.

E. is correct.

This special situation is characteristic for the dendritic process of the olfactoryreceptor cell.

12.026 Which of the following contains a core of elastic cartilage?

A. olfactory mucosa

B. epiglottis

C. larynx

D. trachea/main bronchi

E. conducting bronchiole

F. respiratory bronchiole

G. alveolus

12.027 Macrophages migrate over an epithelial surface in which of the following?

Page 283: Review Questions for Human Histology

A. olfactory mucosa

B. epiglottis

C. larynx

D. trachea/main bronchi

E. conducting bronchiole

F. respiratory bronchiole

G. alveolus

12.028 Which of the following has pores in its wall which can transfer bacteria fromone side to the other side of the wall?

A. olfactory mucosa

B. epiglottis

C. larynx

D. trachea/main bronchi

E. conducting bronchiole

F. respiratory bronchiole

G. alveolus

12.029 Which of the following contains hyaline cartilage in its wall?

A. olfactory mucosa

B. epiglottis

C. pinna of the ear

D. trachea/main bronchi

E. conducting bronchiole

F. respiratory bronchiole

G. alveolus

Page 284: Review Questions for Human Histology

12.030 Bowman's glands are present beneath an epithelium in which of thefollowing?

A. olfactory mucosa

B. epiglottis

C. larynx

D. trachea/main bronchi

E. conducting bronchiole

F. respiratory bronchiole

G. alveolus

B. is correct.

The only structure listed that contains elastic cartilage is the epiglottis. Since theepiglottis bends and closes the airway during swallowing it is important to thisfunction to have elastic, rather than hyaline, cartilage as the supporting specialconnective tissue in this organ.

G. is correct.

Alveolar macrophages or "dust" cells are unique in that they are tissuemacrophages which function by wandering over the alveolar surface ingesting allkinds of foreign objects. The alveolar macrophages in the smoker's lung are filledwith indigestible material such as carbon.

G. is correct.

Alveoli can communicate with each other because of holes or pores in their walls.These alveolar pores (of Kohn) can be seen on scanning electron micrographs.There can be several in each interalveolar wall. They may provide a mechanism toequalize air flow and pressure between adjacent alveoli. They aredisadvantageous because they also provide a pathway for microbes to move fromone alveolus to the next.

D. is correct.

There are supporting pieces of hyaline cartilage in the larynx, trachea, main stembronchi and in the secondary bronchi. Perhaps the main function of thesecartilages is structural support to keep the airway or conducting portions of therespiratory tree open.

Page 285: Review Questions for Human Histology

A. is correct.

In the lamina propria, under the olfactory epithelium, are some serous glands withducts which open onto the surface of the olfactory epithelium. Since these glandsof Bowman are serous they keep the surface of the epithelium wet and probablyhelp dissolve odoriferous substances for "easier" detection by the olfactory bipolarneurons.

12.031 Which of the following contains a ventricle covered or surfaced bypseudostratified ciliated columnar epithelium?

A. olfactory mucosa

B. epiglottis

C. larynx

D. trachea/main bronchi

E. conducting bronchiole

F. respiratory bronchiole

G. alveolus

12.032 C-shaped pieces of hyaline cartilage are found in which of the following?

A. olfactory mucosa

B. epiglottis

C. larynx

D. trachea/main bronchi

E. conducting bronchiole

F. respiratory bronchiole

G. alveolus

12.033 As one proceeds down the respiratory tree, which of the following is the firststructure in which gaseous exchange occurs?

A. olfactory mucosa

B. epiglottis

Page 286: Review Questions for Human Histology

C. larynx

D. trachea/main bronchi

E. conducting bronchiole

F. respiratory bronchiole

G. alveolus

12.034 Clara cells very numerous in which of the following?

A. olfactory mucosa

B. epiglottis

C. larynx

D. trachea/main bronchi

E. conducting bronchiole

F. respiratory bronchiole

G. alveolus

12.035 Surfactant is elaborated by a special cell type in which of the following?

A. olfactory mucosa

B. epiglottis

C. larynx

D. trachea/main bronchi

E. conducting bronchiole

F. respiratory bronchiole

G. alveolus

C. is correct.

The only ventricle in the respiratory system is the diverticulated space in the larynxwhich extends laterally and separates the true and false vocal folds from each

Page 287: Review Questions for Human Histology

other. The deeper reaches of the laryngeal ventricle are lined by pseudostratifiedciliated columnar epithelium.

D. is correct.

The trachea and the main stem bronchi have a continuous C-shaped piece ofhyaline cartilage in their walls. The open part of the C faces posteriorly and isspanned by smooth muscle. Secondary bronchi, or the many divisions of thebronchi from the main stem bronchi all the way to the first bronchioles, have chunksor plates of hyaline cartilage in their walls. These plates get smaller and smallerand finally disappear. The first conducting structures with no hyaline cartilage intheir wall are the bronchioles.

F. is correct.

All of the named structures of the respiratory tree, down to and including theterminal bronchioles, are conducting not respiratory structures. The respiratorybronchiole is the first member of the gaseous exchange or respiratory divisionbecause it has alveoli in its wall.

F. is correct.

Clara cells are non-ciliated cells which contain some secretory granules. Theysecrete glycosaminoglycans onto the bronchiolar surface and this probably hassomething to due with protection of the surface epithelium, which also containsciliated cuboidal cells. Clara cells are quite numerous in the terminalrespiratorybronchiole tree.

G. is correct.

The pneumocyte type II, or great alveolar cell, is the parenchymal cell in the lungwhich produces surfactant. The surfactant can be seen inside of the cell where it ispresent in large secretory vesicles called lamellar bodies.

12.036 The thyroarytenoid muscle is covered by a congregation of elastic fibers inwhich of the following?

A. olfactory mucosa

B. epiglottis

C. larynx

D. trachea/main bronchi

E. conducting bronchiole

Page 288: Review Questions for Human Histology

F. respiratory bronchiole

G. alveolus

12.037 The proximal processes of cells are functional axons in which of thefollowing?

A. olfactory mucosa

B. epiglottis

C. larynx

D. trachea/main bronchi

E. conducting bronchiole

F. respiratory bronchiole

G. alveolus

12.038 The following description: "a fold of tissue covered with stratified squamouswet epithelium, the core of the fold contains some elastic connective tissue anddeep to that is a mass of skeletal muscle" best describes which of the following?

A. olfactory mucosa

B. epiglottis

C. larynx

D. trachea/main bronchi

E. conducting bronchiole

F. respiratory bronchiole

G. alveolus

12.039 In which of the following do some cells have clubshaped expansions orvesicles with cilia on them?

A. olfactory mucosa

B. epiglottis

C. larynx

Page 289: Review Questions for Human Histology

D. trachea/main bronchi

E. conducting bronchiole

F. respiratory bronchiole

G. alveolus

C. is correct.

This is a partial description of the true vocal fold. What was omitted from thedescription is that this fold would have a covering of stratified squamous wet ornon-keratinized epithelium.

A. is correct.

This is a description of the olfactory bipolar neuron with a peripheral dendriticprocess and a central axonal process.

C. is correct.

This is a description of the true vocal fold in the larynx.

A. is correct.

In the olfactory epithelium there are support cells and bipolar neurons that havedendrites which have a bulb or vesicle which projects above the general surface ofthe epithelium. Several olfactory cilia take origin from this dendritic bulb and theyextend into the film of mucus on the surface of the epithelium. These ciliaryextensions are believed to be the receptors for odorous substances. On the axonalside of the olfactory bipolar neuron, an axonal process passes through the basallamina of the epithelium and enters the underlying lamina propria where it joinssimilar axons from other olfactory neurons to form groups of the same called filaolfactoria. These processes pass through the ethmoid bone and enter the olfactorybulb of the brain.

12.040 Which of the following folds or bends on itself during swallowing?

A. olfactory mucosa

B. epiglottis

C. larynx

D. trachea/main bronchi

Page 290: Review Questions for Human Histology

E. conducting bronchiole

F. respiratory bronchiole

G. alveolus

B. is correct.

The epiglottis actually bends or folds to close off the airway during swallowing. Itscore of elastic cartilage is quite appropriate for its structure/function role inswallowing.

Page 291: Review Questions for Human Histology

SECTION 13: GASTROINTESTINAL SYSTEM

13.001 As a pin passes from the lumen of the esophagus outward through its wall itwill encounter the following layers in which sequence: 1=submucosa, 2=laminapropria, 3=muscularis mucosa, 4=muscularis externa, 5=stratified squamous, non-keratinizing epithelium.

A. 5-2-3-111

B. 5-3-2-4-1

C. 5-3-2-4-1

D. 4-1-3-5-2

E. 4-2-3-1-5

13.002 Under 40X you are looking at some nerve cell bodies located between theouter longitudinal and inner circular layers of the muscularis externus in a sectionof the small intestine. Exactly what are these neurons?

A. parasympathetic preganglionic

B. parasympathetic postganglionic

C. sympathetic preganglionic

D. sympathetic postganglionic

E. non-autonomic motor

13.003 If you wanted to find a taste bud quickly, in which of the following would youchose to look?

A. filiform papillae

B. fungiform papillae

C. foliate papillae

D. circumvallate papillae

E. von-Ebner gland

13.004 In the diffuse lymphatic tissue in the lamina propria of the GI tract one can

Page 292: Review Questions for Human Histology

easily find plasma cells. In this region, which of the following types of plasma cellsis by far the most frequent?

A. IgG cells

B. IgA cells

C. IgM cells

D. IgE cells

E. IgD cells

A. is correct.

These layers in the wall of the esophagus represent the general plan of thehistology of the GI tract. In some organs this general plan is modified and manytimes the modification is diagnostic for organ identification. An example would bethe external longitudinal layer of smooth muscle in the colon which has threeenlarged regions, the teniae coli.

B. is correct.

In the myenteric plexus (of Auerbach) and in the submucosal plexus (of Meissner)the nerve cell bodies seen represent outlying ganglia for the parasympatheticdivision of the autonomic nervous system. Therefore they are postganglionicparasympathetic neurons. There is a childhood disease, aganglionosis, in whichthere is a failure of these neurons to differentiate resulting in an aganglionicsegment of the colon. The bowel proximal to the aganglionic segment does containparasympathetic postganglionic neurons and therefore, can participate inperistaltsis, but nothing can move through the aganglionic segment. Thus fecalmaterial builds up proximal to the aganglionic segment causing massive distentionof the normally innervated segment (megacolon).

D. is correct.

Although taste buds can be found scattered in the epithelium of the fungiformpapillae, they are much more numerous in the epithelium of the circumvallatepapillae.

B. is correct.

IgA releasing cells are much more numerous than all other Ig types. IgA coats thelining of the GI, urinary and respiratory tracts and can be thought of as a type of firstline of defense. IgA is quite resistant to proteolytic enzymes.

Page 293: Review Questions for Human Histology

13.005 In which of the following organs is skeletal muscle "diagnostically" found torun in three geometric planes?

A. oblique layer of muscularis externus of stomach

B. middle esophagus, not upper or lower esophagus

C. wall of the anal canal

D. pyloric sphincter

E. tongue

13.006 Which of the following contains the most calcium deposits?

A. dentin

B. enamel

C. cementum

D. calcified cartilage

E. mineralized bone matrix

13.007 Which of the following organs has a stratified squamous epithelium on bothof its surfaces and a core of skeletal muscle between the connective tissue areasunderlying the two epithelial surfaces?

A. recto-anal junction

B. upper anal canal-lower anal canal

C. lip

D. hard palate

E. false vocal fold

13.008 Which of the following organs has a stratified squamous epithelium on bothof its surfaces and a core of skeletal muscle between the connective tissue areasunderlying the two epithelial surfaces?

A. uvula

B. cheek

Page 294: Review Questions for Human Histology

C. soft palate

D. A & B are correct

E. A & B & C are correct

13.009 In the section of the gut tube you are looking at you notice that theoutermost area contains some loose areolar connective tissue with some adiposetissue in it and a simple squamous epithelial covering. You are looking at what?

A. the adventitia of the organ

B. the submucosa of the organ

C. the lamina propria of the organ

D. the serosa of the organ

E. the mucosa of the organ

E. is correct.

This arrangement of skeletal muscle bundles in 3 different planes is a specificcharacteristic of the tongue.

B. is correct.

Enamel is the hardest substance in your body and it contains about 95% calciumsalts. During the development of the tooth the enamel is secreted by ameloblastsand the dentin is secreted by odontoblasts.

C. is correct.

The lip has st. sq. dry epith. on one surface and st. sq. wet on the other surface andthe orbicularis oris as its core.

D. is correct.

The uvula has a core of skeletal muscle and both of its surfaces are covered withst. sq. wet type of epithelium. The cheek has a core of skeletal muscle and one ofits surfaces covered by st. sq. dry type and the other covered by st. sq. wet type ofepithelium. The soft palate has a core of skeletal muscle but only one of itssurfaces contains st. sq. wet type epithelium, the superior surface of the soft palateis covered with "respiratory" epithelium.

D. is correct.

Page 295: Review Questions for Human Histology

A fibrosa or adventitia would blend with the surrounding connective tissue. Aserosa is similar to a fibrosa except that it has a definitive epithelia surface ofmesothelial cells.

13.010 Where is the best place for someone to study the differences betweensmooth and skeletal muscle by having them intermingled with each other for easycomparison?

A. soft palate-uvula

B. gastro-duodenal junction

C. upper esophagus

D. middle esophagus

E. lower esophagus

13.011 In which region of the stomach do the gastric pits enter the mucosa about1/5th of the way toward the muscularis mucosa before branching into gastricglands?

A. cardiac portion

B. fundic portion

C. pyloric portion

13.012 All epithelia turn over. Some turn over faster than others. Relativelyundifferentiated cells which are vegetative intermitotics produce daughter cellswhich have the capacity to differentiate into the more mature cells in the epithelium.Where in the gastric pit-gland are the most undifferentiated cells located?

A. base

B. pit

C. neck

13.013 The muscularis externa of the intestinal tube usually is composed of aninner circular and an outer longitudinal layer. Where is this pattern modified intoinner oblique, middle circular and outer longitudinal layers?

A. middle esophagus

B. ileum

Page 296: Review Questions for Human Histology

C. stomach

D. caecum

E. appendix

13.014 In which organ can mucus glands be found in the submucosa?

A. duodenum

B. esophagus

C. stomach

D. A & B are correct

E. A & B & C are correct

D. is correct.

In the upper esophagus the skeletal muscle fibers of the inferior most pharyngealconstrictor make up the muscularis externa. In the lower esophagus the muscularisexterna has changed over to all smooth muscle which is the pattern until onereaches the skeletal muscle of the external anal sphincter. The middle esophagusis where the skeletal muscle from the upper esophagus blends with and "givesway" to the smooth muscle of the lower esophagus.

B. is correct.

At 4x or with an inverted ocular one can see the depth of the pits in relation to thearea of the glands in the wall of the stomach. In the cardiac and pyloric regions theratio of pit to gland is about 1:1. In the fundic region the ratio is about 1:5 or 1:4.

C. is correct.

The neck region contains the relatively undifferentiated cells, the daughter cells ofwhich move in two directions: up to the pit region where they become pit andsurface neck cells and deeper into the base or fundus of the gland where theydifferentiate into parietal, chief and enteroendocrine cells.

C. is correct.

Only in the stomach are there 3 layers to the muscularis externa and here the"extra" layer of obliquely arranged smooth muscle fibers is added inside of theusually circular layer. The oblique fibers aid in the churning action of the stomach.

D. is correct.

Page 297: Review Questions for Human Histology

The esophagus contains a few mucous glands located in its lamina propria(esophageal cardiac glands) and some more located in its submucosa(esophageal glands proper). The submucosal mucous glands (of Brunner) arediagnostic for the duodenum. The stomach has mucous glands in both its cardiacand pyloric regions but here these glands are located in the lamina propriabetween the muscularis mucosa and the surface epithelium, i.e. they are not foundoutward to the muscularis mucosa or in the submucosa.

13.015 Which structure first picks up chylomicrons for transport, indirectly, to theblood stream?

A. Peyer's patch

B. Burnner's glands

C. lacteals of intestinal villi

D. M or membranous epithelial cells

E. goblet cells

13.016 Where in the GI tract would you expect to find the highest concentration ofgoblet cells in the surface epithelium?

A. stomach

B. duodenum

C. ileum

D. appendix

E. rectum

13.017 Which of the following are involved in increasing the surface area forabsorption in the small intestine?

A. microvilli

B. villi

C. plicae circulares

D. A & B are correct

E. A & B & C are correct

Page 298: Review Questions for Human Histology

13.018 In the intestinal gland or crypt of Lieberkuhn which cell type readily takesup tritiated thymidine?

A. enterocyte

B. Paneth cell

C. goblet cell

D. crypt base columnar cell

E. membranous(M) epithelial cell

C. is correct.

Monoglycerides and free fatty acids absorbed by the enterocytes of the epitheliumof the intestinal villus are synthesized on the SER into lipid-containing vesicleswhich are released into the intercellular spaces at the lateral margins of theenterocytes. These fat globules are the chylomicrons and they enter blind endinglymph capillaries called lacteals located in the center of each intestinal villus. Fromtheir they eventually get collected into the thoracic duct. One way to see thisprocess is to view the abdominal viscera of a laboratory animal in vivo after it hasingested whole milk or cream. In this case the entire intestinal lymphatic tree willhave white limbs and branches.

E. is correct.

At the end of the digestive process about the only thing left to do is to absorb somemore water and, of course, lubricate the luminal contents with mucus fordefecation. The mucus from the goblet cells is the lubricant and the rectum has thehighest concentration of goblet cells of the regions listed.

E. is correct.

The plicae circulares (valves of Kerckring) are large folds of the mucosa andsubmucosa which can be seen with the naked eye. They increase the surface areaof the intestinal lumen. They have villi on them and the villi have absorptive cells orenterocytes which have many microvilli on their surface. In fact the microvilli are sonumerous that they give the cell surface a striated or brush border appearance.

D. is correct.

The crypt base columnar cell located at the base of the crypt of Lieberkuhn is therelatively undifferentiated stem cell from which daughter cells arise whichdifferentiate into all of the other cell types in the crypt. After birth of the daughter

Page 299: Review Questions for Human Histology

cells which will eventually differentiate into the more specialized cell types in thecrypt and on the surface of the villus, the differentiating enterocytes move up thesurface of villus as they differentiate eventually reaching the tip of the villus wherethey are sloughed off.

13.019 How many days does it take for the intestinal villus epithelium to renewitself relative to the processes of differentiation and migration of some of thedaughter cells produced by mitotic events in the crypt base columnar cells?

A. 1

B. 3

C. 5

D. 7

E. 9

13.020 What "forces" the digestive products to pass into the enterocytes instead ofbetween them?

A. terminal web

B. glycocalyx

C. chylomicrons

D. junctional complex

E. striated border

13.021 The following description best fits which cell type: extensive RER,prominent Golgi apparatus and granules which contain the antibacterial enzymelysozyme.

A. enterocyte

B. parietal cell

C. enteroendocrine cell

D. chief cell

E. Paneth cell

13.022 This cell has the capability to trap antigens and present them to adjacent

Page 300: Review Questions for Human Histology

lymphocytes.

A. membranous epithelial(M) cell

B. Paneth cell

C. parietal cell

D. enteroendocrine cell

E. goblet cell

13.023 Which of the following substances is not released by the intestinalenteroendocrine cell?

A. gastrin

B. secretin

C. cholecystokinin-pancreozymin

D. serotonin

E. trypsinogen

C. is correct.

It takes 5-6 days for a cell that is sloughed off from the tip of the villus to bereplaced by differentiation and migration of cells from the crypt base columnarcells.

D. is correct.

The small dots or terminal bars seen with the light microscope actually are thejunctional complexes located on the lateral surface of the enterocyte near its apex.The junctional complex from intestinal lumen outward consists of zonulaoccludens, zonula adherens and macula adherens. The zonula occludens is a beltlike structure which extends completely around the enterocyte. The ZO is anintimate fusion of integral membrane proteins of adjacent enterocytes in such away that in these regions the intercellular space is obliterated.

E. is correct.

The Paneth cell contains large eosinophilic granules which make it easy to identifyin the crypt of Lieberkuhn. It also is known to contain an enzyme, lysozyme, whichcan digest the bacterial cell wall.

Page 301: Review Questions for Human Histology

A. is correct.

The M cell is a specialized epithelial cell found in the epithelial lining of the smallintestine wherever one or more lymphoid follicles are located. M cells haveinvaginations forming pits and in these areas intraepithelial lymphocytes arelocated, presumably being "antigen-instructed" by the M cell.

E. is correct.

Trypsinogen is an enzyme released from the exocrine pancreas. All other choicesare hormones which can be released from various enteroendocrine cells.

13.024 What stimulus causes the release of secretin by the duodenalenteroendocrine cell?

A. acid chyme in lumen

B. gastrin

C. high pH solution in pancreatic duct

D. cholecystokinin-pancreozymin

E. lysozyme from Paneth cell

13.025 All of the following are found in the lamina propria core of a villus in thesmall intestine EXCEPT:

A. eosinophils

B. lymphocytes

C. endothelial cells

D. nerve cell bodies

E. smooth muscle cells

13.026 Congregations or collections of the longitudinal layer of the muscularisextema are characteristic features of all of the following EXCEPT:

A. cecum

B. appendix

C. colon

Page 302: Review Questions for Human Histology

D. rectum

13.027 The internal anal sphincter is composed of?

A. circular smooth muscle fibers

B. longitudinal smooth muscle fibers

C. circular skeletal muscle fibers

D. longitudinal skeletal muscle fibers

13.028 The pectinate line of gross anatomy, microscopically is which of thefollowing:

A. abrupt transition of st. sq. non-keratinizing to st. sq. keratinizing epithelium

B. abrupt transition of simple columnar epith. to st. sq. non-keratinizing epith.

C. abrupt transition of simple columnar epith. to st. sq. keratinizing epith.

D. abrupt transition from villi to a non-villus type of histology

E. abrupt transition from smooth muscle to skeletal muscle.

A. is correct.

The very low pH material, chyme, exiting the stomach stimulates a class ofenteroendocrine cells to release secretin which stimulates the release of a sodiumbicarbonate containing solution from the ductal epithelium of the pancreas.

D. is correct.

The nerve cell bodies in the intestine are located in the myenteric and submucosalplexuses. Endothelial cells are present in the villus as the lining for the blood andlymph capillaries. Lymphocytes and eosinophils are connective tissue cellsnormally found in a lamina propria. Smooth muscle cells are found in the villus.They are extensions from the muscularis mucosa and when they contract they areresponsible for movement of the villus. Villi move more when food is present.These movements help "unload" the lacteals toward larger lymph vessels.

B. is correct.

The teniae coli of the large intestine are not found in the appendix.

A. is correct.

Page 303: Review Questions for Human Histology

The external anal sphincter is composed of circular skeletal muscle fibers.

B. is correct.

When the simple columnar epithelium of mostly goblet cells stops and st. sq. non-keratinizing epithelium begins this is the junction between the upper and lowerportions of the anal canal.

13.029 What are the anal columns (of Morgagni)?

A. extensions of the taenia coli

B. masses of lymph nodules

C. mucosal folds

D. internal extensions of the internal anal sphincter

E. folds of epidermis and dermis extending into the anal canal from the exterior

13.030 Which of the following regions of the digestive tube would have the highestaccumulation of lymph nodules?

A. jejunum

B. esophageal-cardiac jct.

C. cecum

D. jct. between upper and lower anal canal

E. appendix

13.031 With respect to carcinoma of the colon arising in the simple columnarepithelium of an intestinal gland, which of the following histological locations of thecancer would have the highest 5 year surgical survival? Cancer cells localized orextending ?

A. to the muscularis externa

B. through the muscularis externa and adventitia/serosa to outside surface of organ

C. to the submucosa

D. in the lamina propria

E. through the lamina propria to the muscularis mucosa

Page 304: Review Questions for Human Histology

13.032 What structure is found at the center of the classical hepatic lobule?

A. bile ductule

B. branch of common hepatic artery

C. branch of portal vein

D. central vein

13.033 Which of the following structures is not found at the center of a portallobule?

A. lymph vessel

B. bile ductule

C. radicle of hepatic vein

D. radicle of portal vein

E. branch of hepatic artery

13.034 Which of the following zones of the hepatic acinus would contain anirrigating branch of the portal vein and an irrigating branch of the hepatic artery?

A. zone 1

B. zone 2

C. zone 3

C. is correct.

The mucosal folds or anal columns join with one another just below the pectinateline, creating pocketlike anal valves. Superior to each valve is a small recesscalled the anal sinus.

E. is correct.

The wall of the appendix contains many lymph nodules in the lamina propria andsubmucosal layers.

D. is correct.

A carcinoma of the colon arising in the epithelial lining would locally invade into

Page 305: Review Questions for Human Histology

the wall of the colon in a manner similar to the sequence encountered as onepassed a pin through the wall from epithelium outward. The best survival would beassociated with the least amount of invasion, i.e. a tumor confined to the epitheliumand not yet broken through the basement membrane would be the "best" kind tohave. Next worse would be involvement of the lamina propria. Next worse wouldbe extension to the muscularis mucosa, etc.

D. is correct.

The central vein is the central structure in the polygonal classical liver lobule.

C. is correct.

As the hepatic vein branches into the liver to receive the blood exiting the liver, itsterminal branches are the central veins of the classical lobule. The center of thetriangular shaped portal lobule contains all other contents listed. At each of the 3corners of the portal lobule one would find a central vein.

A. is correct.

The irrigating or distributing branches of both the portal vein and the hepatic arteryboth supply blood to the hepatic sinusoids, which eventually is collected by thecentral veins at two opposite sides of the diamond shaped acinus.

13.035 In which region of the hepatic acinus would the hepatocytes have the leastamount of oxygen, relatively?

A. zone 1

B. zone 2

C. zone 3

13.036 In the cords of hepatocytes next to a hepatic sinusoid, which way does thebile flow in the bile canaliculi?

A. from central v. toward portal triad

B. from portal triad toward central v.

C. from central v. toward an adjacent central v.

D. from portal triad toward an adjacent portal triad

E. from bile ductule toward bile canaliculi

13.037 In which organ is the terminal cell of the duct system invaginated into the

Page 306: Review Questions for Human Histology

center of the secretory acinus?

A. parotid gland

B. liver

C. submandibular gland

D. pancreas

E. lingual glands of von Ebner

13.038 Which substance causes the duct system of the pancreas to secrete itswatery secretion containing sodium bicarbonate?

A. bile

B. secretin

C. albumin

D. cholecystokinin-pancreozymin

E. glucagon

13.039 In a patient with congestive heart failure the heart can not pump as well asnormal and blood flow distal to the heart becomes sluggish, such as in thepulmonary capillaries and other capillary beds. In the liver this sluggish blood flowactually causes hepatocytes to die from hypoxia/anoxia. These hepatocytes arereplaced with scar tissue (fibroblasts and collagen fibers). In the liver this scarringis called cirrhosis. Where in the liver acinus would one expect to find the cirrhosisaccompanying heart failure ("cardiac cirrhosis")?

A. zone 1

B. zone 2

C. zone 3

C. is correct.

Zone 3 is farthest away, "down the sinusoids", from the irrigating branches of thehepatic artery and portal vein. Therefore, hepatocytes in this zone are sort of last inline for oxygen because hepatocytes in zone 1 and then zone 2 have utilized a lotof the oxygen available from the distributing branches of the hepatic artery.

A. is correct.

Page 307: Review Questions for Human Histology

Bile flow is opposite to blood flow in the general scheme of the liver lobule, be itclassical or portal, or the liver acinus.

D. is correct.

This is a characteristic feature of the pancreas in comparison to other glands withserous acini. These cells are called centro-acinar cells.

B. is correct.

In response to acid chyme entering the duodenum from the stomach someenteroendocrine cells in the duodenum secrete secretin which stimulates theductal epithelium in the pancreas to release a watery solution which has a high pH.CCK-PZ causes the release of the acinar cell secretory product contained in thezymogen granules. Thus such enzymes as trypsinogen, chymotrypsinogen,amylase, lipase, RNAse, DNAse, etc. are released.

C. is correct.

As blood flow through the sinusoids to the central veins to the hepatic v. to theinferior vena cava to the right heart is slowed due to the cardiac pathology, thealready relatively anoxic hepatocytes in zone 3 die and they are replaced with scartissue. This zone 3 cirrhosis is often termed "cardiac cirrhosis" or scarring in aspecific region of the acinus or classical hepatic lobule, i.e., around the centralveins.

13.040 Arrange the following structures/regions in proper sequence for a moleculein the blood which will be absorbed into the liver: 1=microvilli, 2=perisinusoidalspace of Disse, 3=fenestration in endothelium, 4=hepatocyte SER.

A. 1-2-3-4

B. 3-2-1-4

C. 2-1-3-4

D. 3-1-2-4

E. 2-3-1-4

13.041 Which of the following cell types in the liver is a phagocyte?

A. fat storage cell or lipocyte

B. hepatocyte

Page 308: Review Questions for Human Histology

C. sinusoidal endothelium

D. Kupfier cell

13.042 Which of the following substances represent the exocrine function of theliver?

A. albumin

B. glycogen

C. bilirubin

D. fibrinogen

E. prothrombin

13.043 Where in the liver would you expect phenobarbital to be detoxified?

A. SER of hepatocyte

B. SER Kupffer cell

C. SER of fenestrated endothelium

D. SER of fat storing cell

E. SER of cuboidal cell lining bile ductule

13.044 What type of mitotic cell life does the hepatocyte live?

A. vegetative intermitotic

B. differentiating intermitotic

C. reverting postmitotic

D. fixed postmitotic

B. is correct.

The endothelium lining the hepatic sinusoids is fenestrated allowing almost allelements of the blood, except the formed elements, easy access to the space ofDisse into which the hepatocytes stick their

microvilli.

Page 309: Review Questions for Human Histology

D. is correct.

Kupffer cells are derived from monocytes and are permanent residents of the liversinusoidal wall, sometimes partially bridging the sinusoidal lumen.

C. is correct.

Bilirubin is a component of bile. Bilirubin is a waste product of hemoglobin,produced when worn out RBCs are disposed of mainly by phagocytes in thespleen. Hepatocytes conjugate the bilirubin with glucuronic acid making it watersoluble. Bile, which facilitates the digestion of absorbed fats, is secreted by thehepatocyte into the bile canalicular system which eventually dumps into theterminal branches of the common hepatic duct.

A. is correct.

One of the major functions of the hepatocyte is the detoxification of toxins such assome of the anesthetic agents. This occurs in the SER. The hepatocyte SER also"detoxifies" naturally occurring substances such as estrogen, especially in themale. With chronic alcoholism hepatocytes die and are replaced with scar tissue(alcoholic cirrhosis). Increasing loss of hepatocytes to this process slowly allowsthe circulating level of estrogen (from the zona reticularis of the adrenal gland) torise. This rising level of "non-detoxified" estrogen can cause development of themale breast or gynecomastia.

C. is correct.

If up to 70% of the liver is surgically removed in the human, the remaining 30% willcontinue to support life and it will regenerate. Thus hepatocytes in an extended GOcan enter the cell cycle, replicate their DNA and undergo mitosis, therebyregenerating exactly what was surgically removed.

13.045 Which of the following is not found in the wall of the gall bladder?

A. lamina propria

B. muscularis mucosa

C. muscularis externa

D. adventitia

E. serosa

13.046 Which of the following are functions of the gall bladder?

Page 310: Review Questions for Human Histology

A. store bile

B. concentrate bile

C. manufacture bile

D. A & B are correct

E. A & B & C are correct

13.047 Which substance(s) can directly or indirectly cause the gall bladder torelease its contents?

A. CCK-PZ

B. presence of fats in chyme

C. low pH of chyme in duodenal lumen

D. A & B are correct

E. A & B & C are correct

13.048 Parotid gland is characterized by:

A. about 95 % serous acini

B. about 95% mucous acini

C. about 70% serous and 30% mucous acini

13.049 Submandibular gland is characterized by:

A. about 95 % serous acini

B. about 95% mucous acini

C. about 70% serous and 30% mucous acini

13.050 Sublingual gland is characterized by:

A. about 95 % serous acini

B. about 95% mucous acini

C. about 70% serous and 30% mucous acini

Page 311: Review Questions for Human Histology

13.051 Brunner's gland is characterized by:

A. about 95 % serous acini

B. about 95% mucous acini

C. about 70% serous and 30% mucous acini

13.052 Von Ebner's gland is characterized by:

A. about 95 % serous acini

B. about 95% mucous acini

C. about 70% serous and 30% mucous acini

B. is correct.

Both the muscularis mucosa and the submucosa of the general plan of the GI tractare absent from the wall of the gall bladder. Where the gall bladder is attached tothe liver it has an adventitia. Where the surface of the gall bladder is exposed to theperitoneal cavity it has a serosa.

D. is correct.

Bile is manufactured by the hepatocyte, it is stored and concentrated by the gallbladder. The removal of water from the bile involves a sodium pump which pumpssodium out of the bile and by doing so, water eventually follows the sodium out ofthe bile stored in the lumen of the gall bladder.

D. is correct.

Low pH of chyme would stimulate the secretion of secretin by certainenteroendocrine cells in the duodenum. Secretin stimulates the release of the highpH, watery solution from the pancreatic ductal cells. The presence of dietary fats inthe duodenum causes some enteroendocrine cells to release CCK-PZ whichcause the contraction of the muscularis externs in the wall of the gall bladder andalso the release of zymogen granules from the pancreatic acinar cells.

A. is correct.

C. is correct.

B. is correct.

B. is correct.

Page 312: Review Questions for Human Histology

A. is correct.

13.053 Renin is a product of

A. gastric surface cell

B. enteroendocrine (APUD) cell

C. chief or zymogenic cell

D. parietal cell

E. gastric pit epithelial cell

13.054 The parotid gland is composed of approximately:

A. 95 % serous acini

B. 95% mucous acini

C. 70% serous and 30% mucous acini

13.055 The submandibular gland is composed of approximately:

A. 95 % serous acini

B. 95% mucous acini

C. 70% serous and 30% mucous acini

13.056 The sublingual gland is composed of approximately:

A. 95 % serous acini

B. 95% mucous acini

C. 70% serous and 30% mucous acini

13.057 Brunner's glands are composed of approximately:

A. 95 % serous acini

B. 95% mucous acini

C. 70% serous and 30% mucous acini

13.058 A von Ebner's gland is composed of approximately:

Page 313: Review Questions for Human Histology

A. 95 % serous acini

B. 95% mucous acini

C. 70% serous and 30% mucous acini

13.059 The crown of the tooth is composed of which of the following?

A. dentine

B. enamel

C. cementum

13.060 The root of the tooth is composed of which of the following?

A. dentine

B. enamel

C. cementum

C. is correct.

A. is correct.

The parotid gland is essentially a pure serous gland. In many ways it looks like theexocrine pancreas, however, the presence of the islets of Langerhans in thepancreas is the easiest way to differentiate between these two organs.

C. is correct.

The submandibular gland is composed of approximately 1/3 mucous and 2/3serous acini.

B. is correct.

The sublingual salivary gland is essentially a pure mucous, exocrine gland.

B. is correct.

Brunner's submucosal mucous glands are characteristic of the duodenum. Theysecrete a viscous, alkaline fluid (pH=9.0) which helps protect the internal surface ofthe organ and lowers the pH of the acid chyme coming from the pyloric region ofthe stomach.

A. is correct.

Page 314: Review Questions for Human Histology

These glands are essentially composed of serous cells. Since their ducts open intothe trenches around the circumvallate papillae, they "rinse" out these trenches andkeep the taste buds here "clean".

B. is correct.

The crown is that part of the tooth projecting above the gingiva. The crown issurfaced by enamel.

C. is correct.

The root is that part of the tooth below the gingiva and it is covered, not by enamel,but by cementum.

13.061 The pulp of the tooth is composed of which of the following?

A. dentine

B. enamel

C. cementum

13.062 Which of the follow produces mucus?

A. gastric surface cell

B. enteroendocrine (APUD) cell

C. chief or zymogenic cell

D. parietal cell

13.063 Which of the following produces HCl?

A. gastric surface cell

B. enteroendocrine (APUD) cell

C. chief or zymogenic cell

D. parietal cell

E. gastric pit epithelial cell

13.064 Which of the following produces intrinsic factor?

Page 315: Review Questions for Human Histology

A. gastric surface cell

B. enteroendocrine (APUD) cell

C. chief or zymogenic cell

D. parietal cell

E. gastric pit epithelial cell

13.065 Which of the following produces histamine?

A. gastric surface cell

B. enteroendocrine (APUD) cell

C. chief or zymogenic cell

D. parietal cell

E. gastric pit epithelial cell

13.066 Which of the following produces gastrin?

A. gastric surface cell

B. enteroendocrine (APUD) cell

C. chief or zymogenic cell

D. parietal cell

E. gastric pit epithelial cell

13.067 Which of the following produces lipase?

A. gastric surface cell

B. enteroendocrine (APUD) cell

C. chief or zymogenic cell

D. parietal cell

E. gastric pit epithelial cell

A. is correct.

Page 316: Review Questions for Human Histology

Dentine is the layer outside the pulp cavity.

A. is correct.

Both the gastric surface cell and the gastric pit or gastric neck epithelial cellproduce mucus. The biochemical characteristic of the mucus produced by thepit/neck cells is different from that produced by the mucous surface cells. Thesetwo types of mucus function to 1) lubricate the gastric surface and 2) protect thesurface epithelium from trauma induced by the food and enzymatic digestion fromthe gastric juice.

D. is correct.

The gastric parietal cell releases HCI. The parietal cell contains an enzyme,carbonic anhydrase, which can act on carbon dioxide releasing carbonic acidwhich can dissociate into free hydrogen and bicarbonate. The free hydrogen canthen combine with chloride obtained from the blood/tissue fluid.

D. is correct.

In addition to releasing HCl the parietal cell also releases intrinsic factor whichbinds vitamin B12 and promotes its absorption by the ileum.

B. is correct.

Histamine is believed to be the product of one particular type of gastricenteroendocrine cell.

B. is correct.

Gastrin is a hormone which stimulates the secretion of gastric juice. The onlyendocrine type cell listed is the enteroendocrine or APUD cell. APUD refers toamine precursors uptake decarboxylase. Most authors use the easiernomenclature=enteroendocrine.

C. is correct.

One of the enzymes synthesized and released by the chief cell is a lipase. Morelipase will be produced by the acinar cells in the pancreas.

13.068 Which of the following produces serotonin?

A. gastric surface cell

B. enteroendocrine (APUD) cell

Page 317: Review Questions for Human Histology

C. chief or zymogenic cell

D. parietal cell

E. gastric pit epithelial cell

13.069 Which of the following produces pepsinogen?

A. gastric surface cell

B. enteroendocrine (APUD) cell

C. chief or zymogenic cell

D. parietal cell

E. gastric pit epithelial cell

13.070 Which of the following contains intracellular canaliculi?

A. surface mucous cell

B. mucous neck cell

C. enteroendocrine cell

D. chief or zymogenic cell

E. parietal cell

13.071 Which of the following would have granules in its apical cytoplasm?

A. surface mucous cell

B. mucous neck cell

C. enteroendocrine cell

D. chief or zymogenic cell

E. parietal cell

13.072 Which of the following would have granules in its basal cytoplasm?

A. surface mucous cell

B. mucous neck cell

Page 318: Review Questions for Human Histology

C. enteroendocrine cell

D. chief or zymogenic cell

E. parietal cell

13.073 Which of the following would have lots of RER?

A. surface mucous cell

B. mucous neck cell

C. enteroendocrine cell

D. chief or zymogenic cell

E. parietal cell

B. is correct.

Serotonin is a product of one type of enteroendocrine cell. It is a vasoconstrictorsubstance.

C. is correct.

Pepsinogen is the primary secretory product of the chief cell. Pepsinogen isinactive but when it finds itself in a acid environment it becomes activated into thegastric enzyme pepsin. Pepsin probably is a general name for several proteolyticenzymes which hydrolyze proteins into polypeptides.

E. is correct.

This is a characteristic feature of the parietal cell. The intracellular canaliculiactually represent deep invaginations of the apical plasmalemma.

D. is correct.

The chief cells secrete enzymes into the lumen of the intestinal gland and thereforehave their stored secretory product in the cytoplasm close to the gland lumen.

C. is correct.

Since the enteroendocrine cells are hormone producing cells they secrete towardthe capillaries located in the lamina propria just beneath the epithelium and itsbasal lamina. In other words, these cells have their apices toward the basal lamina,not toward the lumen of the intestinal gland. Thus, the hormone to be released is

Page 319: Review Questions for Human Histology

found in secretory granules in the "basal" (but really apical) region of the cellrelative to all of the other non-endocrine cells in the intestinal gland, which secretetoward the lumen of the gland not toward the capillaries in the lamina propria.

D. is correct.

Since the secretory product of the chief cell is enzymes such as lipase and inparticular pepsinogen, and since enzymes are proteins, the cytoplasmic machineryof the chief cell is exactly what one would expect to find in a cell that synthesizesand secretes protein: lots of RER, good Golgi and lots of secretoryvesicles/granules waiting to be exocytosed.

13.074 Which of the following has eosinophilic cytoplasm?

A. surface mucous cell

B. mucous neck cell

C. enteroendocrine cell

D. chief or zymogenic cell

E. parietal cell

13.075 Which of the following is a vasomotor substance?

A. gastrin

B. serotonin

C. secretin

D. cholecystokinin-pancreozymin

13.076 Which of the following stimulates sodium bicarbonate release?

A. gastrin

B. serotonin

C. secretin

D. cholecystokinin-pancreozymin

13.077 Which of the following stimulates release of bile from the gall bladder?

A. gastrin

Page 320: Review Questions for Human Histology

B. serotonin

C. secretin

D. cholecystokinin-pancreozymin

13.078 Which of the following stimulates a low pH secretion?

A. gastrin

B. serotonin

C. secretin

D. cholecystokinin-pancreozymin

E. is correct.

In regular H&E sections of the fundic stomach the parietal cells stain red. This isprobably due to the large number of mitochondria in the cytoplasm of these cellsand has nothing to do with the fact that these cells secrete hydrochloric acid. Acidiccell components such as DNA and RNA (rough ER or free polysomes) arebasophilic and attract hematoxylin.

B. is correct.

Serotonin is released by some of the enteroendocrine cells. It is a vasomotorsubstance which can induce the smooth muscle of the tunica media of vessels,particularly arterioles, to contract.

C. is correct.

Secretin is produced by enteroendocrine cells in the mucosa of the duodenum inresponse to the presence of acid chyme in the lumen of the duodenum. The targetfor secretin is the ductal epithelium of the pancreas which, in response to secretin,releases a watery secretion containing sodium bicarbonate which will raise (buffer)the pH in the lumen of the duodenum.

D. is correct.

CCK-PZ is now known to be one hormone. One of its actions is to cause the gallbladder to release bile into the common bile duct and, of course, eventually into thelumen of the duodenum.

A. is correct.

Page 321: Review Questions for Human Histology

Gastrin is released by enteroendocrine cells in the gastric glands and it stimulatesgastric secretion, some of which involves the release of HCl by the parietal cells.

Page 322: Review Questions for Human Histology

SECTION 14: URINARY SYSTEM

14.001 Arrange the following regions in proper sequence for the passage of filtratethrough the nephron: 1=distal convoluted tubule, 2=proximal convoluted tubule,3=descending limb of the loop of Henle, 4=ascending limb of loop of Henle, 5=thinsegment of loop of Henle, 6=arched collecting tubule.

A. 1-2-3-4-5-6

B. 2-1-4-3-5-6

C. 3-2-4-1-5-6

D. 2-3-511-1-6

E. 3-4-5-1-2-6

14.002 In general the kidney has a cortex and a medulla. Identify the renalstructure which is a mass of cortex extending into the medulla?

A. pyramid

B. papilla

C. minor calyx

D. lobule

E. column (of Bertin)

14.003 Arrange the following structures in proper sequence for the flow of urine:1=ureter, 2=urinary bladder, 3=papilla, 4=renal pelvis, 5=minor calyx, 6=majorcalyx.

A. 1-4-5-3-6-2

B. 6-4-5-3-1-2

C. 4-1-3-5-6-2

D. 5-6-4-1-3-2

E. 3-5-6-4-1-2

14.004 What structure is found at the center of a renal lobule?

Page 323: Review Questions for Human Histology

A. column of Bertin

B. afferent arteriole

C. efferent arteriole

D. medullary ray

E. macula densa

14.005 What structure is found at the periphery of a renal lobule?

A. column of Bertin

B. afferent arteriole

C. efferent arteriole

D. muscular artery

E. straight descending limb of loop of Henle

D. is correct.

This is the sequence of structures encountered as glomerular filtrate passesthrough the nephron.

E. is correct.

The renal columns are masses of cortex which extend into and sort of separatemasses of medulla from each other. The conical masses of medulla are thepyramids. When the cap of cortex is added to a pyramid the combination results ina renal lobe. The papilla is the rounded off tip of the lobe which projects into theminor calyx.

E. is correct.

D. is correct.

Areas of medulla extend outward into the cortex from the base of the medullarypyramid. These medullary rays are composed of collecting ducts and collectivelyform the center of a renal lobule.

D. is correct.

The small artery located at the outside edge of a renal lobule histologically is a

Page 324: Review Questions for Human Histology

muscular artery. In this region this vessel also can be called an interlobular artery.Thus a renal lobule has an interlobular artery at both "edges" and a medullary rayin its center.

14.006 Arrange the following vessels in proper sequence for blood flow into thekidney: 1=renal artery, 2=arcuate artery, 3=interlobar artery, 4=afferent arteriole,5=interlobular artery, 6=efferent arteriole.

A. 1-3-2-5-4-6

B. 1-2-34-5-6

C. 111-3-2-5-6

D. 1-54-3-2-6

E. 1-3-2-4-5-6

14.007 Which of the following cell types is equivalent to the visceral layer ofBowman's capsule?

A. continuous endothelium

B. fenestrated endothelium

C. podocytes

D. simple squamous epithelium(not endothelium)

E. cuboidal epithelium with prominent microvilli

14.008 What "bounds" or encloses the urinary space?

A. visceral layer of Bowman's capsule

B. parietal layer of Bowman's capsule

C. opposing layers of transitional epithelium

D. A & B are correct

E. A & B & C are correct

14.009 Which region/structure communicates directly with the urinary space?

A. lumen of DCT

Page 325: Review Questions for Human Histology

B. lumen of PCT

C. lumen of afferent arteriole

D. lumen of efferent arteriole

E. lumen of loop of Henle

14.010 Arrange in sequence for the formation of glomerular filtrate: 1=glomerularcapillary lumen, 2=urinary space, 3=lamina lucida (rara) externa, 4=lamina densa,5=lamina lucida (rara) interna, 6=endothelial fenestration.

A. 1-311-5-6-2

B. 1-6-34-5-2

C. 1-4-5-6-3-2

D. 1-6-5-4-3-2

E. 1-54-3-6-2

A. is correct.

C. is correct.

During development as the capillary tuff, the glomerulus, invaginates Bowman'scapsule, one layer of the capsule becomes intimately associated with theglomerulus and this is the visceral layer of Bowman's capsule. The cells of thevisceral layer of Bowman's capsule are the podocytes.

D. is correct.

The urinary space is that area found between the visceral and parietal layers ofBowman's capsule. The visceral layer is composed of podocytes and the parietallayer is composed of simple squamous epithelium.

B. is correct.

The urinary space is in direct communication with the lumen of the PCT. This placeof direct communication between the PCT and the urinary space is called theurinary pole.

D. is correct.

14.011 Not all molecules pass through the glomerular basement membrane. Whichof the following is/are true for the molecular sieve function of the glomerular

Page 326: Review Questions for Human Histology

basement membrane?

A. molecules with a molecular weight of less than about 70,000 pass through

B. molecules with a molecular weight of less than about 70,000, but with a positivecharge pass through

C. molecules with a molecular weight of less than about 70,000, but with a highnegative charge pass through

D. A & B are correct

E. A & B & C are correct

14.012 Which of the following cell types manufactures and releases renin?

A. modified smooth muscle cells of the tunica media of the efferent arteriole

B. modified smooth muscle cells of the tunics media of the afferent arteriole

C. cells of the macula dense

D. lacis cells

E. mesangial cells

14.013 Renin release is stimulated by which of the following?

A. reduced stretch on the JG cells

B. above normal osmolarity of the filtrate in the lumen of the distal convolutedtubule

C. drop in systemic blood pressure

D. A & B are correct

E. A & B & C are correct

14.014 What does renin do?

A. convert angiotensinogen to angiotensin I

B. convert angiotensin Ito angiotensin II

C. convert angiotensin II to angiotensin III

Page 327: Review Questions for Human Histology

14.015 Which of the following cell types most closely resembles a regular capillarypericyte?

A. lacis cells

B. cells of the macula dense

C. JG cells

D. mesangial cells

E. cells in the tunica media of efferent arteriole

D. is correct.

Macromolecules with a molecular weight slightly less than 70,000, but with astrong negative charge due not pass through. The exclusion of negatively chargedmacromolecules with molecular weights of less than 70,000 is a function of thenegatively charged heparan sulfate proteoglycan in the glomerular basementmembrane.

B. is correct.

The juxtaglomerular cells or JG cells are the cells which release renin. The JGcells are modified smooth muscle cells in the wall of the afferent arteriole.

E. is correct.

If the systemic blood pressure drops the degree of stretch in the JG cells lessens,this will stimulate the release of renin. The macula dense monitors the osmolarityof the filtrate in the lumen of the DCT. If this osmolarity is not normal the maculadense "signals" the JG cells to release renin.

A. is correct.

Renin converts the glycoprotein angiotensinogen to the decapeptide angiotensin I.Angiotensin I is converted to angiotensin II by a converting enzyme located in theendothelial cells of the pulmonary capillaries. This series of events is known as therenin-angiotensin axis (RAS). Organ specific or local acting RASs have beenfound in brain, heart, testis and also kidney. The local RASs can work in concertwith the systemic RAS or independently.

D. is correct.

Mesangial cells are pericyte-like cells found in the wall of the glomerular capillary.

Page 328: Review Questions for Human Histology

14.016 What does angiotensin II do?

A. contraction of smooth muscle in systemic arterioles

B. stimulates zona glomerulosa of adrenal gland

C. raises level of aldosterone in blood

D. A & B are correct

E. A & B & C are correct

14.017 What does aldosterone do?

A. causes retention of potassium by DCT

B. causes retention of sodium by DCT

C. causes retention of potassium by PCT

D. causes retention of sodium by PCT

E. causes retention of ADH by DCT

14.018 A patient in your care has a high volume of urine production. The urine ispale and very dilute. Which of the following correctly applies to this situation?

A. not enough ADH

B. too much ADH

C. diabetes mellitus

D. not enough aldosterone

E. not enough angiotensinogen

14.019 All of the following have a transitional epithelium lining EXCEPT:

A. urinary bladder

B. renal pelvis

C. ureter

D. proximal urethra

Page 329: Review Questions for Human Histology

E. distal urethra

14.020 If blood flow into the kidneys is below normal the kidney will be ischemic.What substance or substances will the ischemic kidney release in an attempt toreverse this ischemia?

A. renin

B. erythropoietin

C. aldosterone

D. A & B are correct

E. A & B & C are correct

E. is correct.

Angiotensin II directly stimulates the tunica media of the systemic arterioles.Angiotensin II also stimulates the zona glomerulosa of the adrenal gland to releasealdosterone.

B. is correct.

ADH causes the DCT to reabsorb sodium ions and secrete potassium ions.

A. is correct.

In the absence of ADH the collecting ducts are not permeable to water andtherefore water can not be removed from the tubular urine. The defect is in thehypothalamus where ADH is synthesized. If little ADH is synthesized byhypothalamic neurons, then little of it can be released by the pars nervosa of thehypophysis.

E. is correct.

From the minor calyces to the proximal urethra the lining is transitional epithelium.At some point distal to the proximal urethra the transitional epithelium gives way toother types of lining epithelium such as stratified or pseudostratified columnarepithelium.

D. is correct.

Poor oxygenation of the kidneys will cause the release of renin and erythropoietin.In a way the kidney is attempting to reoxygenate itself by boosting the production ofRBCs. In addition, when renin is released and the angiotensin I - II cascade occurs

Page 330: Review Questions for Human Histology

there is an increase in the tonus of systemic arterioles which will increase systemicblood pressure. In addition the zona glomerulosa will release aldosterone whichwill cause the DCT to conserve sodium which will be followed by chloride. AddingNaCl to the blood will cause water to follow and the blood volume will beincreased, thereby increasing the blood pressure.

14.021 Inflammatory damage to the glomerular basement membrane may result inwhich of the following?

A. massive loss of plasma proteins to the urine

B. proteinuria

C. hypoalbuminemia

D. A & B are correct

E. A & B & C are correct

14.022 What are the "vasa recta"?

A. branches from the efferent arteriole

B. straight capillaries running adjacent to the descending and ascending limbs

C. branches from the arcuate arteries

D. A & B are correct

E. A & B & C are correct

14.023 Presence of well developed microvilli forming a brush or striated border:

A. proximal convoluted tubule

B. descending thin limb of loop of Henle

C. ascending thin limb of loop of Henle

D. distal convoluted tubule

E. straight collecting duct

14.024 Reabsorbs all plasma proteins escaping through glomerular basementmembrane:

A. proximal convoluted tubule

Page 331: Review Questions for Human Histology

B. descending thin limb of loop of Henle

C. ascending thin limb of loop of Henle

D. distal convoluted tubule

E. straight collecting duct

14.025 Not permeable to water unless ADH is present:

A. proximal convoluted tubule

B. descending thin limb of loop of Henle

C. ascending thin limb of loop of Henle

D. distal convoluted tubule

E. straight collecting duct

14.026 Directly connected to the urinary space:

A. proximal convoluted tubule

B. descending thin limb of loop of Henle

C. ascending thin limb of loop of Henle

D. distal convoluted tubule

E. straight collecting duct

E. is correct.

Damage to the glomerular basement membrane can result in a very "leaky"basement membrane. The leaky membrane leaks plasma protein such asalbumen. Protein in the urine is proteinuria. Less albumin in the blood ishypoalbuminemia. These clinical conditions are components of the nephroticsyndrome.

D. is correct.

The efferent arterioles break up into peritubular capillaries. Those arising from thejuxtamedullary glomeruli pursue straight courses into the depths of the medullaservicing the loops of Henle and then returning to the arcuate veins at the cortico-medullary junction.

Page 332: Review Questions for Human Histology

A. is correct.

A. is correct.

E. is correct.

A. is correct.

14.027 Best place to fmd canaliculi between microvilli and lots of evidence ofpinocytotic vesicles:

A. proximal convoluted tubule

B. descending thin limb of loop of Henle

C. ascending thin limb of loop of Henle

D. distal convoluted tubule

E. straight collecting duct

14.028 Reabsorbs 100% of the glucose and amino acids:

A. proximal convoluted tubule

B. descending thin limb of loop of Henle

C. ascending thin limb of loop of Henle

D. distal convoluted tubule

E. straight collecting duct

14.029 Impermeable to water, but sodium pump works OK:

A. proximal convoluted tubule

B. descending thin limb of loop of Henle

C. ascending thin limb of loop of Henle

D. distal convoluted tubule

E. straight collecting duct

14.030 Principal target tissue for aldosterone:

Page 333: Review Questions for Human Histology

A. proximal convoluted tubule

B. descending thin limb of loop of Henle

C. ascending thin limb of loop of Henle

D. distal convoluted tubule

E. straight collecting duct

14.031 Secretes ammonium ions into the tubular urine:

A. proximal convoluted tubule

B. descending thin limb of loop of Henle

C. ascending thin limb of loop of Henle

D. distal convoluted tubule

E. straight collecting duct

14.032 Histological appearance closely resembles a capillary:

A. proximal convoluted tubule

B. ascending or descending thin limb of loop of Henle

C. distal convoluted tubule

D. straight collecting duct

A. is correct.

A. is correct.

C. is correct.

D. is correct.

D. is correct.

B. is correct.

Either the ascending or the descending thin limbs of the loop of Henle look thesame. They are lined by a simple squamous epithelium(not endothelium), whichmakes them look like a capillary without any blood in it.

Page 334: Review Questions for Human Histology

14.033 Macula densa found here:

A. proximal convoluted tubule

B. descending thin limb of loop of Henle

C. ascending thin limb of loop of Henle

D. distal convoluted tubule

E. straight collecting duct

14.034 Which of the following structures have cells with well developed microvilliforming a brush or striated border?

A. proximal convoluted tubule

B. descending thin limb of loop of Henle

C. ascending thin limb of loop of Henle

D. distal convoluted tubule

E. straight collecting duct

14.035 All plasma proteins escaping through the glomerular basement membraneare reabsorbed in which of the following regions?

A. proximal convoluted tubule

B. descending thin limb of loop of Henle

C. ascending thin limb of loop of Henle

D. distal convoluted tubule

E. straight collecting duct

14.036 Which of the following is not permeable to water unless ADH is present?

A. proximal convoluted tubule

B. descending thin limb of loop of Henle

C. ascending thin limb of loop of Henle

D. distal convoluted tubule

Page 335: Review Questions for Human Histology

E. straight collecting duct

D. is correct.

The macula densa is a spot or area on the DCT as it returns close to it glomerulus.It monitors the amount of sodium leaving in the urine. If too much sodium is aboutto be lost the cells of the macula densa "tell" their close neighbors, thejuxtaglomerular cells in the wall of the afferent arteriole to release renin. Reninconverts blood angiotensinogen to angiotensin-I which will be converted toangiotensin-II by an enzyme in the lung. One of the actions of angiotensin-II, is tostimulate the zona glomerulosa of the adrenal cortex to release aldosterone.Aldosterone "tells" the DCT to reabsorb more sodium thereby "correcting" theproblem. If sodium is retained in the blood, chloride will follow and water will moveto the NaCl. This will, in effect, increase the blood volume and by doing so willincrease the blood pressure.

A. is correct.

So much reabsorptive activity needs to happen in the PCT that the cells arestructurally "geared-up" to accomplish this. Part of this cytological story is thepresence of well developed microvilli which form the brush or striated bordercharacteristic of the PCT.

A. is correct.

This is the normal role of the cells of the PCT.

E. is correct.

ADH is required for the collecting ducts to be permeable to water. As the collectingducts pass through the medulla on their way to the tip of the renal pyramid theypass through a hypertonic interstitium and water is allowed to exit the collectingducts thereby concentrating the urine before it leaves the tip of the renal pyramidand moves into the minor calyx. Loss of ADH results in large amounts of waterbeing retained in the urine. This is diabetes insipidus.

14.037 Which of the following is directly connected to the urinary space?

A. proximal convoluted tubule

B. descending thin limb of loop of Henle

C. ascending thin limb of loop of Henle

D. distal convoluted tubule

Page 336: Review Questions for Human Histology

E. straight collecting duct

14.038 Which of the following is the best place to find canaliculi between microvillinear the cell surface and lots of evidence of pinocytotic vesicles?

A. proximal convoluted tubule

B. descending thin limb of loop of Henle

C. ascending thin limb of loop of Henle

D. distal convoluted tubule

E. straight collecting duct

14.039 Which region is responsible for the reabsorption of 100% of the glucoseand amino acids that pass through the glomerular basal lamina?

A. proximal convoluted tubule

B. descending thin limb of loop of Henle

C. ascending thin limb of loop of Henle

D. distal convoluted tubule

E. straight collecting duct

14.040 Which of the following is impermeable to water, but its sodium pump worksOK?

A. proximal convoluted tubule

B. descending thin limb of loop of Henle

C. ascending thin limb of loop of Henle

D. distal convoluted tubule

E. straight collecting duct

14.041 Which of the following is the principal target tissue for aldosterone?

A. proximal convoluted tubule

B. descending thin limb of loop of Henle

Page 337: Review Questions for Human Histology

C. ascending thin limb of loop of Henle

D. distal convoluted tubule

E. straight collecting duct

14.042 Which of the following secretes ammonium ions into the tubular urine?

A. proximal convoluted tubule

B. descending thin limb of loop of Henle

C. ascending thin limb of loop of Henle

D. distal convoluted tubule

E. straight collecting duct

A. is correct.

The lumen of the urinary, or Bowman's, space in the glomerulus is continuous withthe lumen of the PCT.

A. is correct.

So much reabsorption is taking place in the PCT that there is ample ultrastructuralevidence of this activity in the individual cells of the PCT.

A. is correct.

This is one of the normal roles of the cells of the PCT.

C. is correct.

On the ascending limb of the loop of Henle the cells pump sodium from urine toblood, but since they are impermeable to water, water can not follow the sodium.Leaving the water behind in the forming urine after having removed sodium, willcause the forming urine to become progressively more hypotonic as it ascends inthe ascending limb of the loop of Henle.

D. is correct.

The DCT is the major target for aldosterone. Aldosterone causes the DCT cells toreabsorb more sodium into the blood. If sodium is added to blood water willeventually follow to keep the tonicity of the blood normal. This will expand thevolume of blood in the cardiovascular system and will result in an increase in bloodpressure, all other things being equal.

Page 338: Review Questions for Human Histology

D. is correct.

This is one of the functions of the DCT, i.e. to secrete or add to the urineammonium and also potassium ions.

Page 339: Review Questions for Human Histology

SECTION 15: ENDOCRINE SYSTEM

15.001 When a hormone or hormone-like chemical messenger is mostly activelocally instead of at long distances from the source of the message, the process iscalled?

A. autocrine

B. paracrine

C. endocrine

D. exocrine

E. merocrine

15.002 Receptors for hormones can be located where?

A. cell surface site

B. intracellular site

C. extracellular site

D. A & B are correct

E. A & B & C are correct

15.003 Some hormones directly effect their target tissue and other hormones usean intermediary or indirect route which involves the release of a second hormone.Which of the following hormones use the direct method of control?

A. insulin

B. parathyroid hormone

C. thyroid releasing hormone

D. A & B are correct

E. A & B & C are correct

15.004 All of the following components or regions of the hypophysis are derivedfrom oral ectoderm EXCEPT:

Page 340: Review Questions for Human Histology

A. pars distalis

B. pars nervosa

C. pars tuberalis

D. pars intermedia

15.005 All of the following apply to Rathke's pouch EXCEPT:

A. it gives rise to the pituicytes

B. it gives rise to the basophils

C. it arises from oral ectoderm

D. it gives rise to the chromophobes

E. it gives rise to the acidophils

B. is correct.

When one cell type releases a chemical messenger and it acts locally on adjacentcells, this is called paracrine communication or chemical signaling. When a cellstimulates itself the word autocrine applies. Endocrine refers to a relatively longdistance chemical signaling which uses the blood stream as the transporter of themessage. The signaling molecules produced by endocrine cells are calledhormones.

D. is correct.

In general the lipid soluble hormones such as the steroid hormones pass throughthe cell membrane and connect up with their receptor site inside of the cell. Non-lipid soluble hormones such as those which are not derived from cholesterol butfrom amino acids, have their receptor sites on the cell surface.

D. is correct.

Thyroid releasing hormone or TRH stimulates a subset of basophils in the parsdistalis to release thyroid stimulating hormone which is the hormone whichstimulates the thyroid gland to release its hormones. Insulin and parathyroidhormone act directly on their target tissues without involving an intermediatehormone step.

B. is correct.

The pars nervosa is derived from an outgrowth of the developing brain. All of the

Page 341: Review Questions for Human Histology

other parts listed are derived from an outpouching of oral epithelium.

A. is correct.

The pituicytes are the stromal cells of the pars nervosa which is derived from theneuroectoderm. All other cell types are derived from Rathke's pouch.

15.006 Which of the following in the adult pituitary gland represents the remains ofthe lumen of Rathke's pouch in the embryo?

A. sinusoid in pars distalis

B. center of small follicle in pars intermedia

C. Herring body in pars nervosa

D. group of chromophobes

E. capillary in pars nervosa

15.007 Which of the following endocrine glands contains a portal system ofvessels?

A. thyroid

B. parathyroid

C. adrenal cortex - adrenal medulla

D. hypophysis

E. ovary

15.008 All of the following hormones are synthesized in the parenchymal cells ofthe hypophysis EXCEPT:

A. ACTH

B. TSH

C. ADH

D. FSH

E. STH

15.009 A neuron with a short axon in the hypothalamus monitors the blood level of

Page 342: Review Questions for Human Histology

thyroid hormone. When the blood level of thyroid hormone drops below a certainthreshold level what cascade of events happens in what sequence: 1=TSHreleased from basophil, 2=TSH-RF released from axon terminal, 3=T3 & T4released from thyroid parenchyma, 4=TSH-RF finds its receptor in its target cell,5=blood level of thyroid hormone returns to normal in capillaries of hypothalamus.

A. 1-2-3-4-5

B. 213-1-3-5

C. 4-1-2-3-5

D. 3-2-1-4-5

E. 4-2-3-1-5

B. is correct.

The pars intermedia can contain small follicles. The lumen of a follicle in the parsintermedia is a remnant of the original lumen of Rathke's pouch.

D. is correct.

The pituitary gland has two sets of capillaries connected by one or two veins thusforming a portal circulatory system (one capillary bed connected to a secondcapillary bed by venous structures as in the well-known example for the intestinalcapillaries collected into the portal vein which then forms a second capillarynetwork in the liver). One set of capillaries is located in the inferior extent of thehypothalamus and the other capillary bed is in the adenohypophysis. Thisarrangement of vessels in this region of the body is called the hypophyseal portalsystem.

C. is correct.

ADH or antidiuretic hormone is made in nerve cell bodies in the hypothalamus. It isthen transported down a long axon system which terminates adjacent to capillariesin the pars nervosa. All other hormones listed are manufactured by parenchymalcells of the adenohypophysis.

B. is correct.

This is a negative feedback loop in which rising levels of T3 & T4 inhibitmanufacture and release of TSHRH which in turn would lessen the amount of TSHreleased, which in turn would decrease the amount of T3 & T4 in the blood.Decreasing blood levels of T3 & T4 would then "restart" the sequence listed inchoice B.

Page 343: Review Questions for Human Histology

15.010 Which of the following cell types produces not one but two hormones?

A. a neuron in the supraoptic nucleus of the hypothalamus

B. a neuron in the paraventricular nucleus of the hypothalamus

C. mammotropic cell

D. corticotropic cell

E. gonadotropic cell

15.011 Which of the following hormones from the pars distalis are known to haveboth stimulating and inhibiting releasing factors effect them from thehypothalamus?

A. prolactin

B. ACTH

C. FSH

D. A & B are correct

E. A & B & C are correct

15.012 Which of the following clinical conditions would result from too muchgrowth hormone being produced by a benign, functioning neoplasm (adenoma) ofthe anterior pituitary gland occurring in a 40 yr. old male?

A. gigantism

B. dwarfism

C. diabetes insipidus

D. acromegaly

15.013 As thyroid hormone is being produced and stored inside of the thyroidfollicle in the colloid, where does iodination of the glycoprotein occur?

A. SER

B. RER

C. Golgi

Page 344: Review Questions for Human Histology

D. secretory vesicles as they approach the cell surface for exocytosis

E. contents of secretory vesicles immediately after exocytosis

15.014 When TSH arrives and stimulates the follicular cells in the thyroid tobecome activated, what do they do?

A. synthesize and deliver to the lumen of the follicle thyroglobulin

B. iodinated thyroglobulin is ingested and cleaved by lysosomal enzymes

C. T3 and T4 are released via intracellular hydrolytic action on thyroglobulin andthese two molecules escape from the follicular cells into the blood

D. A & B are correct

E. A & B & C are correct

E. is correct.

The rule is that one hormone comes from one cell, or a single cell can notsynthesize and release two hormones. However, the gonadotropic basophil in thepars distalis is known to synthesize and release both FSH and LH.

A. is correct.

Prolactin and one other anterior pituitary hormone, growth hormone, have bothinhibitory and stimulatory hormones effect them from the hypothalamus. Thus thereis GRH or growth hormone releasing hormone and GIH or growth hormoneinhibiting hormone. The same situation applies to prolactin with PRH and PIHbeing made in some of the neurons in the hypothalamus.

D. is correct.

Gigantism would result from too much GH being produced before closure of theepiphyses. The opposite of gigantism would be dwarfism in which instance notenough GH was produced

E. is correct.

The iodination of the tyrosyl groups occurs outside the cell membrane immediatelyafter exocytosis of the uniodinated glycoprotein.

E. is correct.

TSH stimulated follicular cells perform all of these functions at the same time. One

Page 345: Review Questions for Human Histology

mode is a synthesis mode for the storage of more hormone in thyroglobulin and theother mode is a release mode for the liberation of T3 and T4 to the blood.

15.015 You are getting ready to look at a biopsy from a thyroid gland from a patientwith a long-standing basophil adenoma of the anterior pituitary gland in whichabnormal levels of TSH were produced. What do you expect to see, histologicallyin the biopsy?

A. flat squamosal follicular cells and lots of colloid

B. cuboidal follicular cells and lots of colloid

C. increased numbers of parafollicular cells

D. tall columnar follicular cells and not much colloid

E. decreased numbers of parafollicular cells

15.016 When the blood calcium level rises to abovenormal levels what cell typesbecome activated?

A. osteoclast

B. parafollicular or C cells in thyroid gland

C. thyroid follicular epithelium

D. chief cells of parathyroid gland

E. oxyphil cells of parathyroid gland

15.017 Which of the following cell types would contain the highest concentration ofintracellular mitochondria?

A. squamosal thyroid follicular cell

B. parafollicular cell in thyroid gland

C. chief cell of parathyroid gland

D. oxyphil cells of parathyroid gland

E. chromophobe of pars distalis

15.018 The near absence of which of the following hormones is associated withhyperexcitability and spastic contraction of skeletal muscle cells all over the body(tetany)?

Page 346: Review Questions for Human Histology

A. calcitonin

B. renin

C. ACTH

D. PTH

E. T3 & T4

15.019 Which of the following cell types is NOT an example of a parenchymal cell?

A. hypophysealbasophil

B. chromafl'm cell in adrenal medulla

C. parafollicular or C cell in the thyroid gland

D. pituicyte near a Herring body in the pars nervosa

E. chief cell in parathyroid gland

D. is correct.

High levels of TSH for a prolonged period of time would cause a hypertrophy of thefollicular cells so that they would be columnar in shape. This increase in the size ofthe follicular cells throughout the gland would increase the size of the gland andthis externally obvious enlargement is called a goiter.

B. is correct.

Above-normal level of calcium stimulates the parafollicular or C cells to releasecalcitonin which inhibits osteoclastic activity, thereby eventually lowering the bloodlevel of calcium.

D. is correct.

The oxyphil cells of the parathyroid gland stain pink/red on H & E and under theEM, they are noted to have a high number of mitochondria, perhaps one of thehighest concentrations of mitochondria of all cell types.

D. is correct.

The loss of parathyroid hormone would mean that the blood level of calcium woulddrop due to nonstimulation of osteoclastic activity. This low level of serum calciumresults in tetany, which is extremely serious when one considers that laryngeal

Page 347: Review Questions for Human Histology

spasms could cause suffocation.

D. is correct.

The pituicyte is equivalent to a glial component of the CNS, i.e. it represents astromal element, not a parenchymal element. Pituicytes do not secrete a hormone.The hormones released from the pars nervosa are synthesized in neuron cellbodies in the paraventricular and supraoptic nuclei of the hypothalamus. ADH andoxytocin are transported down a long axon system to be temporarily stored inswollen axon terminals called Herring bodies prior to release into nearbycapillaries.

15.020 This endocrine gland receives light/dark information by way of the retina-optic chiasmpostganglionic fibers from the superior cervical ganglion.

A. adrenal medulla

B. posterior pituitary gland

C. pineal gland

D. anterior pituitary gland

E. zona reticularis of adrenal cortex

15.021 Rats exposed to continuous darkness have which of the following?

A. they don't enter estrus

B. they have high levels of melatonin

C. they demonstrate a high amplitude circadian rhythm in melatonin levels

D. A & B are correct

E. A & B & C are correct

15.022 Insulin-like growth factor or IGF-1 is now known to be the same as ______

A. insulin

B. glucagon

C. somatomedin

D. somatostatin

Page 348: Review Questions for Human Histology

E. melatonin

15.023 A neurosurgeon removes a benign neoplasm or adenoma of basophils inthe pars distalis. What disease is being treated?

A. diabetes insipidus

B. diabetes mellitus

C. Cushing's syndrome of adrenal gland origin

D. Addison's disease of adrenal gland origin

E. inappropriate release of catecholamines causing abrupt rises in blood pressure,nocturnal sweats

15.024 ACTH is produced by this cell:

A. chromophobe

B. basophil

C. acidophil

D. hypothalamic neuron with short axons

E. hypothalamic neuron with long axons

C. is correct.

The pineal gland is "innervated" with postganglionic sympathetic fibers from thesuperior cervical ganglion which receives its "visual information" from the pathwaymentioned in the question.

D. is correct.

Under normal environmental conditions there is a high amplitude circadian rhythmin melatonin levels with values during the nocturnal phase about 3 times higherthan during the diurnal phase. However, under continuous darkness the levels ofmelatonin remain high and since melatonin has an antigonadotrophic effect,animals kept in continuous darkness are prevented from entering estrus. If animalswere kept in continuous light, the levels of melatonin would be low and thus theantigonadotrophic effect would be absent. Animal reared under these conditionsenter estrus prematurely.

C. is correct.

Page 349: Review Questions for Human Histology

Growth hormone or somatotrophic hormone released from a type of acidophil in thepars distalis acts on the liver to release somatomedin which stimulates the growthof chondrocytes in the epiphyseal plate. When the biochemical structure of IGF-1and somatomedin were worked out it was discovered that they were identical.

C. is correct.

90% of the patients with Cushing's syndrome or too much hormone from theadrenal cortex have a benign functioning adenoma of basophils in the pars distaliswhich secrets too much ACTH.

B. is correct.

ACTH is produced by one of the three types of basophils in the pars distalis of thehypophysis.

15.025 TSH is produced by this cell:

A. chromophobe

B. basophil

C. acidophil

D. hypothalamic neuron with short axons

E. hypothalamic neuron with long axons

15.026 Oxytocin is produced by this cell:

A. chromophobe

B. basophil

C. acidophil

D. hypothalamic neuron with short axons

E. hypothalamic neuron with long axons

15.027 TSH-RH is produced by this cell:

A. chromophobe

B. basophil

C. acidophil

Page 350: Review Questions for Human Histology

D. hypothalamic neuron with short axons

E. hypothalamic neuron with long axons

15.028 STH is produced by this cell:

A. chromophobe

B. basophil

C. acidophil

D. hypothalamic neuron with short axons

E. hypothalamic neuron with long axons

15.029 LTH is produced by this cell

A. chromophobe

B. basophil

C. acidophil

D. hypothalamic neuron with short axons

E. hypothalamic neuron with long axons

15.030 FSH-RH is produced by this cell:

A. chromophobe

B. basophil

C. acidophil

D. hypothalamic neuron with short axons

E. hypothalamic neuron with long axons

B. is correct.

TSH or thyroid stimulating hormone is a product of one of the three different typesof basophils in the pars distalis of the hypophysis.

E. is correct.

Page 351: Review Questions for Human Histology

Oxytocin is made by a type of neuron which can be found in both theparaventricular and supraoptic nuclei of the hypothalamus. The hormone travels tothe axon terminal in the pars nervosa over a relatively long axon when comparedto the axon length used by the releasing factors or hormones.

D. is correct.

TSH-RH is made in neuron cell bodies in the hypothalamus. The hormone istransported over a short axon system to be released into the hypothalmo-hypophyseal portal system, which is two capillary beds, one in the pars distalis andthe other in the median eminence, connected by a portal vein.

C. is correct.

STH or somatotropic hormone is a product of one type of acidophil of the parsdistalis.

C. is correct.

LTH or lactogenic hormone, also known as prolactin, is a product of one of the twotypes of acidophils of the pars distalis.

D. is correct.

FSH-RH or follicle stimulating hormone releasing hormone is made in neuron cellbodies in the hypothalamus. It is transported down a relatively short axon whichterminates on a capillary in the median eminence region of the brain. The hormoneis released into the blood of the capillary where it is collected in a portal vein whichforms another capillary bed in the pars distalis where FSH-RH will stimulate itstarget cell, a type of basophil, to release FSH.

15.031 ADH is produced by this cell:

A. chromophobe

B. basophil

C. acidophil

D. hypothalamic neuron with short axons

E. hypothalamic neuron with long axons

15.032 LH is produced by this cell:

A. chromophobe

Page 352: Review Questions for Human Histology

B. basophil

C. acidophil

D. hypothalamic neuron with short axons

E. hypothalamic neuron with long axons

15.033 FSH is produced by this cell:

A. chromophobe

B. basophil

C. acidophil

D. hypothalamic neuron with short axons

E. hypothalamic neuron with long axons

15.034 This hormone promotes milk secretion:

A. ACTH

B. LTH (prolactin)

C. TSH

D. LH

E. STH

F. FSH

15.035 This hormone promotes release of another hormone from colloid:

A. ACTH

B. LTH (prolactin)

C. TSH

D. LH

E. STH

F. FSH

Page 353: Review Questions for Human Histology

15.036 This hormone promotes release of corticosteroids:

A. ACTH

B. LTH (prolactin)

C. TSH

D. LH

E. STH

F. FSH

E. is correct.

ADH or antidiuretic hormone is produced by neuron cell bodies in both theparaventricular and supraoptic nuclei of the hypothalamus. These neurons sendthis hormone over a long axon which ends in the pars nervosa of the hypophysiswhere the hormone is released into nearby capillaries. ADH can be stored in theaxon terminal before release. In this case the stored hormone "fill" the axonterminal and this little swelling is termed a Herring body.

B. is correct.

LH is produced by one type of basophil or gonadotroph in the pars distalis of thepituitary gland.

B. is correct.

FSH is produced by one type of basophil in the pars distalis of the hypophysis.

B. is correct.

Prolactin or lactogenic hormone (LTH) stimulates the glandular epithelium of thebreast to secrete milk.

C. is correct.

Colloid is the extracellular storage form of thyroid hormone (as thyroglobulin).Thyroid stimulating hormone or TSH promotes the uptake of thyroglobulin by thefollicular epithelial cells which then break the thyroglobulin down into TI, T2, T3and T4, with T3 and T4 being the active thyroid hormones.

A. is correct.

Page 354: Review Questions for Human Histology

ACTH or adrenocorticotropic hormone in general stimulates the adrenal cortex tosynthesize/release its hormones. ACTH is strongly stimulatory to the zonafasciculata.

15.037 This hormone promotes ovulation:

A. ACTH

B. LTH (prolactin)

C. TSH

D. LH

E. STH

F. FSH

15.038 This hormone stimulates Leydig cells:

A. ACTH

B. LTH (prolactin)

C. TSH

D. LH

E. STH

F. FSH

15.039 This hormone stimulates chondrocytes in the epiphyseal plate:

A. ACTH

B. LTH (prolactin)

C. TSH

D. LH

E. STH

F. FSH

15.040 This hormone stimulates the follicular cells of the ovary:

Page 355: Review Questions for Human Histology

A. ACTH

B. LTH (prolactin)

C. TSH

D. LH

E. STH

F. FSH

15.041 This region is derived from neural crest:

A. zona glomerulosa

B. zona fasciculata

C. zona reticularis

D. medulla

E. none of the above

15.042 The region is the target for angiotensin II:

A. zona glomerulosa

B. zona fasciculata

C. zona reticularis

D. medulla

E. none of the above

D. is correct.

The estrogen level in the blood rises due to the growth of a group of ovarianfollicles and the granulosa cells therein. This rising level of estrogen results in anegative feedback on FSH production/release but a positive feedback for LHproduction/release. Just before ovulation there is an surge of LH which in ratio tothe blood level of FSH causes ovulation.

D. is correct.

LH in the male is interstitial cell stimulating hormone or ICSH. Decreasing blood

Page 356: Review Questions for Human Histology

levels of testosterone is a positive feedback for ICSH production/release.

E. is correct.

STH or growth hormone actually causes the liver to release somatomedin, nowknown as insulin-like growth factor 1. IGF-1 stimulates the growth of thechondrocytes in the epiphyseal plate.

F. is correct.

FSH stimulates 10-15 primordial follicles, each containing a primary oocyte inprophase I of meiosis, to begin to grow and develop.. LH is also probablynecessary for this growth and differentiation.

D. is correct.

The parenchyma of the adrenal medulla is derived from neural crest

A. is correct.

The target tissue for angiotensin II is: a) the smooth muscle of arterioles and smallarteries in general and b) the zona glomerulosa, which is stimulated by angiotensinII to release aldosterone

15.043 This region is the main target for ACTH:

A. zona glomerulosa

B. zona fasciculata

C. zona reticularis

D. medulla

E. none of the above

15.044 Its hormone fosters the breakdown of glycogen to glucose in the liver:

A. alpha cell

B. beta cell

C. delta cell

D. none of the above apply

15.045 Possible autoimmune destruction of these cells results in type I diabetes:

Page 357: Review Questions for Human Histology

A. alpha cell

B. beta cell

C. delta cell

D. none of the above apply

15.046 Destruction of these cells results in diabetes insipidus:

A. alpha cell

B. beta cell

C. delta cell

D. none of the above apply

15.047 The region releases aldosterone:

A. zona glomerulosa

B. zona fasciculata

C. zona reticularis

D. medulla

E. none of the above

15.048 This region releases epinephrine:

A. zona glomerulosa

B. zona fasciculata

C. zona reticularis

D. medulla

E. none of the above

15.049 This region releases hydrocortisone:

A. zona glomerulosa

Page 358: Review Questions for Human Histology

B. zona fasciculata

C. zona reticularis

D. medulla

E. none of the above

B. is correct.

ACTH has a major effect on the zona fasciculata. ACTH has less of an effect onthe zona reticularis and still less of an effect on the zona glomerulosa

A. is correct.

Glucagon is a hyperglycenic factor produced by the alpha cells of the pancreaticislet. Insulin is antagonistic to glucagon because insulin is a hypoglycemic factor.

B. is correct.

Type I diabetes or diabetes mellitus results from the loss of the beta cells of thepancreatic islet.

D. is correct.

Diabetes insipidus results from a loss of antidiuretic hormone (ADH) produced byneurons in both the supraoptic and paraventricular nuclei of the hypothalamus andtransported over a long axon system to the pars nervosa of the hypophysis whereADH is released into capillaries.

A. is correct.

Aldosterone is a mineralocorticoid synthesized and released from the zonaglomerulosa.

D. is correct.

Epinephrine and norepinephrine are released from separate cell types in theadrenal medulla.

B. is correct.

Cortisone and hydrocortisone are synthesized on the smooth endoplasmicreticulum of the cells in the zona fasciculata.

15.050 This region releases some androgens:

Page 359: Review Questions for Human Histology

A. zona glomerulosa

B. zona fasciculata

C. zona reticularis

D. medulla

E. none of the above

15.051 This region releases a class of hormones called mineralocorticoids:

A. zona glomerulosa

B. zona fasciculata

C. zona reticularis

D. medulla

E. none of the above

15.052 This region releases melatonin:

A. zona glomerulosa B. zona fasciculata

C. zona reticularis

D. medulla

E. none of the above

15.053 Cords of cells in this region are arranged radially:

A. zona glomerulosa

B. zona fasciculata

C. zona reticularis

D. medulla

E. none of the above

15.054 This region releases a hormone antagonistic to aldosterone:

A. zona glomerulosa

Page 360: Review Questions for Human Histology

B. zona fasciculata

C. zona reticularis

D. medulla

E. none of the above

15.055 This region releases some estrogens:

A. zona glomerulosa

B. zona fasciculata

C. zona reticularis

D. medulla

E. none of the above

15.056 The parenchyma in this region is sometimes referred to as chromaffmtissue:

A. zona glomerulosa

B. zona fasciculata

C. zona reticularis

D. medulla

E. none of the above

C. is correct.

Androgens and estrogens are synthesized and released from the zona reticularis.Thus, there is a little bit of "female" in every male and a little bit of "male" in everyfemale. Normally these sex steroids are "detoxified" in the liver.

A. is correct.

The zona glomerulosa releases mineralocorticoids, the most famous of which isaldosterone.

E. is correct.

Melatonin is synthesized and released by the parenchymal cells of the pineal

Page 361: Review Questions for Human Histology

gland.

B. is correct.

In the zona glomerulosa the parenchymal cells are arranged in small islands orglomeruli. Radiating deep from these glomeruli are rather straight cords ofparenchymal cells comprising the zona fasciculata.

E. is correct.

Aldosterone acts on the kidney to conserve sodium. The hormone which acts onthe kidney to release sodium into the urine is atria natriuretic peptide which issynthesized and released by atria cardiomyocytes.

C. is correct.

The zona reticularis synthesizes and releases androgens and estrogens.

D. is correct.

Both the epinephrine and norepinephrine parenchymal cells of the adrenalmedulla take on a brownish-gray cast if potassium dichromate is included in thefixative.

15.057 This pancreatic islet cell contains water soluble, alcohol insolublegranules:

A. alpha cell

B. beta cell

C. delta cell

D. none of the above

15.058 This pancreatic islet cell contains alcohol soluble granules:

A. alpha cell

B. beta cell

C. delta cell

D. none of the above

15.059 This pancreatic islet cell releases somatostatin:

Page 362: Review Questions for Human Histology

A. alpha cell

B. beta cell

C. delta cell

D. none of the above

15.060 The hormone produced by this pancreatic islet cell works mostly by theparacrine, not endocrine route:

A. alpha cell

B. beta cell

C. delta cell

D. none of the above

15.061 The hormone produced by this pancreatic islet cell fosters the movement ofglucose into cells:

A. alpha cell

B. beta cell

C. delta cell

D. none of the above

15.062 The hormone produced by this pancreatic islet cell fosters the breakdownof glycogen to glucose in the liver:

A. alpha cell

B. beta cell

C. delta cell

D. none of the above

A. is correct.

These are properties of the alpha cell or glucagon.

B. is correct.

Page 363: Review Questions for Human Histology

The granules of proinsulin in the beta cell are alcohol soluble. In the history ofdiabetes research on the cells in the pancreatic islet this fact played an importantrole in helping to distinguish between different cell types.

C. is correct.

The delta cell synthesizes and releases somatostatin.

C. is correct.

Somatostatin is produced by the pancreatic delta cell. It works as a suppressor toboth the alpha and beta cells of the pancreatic islet. It suppresses the function of itstwo "neighbors" (the alpha and beta cells) via a paracrine route (transported toadjacent cells in tissue fluid in this case, or in saliva and mucus in the case ofepidermal growth factor). Somatostatin also suppresses the release of growthhormone from one of the two types of hypophyseal acidophils. In the latter instancesomatostatin uses a true endocrine (transported by the blood stream to its target)route.

B. is correct.

Insulin is a hypoglycemic factor, it fosters the movement of blood glucose fromblood into cells and tissues.

A. is correct.

Glucagon is a hyperglycenic factor produced by the alpha cells of the pancreaticislet. Insulin is antagonistic to glucagon because insulin is a hypoglycemic factor.

Page 364: Review Questions for Human Histology

SECTION 16: MALE REPRODUCTIVE SYSTEM

16.001 About how many days does it take for the process of spermatogenesis(spermatogonia to spermatozoon) to occur in the seminiferous epithelium?

A. 10

B. 30

C. 60

D. 90

E. 120

16.002 What is the normal concentration of sperm in each ml of the 3 ml of semenproduced at a normal ejaculation?

A. 100 thousand

B. 500 thousand

C. one million

D. 100 million

E. 300 million

16.003 Arrange the following structures in proper sequence for sperm transportduring ejaculation: 1=ductus epididymis, 2=ampulla of ductus deferens,3=seminiferous tubule, 4=efferent ductules, 5=tubuli recti, 6=rete testis, 7=prostaticurethra.

A. 3-2-1-4-5-6-7

B. 3-4-6-5-2-1-7

C. 3-1-2-6-5-4-7

D. 3-6-5-4-2-1-7

E. 3-5-6-4-1-2-7

16.004 Where does the final biochemical/functional maturation of sperm occur?

Page 365: Review Questions for Human Histology

A. head of epididymis

B. tail of epididymis

C. prostatic urethra

D. after addition of secretory product of bulbourethral glands (of Cowper)

E. vagina/uterus/oviduct

C. is correct.

It takes a little over two months to go from spermatogonium type A to a freshlyproduced sperm. Newly produced sperm are non-motile and move out of the testisinto the epididymis by fluid currents and peristaltic type contractions.

D. is correct.

The normal ejaculate contains 3 ml and 300 million sperm, therefore there areabout 100 million sperm in each nil of ejaculate. A sperm count below about 20million/ml usually means sterility.

E. is correct.

E. is correct.

Sperm taken from the head of the epididymis are infertile. Sperm taken from the tailof the epididymis are fertile. "Fertile" sperm are capable of undergoing capacitationwhich means they are then capable of the acrosomal reaction during the process offertilization. Capacitation is the process by which sperm become completelybiochemically functional. It takes about 8 hrs. for sperm to complete capacitationand this occurs in the female reproductive tract, not in the male reproductive tract.

16.005 Arrange the following in proper sequence for fertilization: 1=sperm inperivitelline space, 2=DNA synthesis occurs in both pronuclei, 3=zona-lysindigests a tunnel in a glycoprotein layer, 4=hyaluronidase released from acrosome,5=syngamy of pronuclei occurs.

A. 4-3-1-2-5

B. 1-2-3-4-5

C. 3-1-4-2-5

D. 2-1-3-4-5

E. 3-1-2-4-5

Page 366: Review Questions for Human Histology

16.006 Which of the following organs has an epithelial lining which contains"stereocilia"?

A. ampulla of ductus deferens

B. tubuli recti

C. prostatic urethra

D. ductus epididymis

E. seminiferous tubules

16.007 Vasectomy refers to removal of part or all of what?

A. seminal vesicle

B. ampulla of the ductus deferens

C. non-ampullary part of the ductus deferens

D. membranous urethra

E. tail of the epididymis

16.008 Which of the following organs has 3 distinct layers of smooth muscle in itswall?

A. seminiferous tubule

B. ductus epididymis

C. ductus deferens

D. ejaculatory duct

E. seminal vesicle

16.009 The following description best fits which organ: fibromuscular stroma,folded secretory epithelium which can be simple or pseudostratified, comprised ofmany compound tubuloalveolar glands.

A. ampulla of the ductus deferens

B. seminal vesicle

Page 367: Review Questions for Human Histology

C. prostate gland

D. bulbourethral gland

E. glands of Littre

16.010 How would an individual with XYY be produced?

A. nondisjunction in oogenesis - meiosis I

B. nondisjunction in oogenesis - meiosis II

C. nondisjunction in spermatogenesis - meiosis I

D. nondisjunction in spermatogenesis - meiosis II

E. nondisjunction in spermatogenesis in both meiosis I and meiosis II

A. is correct.

D. is correct.

The epithelial lining of the ductus epididymis is pseudostratified or stratifiedcolumnar epithelium with long, non-motile microvilli projecting into the lumen of theduct. The large microvilli are the stereocilia.

C. is correct.

The ductus or vas deferens is the muscular tube which connects the tail of theepididymis to the ejaculatory duct. As it ascends in the spermatic cord it is easy toapproach surgically. Removal of a segment of the ductus deferens with ligation ofthe two remaining free ends is a vasectomy.

C. is correct.

Of all of the choices, the ductus deferens has the thickest coat of smooth muscle.The smooth muscle in the wall of the ductus deferens is arranged into 3 distinctlayers: inner longitudinal, middle circular and outer longitudinal.

C. is correct.

In addition to the items listed in the description the prostate would have concretionsor areas of calcified secretory product. The presence of fibroblasts and smoothmuscle in the capsule and other stromal areas of this gland gives it a firmconsistency on palpation. Hyperplasia of the glandular elements, the connectivetissue elements and the muscular elements results in BPH or benign prostatichypertrophy or hyperplasia.

Page 368: Review Questions for Human Histology

D. is correct.

In order to have a gamete with YY the problem must occur in the male. In order fora sperm to end up with YY nondisjunction or non-separation of the two Ychromatids must occur at anaphase II, resulting in one sperm with YY and the otherwith no sex chromosomes.

16.011 During spermiogenesis the Golgi complex and some of the endoplasmicreticulum moves to one end of the nucleus. What will form in this region?

A. midpiece

B. flagellum

C. accessory dense fibers of tail

D. acrosome

E. plasmalemma of end of tail

16.012 During spermiogenesis the chromatin in the nucleus undergoes intensecondensation. What is happening to this chromatin?

A. histone chromosomal content increases

B. nonhistone chromosomal protein content increases

C. histones and nonhistone chromosomal proteins are replaced with acidicproteins

D. histones and nonhistone chromosomal proteins are replaced with very basicproteins (protamines)

16.013 During an erection of the penis which of the following connective areas isthe most rigid or nondistensible?

A. papillary layer of dermis

B. reticular layer of dermis

C. tunica albuginea of corpora cavernosa

D. tunica albuginea of corpus spongiosum

E. lamina propria of penile urethra

Page 369: Review Questions for Human Histology

16.014 This cell contains 46 chromosomes, each with 2 chromatids:

A. primary spermatocyte in prophase

B. secondary spermatocyte

C. spermatid

16.015 This cell contains 23 chromosomes, each with 1 chromatid:

A. primary spermatocyte in prophase

B. secondary spermatocyte

C. spermatid

16.016 This cells contains 1 C amount of DNA:

A. primary spermatocyte in prophase

B. secondary spermatocyte

C. spermatid

16.017 This cell has 4C amount of DNA:

A. primary spermatocyte in prophase

B. secondary spermatocyte

C. spermatid

D. is convect.

Once the Golgi body moves to the tip of the nucleus it produces a collection ofvesicles which eventually fuse together to form the acrosome.

D. is correct.

Highly basic proteins, protamines, replace the chromosomal histone andnonhistone proteins and this is a major factor in the condensation of the spermnucleus.

C. is correct.

The tunica albuginea of the corpora cavernosa is much like a thick-walled gardenhose whereas the t.a. of the corpus spongiosum is a thin-walled garden hose

Page 370: Review Questions for Human Histology

which makes it distensible when the corpus spongiosum fills with blood. Thisdistensibility of the t.a. of the corpus spongiosum permits the penile urethra toremain patent during an erection for the uncompromized transport of the semenduring ejaculation.

A. is correct.

The primary spermatocyte, having complete DNA synthesis and entered prophaseI of meiosis, contains 4C amount of DNA, 46 chromosomes and 92 chromatids.

C. is correct.

The spermatid, having completed both the reductional and equational divisionshas 23 chromosomes, each composed of one chromatid, and 1 C amount of DNA.

C. is correct.

IC amount of DNA is the gamete amount of DNA and is found in the spennatid andthe spermatozoan.

A. is correct.

This is the post-S phase amount of DNA or the amount of DNA in G2 and prophaseat the start of the meiotic process.

16.018 This cell contains both an X AND a Y chromosome:

A. primary spermatocyte in prophase

B. secondary spermatocyte

C. spenmatid

16.019 Has either a X OR a Y chromosome:

A. spermatid

B. secondary spermatocyte

C. primary spenmatocyte

D. A & B are correct

E. A, B & C are correct

16.020 Within which cell can the tetrad formation of meiotic chromosomes befound?

Page 371: Review Questions for Human Histology

A. primary spermatocyte in prophase

B. secondary spermatocyte

C. spermatid

16.021 This cell is the result of the reductional division of meiosis:

A. primary spennatocyte in prophase

B. secondary spermatocyte

C. spermatid

16.022 This cell is the result of the equational division of meiosis:

A. primary spermatocyte in prophase

B. secondary spermatocyte

C. spermatid

A. is correct.

This applies to the primary spermatocyte which has the full complement ofreplicated chromosomes, i.e. all homologous pairs of chromosomes with eachchromosome containing two chromatids.

D. is correct.

After disjunction of the paired homologous chromosomes occurs at anaphase I ofmeiosis, the pair of sex chromosomes separate and one is "given" to onesecondary spennatocyte and the other is "given" to the other secondaryspermatocyte. Thus a secondary spermatocyte can have only one sexchromosome and it can be either a X OR a Y. The X OR the Y would undergoequational division in meiosis H. In this way each spermatid would also containonly one sex chromosome and it would have to be a X OR a Y.

A. is correct.

During prophase I of meiosis I, homologous chromosomes of each pair ofchromosomes actually "fmd" each other and pair up. One member of the pair isderived from the maternal parent and the other member of the pair is derived fromthe paternal parent. Since each chromosome of each pair of homologouschromosomes has completed DNA synthesis during the preceding S phase, eachchromosome has two chromatids. Pairing two homologous chromosomes, each

Page 372: Review Questions for Human Histology

composed of two chromatids, results in the formation of a tetrad or 4 chromatids ontwo chromosomes paired together very closely.

B. is correct.

The reductional division of the meiotic process begins in the primary spermatocyteand ends with the production of two daughter cells called secondaryspermatocytes.

C. is correct.

The equational division of meiosis is the second division in the meiotic process.The equational division occurs in each secondary spermatocyte which divides intotwo daughter cells called spermatids.

16.023 This cell contains 23 chromosomes each of which contains 2 chromatids:

A. primary spermatocyte in prophase

B. secondary spermatocyte

C. spermatid

16.024 This cell contains a single acrosomal granule in an acrosomal vesicle:

A. primary spermatocyte in prophase

B. secondary spermatocyte

C. spermatid

16.025 Which of the following is a true stem cell, relatively undifferentiated?

A. type A spermatogonium

B. type B spermatogonium

C. Sertoli cell

D. Leydig cell

E. primary or secondary spermatocyte

16.026 Which of the following is the source of testosterone?

A. type A spermatogonium

Page 373: Review Questions for Human Histology

B. type B spermatogonium

C. Sertoli cell

D. Leydig cell

E. primary or secondary spermatocyte

16.027 Which of the following synthesizes androgenbinding protein?

A. type A spermatogonium

B. type B spermatogonium

C. Sertoli cell

D. Leydig cell

E. primary or secondary spermatocyte

16.028 The primary spermatocyte arises from this cell:

A. type A spermatogonium

B. type B spermatogonium

C. Sertoli cell

D. Leydig cell

E. primary or secondary spermatocyte

B. is correct.

This describes the secondary spermatocyte which has 23 chromosomes, 46chromatids and 2C amount of DNA. This cell will undergo meiosis II or theequational division of meiosis and at anaphase II the two chromatids of eachchromosome will split and move to opposite poles of the cell. The daughter cellsthus produced will contain 23 chromosomes, each composed of a singlechromatid, and 1 C amount of DNA.

C. is correct.

Formation of the acrosome begins after meiosis II is completed and the cellproduced is a spermatid.

A. is correct.

Page 374: Review Questions for Human Histology

The type A spermatogonium is the truce stem cell or vegetative intermitotic for theseminiferous epithelium.

D. is correct.

The interstitial cells of Leydig are the cell source of testosterone.

C. is correct.

The Sertoli cell synthesizes androgen-binding protein which binds testosteroneand keeps high levels of testosterone in the seminiferous epithelium. ABP is alsoreleased into the lumen of the seminiferous tubules where it moves testosterone"downstream" to the epithelia of the duct system out to at least the epididymis.

B. is correct.

Some type A spermatogonia differentia into type B spermatogonia which continueto differentiate into primary spermatocytes.

16.029 This cell secretes inhibin which decreases FSH release:

A. type A spermatogonium

B. type B spermatogonium

C. Sertoli cell

D. Leydig cell

E. primary or secondary spermatocyte

16.030 The product of this cell inhibits LH release:

A. type A spermatogonium

B. type B spermatogonium

C. Sertoli cell

D. Leydig cell

E. primary or secondary spermatocyte

16.031 This cell contains the enzymes to convert progesterone tohydroxyprogesterone to androstenedione:

Page 375: Review Questions for Human Histology

A. type A spermatogonium

B. type B spermatogonium

C. Sertoli cell

D. Leydig cell

E. primary or secondary spermatocyte

16.032 During spermatogenesis, cells of which type are not connected byintercellular bridges?

A. type A spermatogonium

B. type B spermatogonium

C. primary spermatocyte

D. secondary spermatocyte

E. spermatid

16.033 Cells of which type contribute occludens type junctions to form the blood-testis barrier?

A. type A spermatogonium

B. type B spermatogonium

C. Sertoli cell

D. Leydig cell

E. primary or secondary spermatocyte

C. is correct.

The Sertoli cell releases "inhibin" and this hormone inhibits the release of FSHwhich is necessary for normal spermatogenesis. Inhibin release is increased whenthe cell population in the seminiferous epithelium drops below a certain normalthreshold number or concentration.

D. is correct.

LH or ICSH (interstitial cell stimulating hormone) stimulates the Leydig cells tosynthesize/release more testosterone. High levels of testosterone have a negative

Page 376: Review Questions for Human Histology

feedback effect on ICSH release. Low levels of testosterone have a positivefeedback effect on ICSH release.

D. is correct.

These are some of the enzymatic steps the Leydig cell employs to manufacturetestosterone.

A. is correct.

Only the most primitive type A spermatogonia remain as individual cells whichserve as stem cells or vegetative intermitotics for the seminiferous epithelium.Once some of the daughter cells, derived from divisions of the type Aspermatogonia, begin to differentiate and eventually become primaryspermatocytes, they are connected by intercellular cytoplasmic bridges. Thus allspermatogenic cell types from the type B spermatogonia to mature spermatozoaare connected by intercellular bridges. Some authors would state that intercellularbridges can be found in the later stages of the type A spermatogonia as theydifferentiate into type B spermatogonia.

C. is correct.

The Sertoli cells form occludens type junctions with each other forcing tissue fluidderived from the capillaries of the interstitial connective tissue areas outside ofeach seminiferous tubule, to pass through Sertoli cell cytoplasm before it reachesthe spermatocytes, spermatids and spermatozoa. This is the blood-testis barrier.

16.034 Which of the following cell types is located in the basal compartment of theseminiferous tubule?

A. type A spermatogonium

B. spermatid

C. Sertoli cell

D. Leydig cell

E. primary or secondary spermatocyte

16.035 Which of the following cell types is located in the adluminal compartment ofthe seminiferous tubule?

A. type A spermatogonium

B. type B spermatogonium

Page 377: Review Questions for Human Histology

C. Sertoli cell

D. Leydig cell

E. primary or secondary spermatocyte

16.036 Phagocytosis and digestion of the residual bodies formed duringspermiogenesis are found in which cell type?

A. type A spermatogonium

B. type B spermatogonium

C. Sertoli cell

D. Leydig cell

E. primary or secondary spermatocyte

16.037 Where is the mitochondrial sheath found?

A. in the head of the spermatozoon

B. in the midpiece of the spermatozoon

C. in the tail of the spermatozoon

D. none of the above

16.038 Where are 23 chromatids found?

A. in the head of the spermatozoon

B. in the midpiece of the spermatozoon

C. in the tail of the spermatozoon

D. none of the above

16.039 Where are hyaluronidase and zona-lysin found?

A. in the head of the spermatozoon

B. in the midpiece of the spermatozoon

C. in the tail of the spermatozoon

Page 378: Review Questions for Human Histology

D. none of the above

16.040 Where is heterochromatin found?

A. in the head of the spermatozoon

B. in the midpiece of the spermatozoon

C. in the tail of the spermatozoon

D. none of the above

A. is correct.

The basal compartment is that area between the blood-testis barrier formed by theSertoli cells and the basal lamina of the seminiferous epithelium. It contains thespermatogonia.

E. is correct.

The adluminal compartment is the tissue compartment on the luminal side of theblood-testis barrier formed by the Sertoli cells. In this compartment which is"behind" the blood-testis barrier, one finds the primarily and secondaryspermatozoa, the spermatids and the spermatozoa.

C. is correct.

As spermiogenesis occurs the extra material not utilized in the formation of thesperm, such as tabs of cytoplasm and residual bodies, is phagocytized by theSertoli cells.

B. is correct.

Mitochondria are arranged in a helical arrangement around the axial and coarsefiber bundles in the midpiece of the sperm.

A. is correct.

The head of the sperm is where the chromatin is located. It is highly condensed. Inthis condensed chromatin is the gamete amount of DNA (1C) and the gameteamount of chromosomes, each comprised of one chromatid, or 23chromosomes/chromatids.

A. is correct.

Hyaluronidase is used by the mature sperm to first "digest" its way through thecorona radiata of the ovulated ovum (secondary oocyte), and then through the zona

Page 379: Review Questions for Human Histology

pellucida with the added help of zona lysin which is attached to the innermembrane of the acrosome.

A. is correct.

In the mature sperm, which has completed spermiogenesis, the DNA is completelyheterochromatinized or packaged into a dense, pyknotic mass.

16.041 Where is ATP generated?

A. in the head of the spermatozoon

B. in the midpiece of the spermatozoon

C. in the tail of the spermatozoon

D. none of the above

16.042 Where are 9 peripheral doublets of microtubules & one central pair ofsinglets all surrounded by a dorsal and a ventral column of coarse fibers found?

A. in the head of the spermatozoon

B. in the midpiece of the spermatozoon

C. in the tail of the spermatozoon

D. none of the above

16.043 When do gonocytes or primordial germ cells migrate from yolk sacendoderm into the gonad?

A. during the fetal period

B. at puberty-in general

C. at the time and place of fertilization

D. none of the above

E. during both the fetal period AND at puberty

16.044 When do primary spermatocytes first appear?

A. during the fetal period

B. at puberty-in general

Page 380: Review Questions for Human Histology

C. at the time and place of fertilization

D. none of the above

E. during both the fetal period AND at puberty

16.045 Secondary spermatocyte appears:

A. during fetal period

B. at puberty-in general

C. at the time and place of fertilization

D. none of the above

E. both during fetal period AND at puberty

16.046 Corpora amylacea are found in which of the following organs?

A. testis

B. efferent ductule

C. ductus epididymis

D. prostate gland

E. seminal vesicle

F. ductus deferens

16.047 In this organ huge microvilli are present on a pseudostratified columnarepithelium:

A. testis

B. efferent ductule

C. ductus epididymis

D. prostate gland

E. seminal vesicle

F. ductus deferens

Page 381: Review Questions for Human Histology

B. is correct.

The ATP source in the mature sperm is from the mitochondria congregated in themidpiece of the sperm.

C. is correct.

This is a description of the ultrastructure of the tail of a mature sperm.

A. is correct.

The migration of gonocytes (spermatogonia and oogonia) into the developinggonad occurs during the embryonic/fetal period.

B. is correct.

Primary spermatocytes begin to appear in the seminiferous epithelium at puberty.Prior to then the seminiferous epithelium is comprised of only spermatogonia andSertoli cells.

B. is correct.

Spermatogenesis begins at puberty so secondary spermatocytes can not be foundin the testis prior to puberty. Prior to puberty the seminiferous tubules are lined byonly Sertoli cells and spermatogonia.

D. is correct.

The corpora amylacea are the concretions or precipitated secretory product foundin the glandular alveoli of the prostate gland. The older the gland, the moreconcretions.

C. is correct.

This is a description of the epithelial lining of the ductus epididymis. The largemicrovilli absorb a lot of fluid, thereby establishing a current from testis toepididymis.

16.048 Mucus is produced by which of the following?

A. prostate gland

B. bulbourethral gland

C. seminal vesicle

D. glands of Littre

Page 382: Review Questions for Human Histology

E. more than one of the above

16.049 Which of the following has fructose as an important component of itssecretory product?

A. prostate gland

B. bulbourethral gland

C. seminal vesicle

D. glands of Littre

16.050 Acid phosphatase is an enzyme characteristically found in which of thefollowing?

A. prostate gland

B. bulbourethral gland

C. seminal vesicle

D. glands of Littre

16.051 Concretions are found in which of the following?

A. prostate gland

B. bulbourethral gland

C. seminal vesicle

D. glands of Littre

16.052 Which of the following surrounds the penile urethra?

A. corpora cavemosa

B. corpus spongiosum

C. both corpora cavernosa and corpus spongiosum

D. neither corpora cavernosa nor corpus spongiosum

16.053 Which of the following forms the glans penis?

Page 383: Review Questions for Human Histology

A. corpora cavernosa

B. corpus spongiosum

C. both corpora cavernosa and corpus spongiosum

D. neither corpora cavernosa nor corpus spongiosum

E. penile urethra

16.054 Which of the following are (is) composed of erectile tissue?

A. corpora cavernosa

B. corpus spongiosum

C. both corpora cavernosa and corpus spongiosum

D. neither corpora cavernosa nor corpus spongiosum

E. penile urethra

E. is correct.

Both the bulbourethral glands and the glands of Littre are mucus producers.

C. is correct.

Fructose is a component of the secretory product of the seminal vesicle.

A. is correct.

This enzyme is produced by the prostate gland. Increased levels of this enzyme inthe blood could indicate cancer of the prostate.

A. is correct.

Concretions are found in the prostate gland and they increase with the age of theindividual.

B. is correct.

Since the penile urethra courses through the corpus cavernosum urethra (corpusspongiosum), this mass of erectile tissue surrounds the penile urethra.

B. is correct.

Page 384: Review Questions for Human Histology

The glans penis represents an enlargement of the corpus cavemosum urethra(spongiosum).

C. is correct.

The large, potential vascular spaces which comprise the erectile tissue of the penisare found in both the corpora cavemosae and corpus spongiosum (corpuscavernosum urethra).

16.055 The glands of Littre secrete into the lumen of which of the following?

A. corpora cavernosa

B. corpus spongiosum

C. both corpora cavernosa and corpus spongiosum

D. neither corpora cavernosa nor corpus spongiosum

E. penile urethra

16.056 The lumen/spaces of which of the following is/are lined by pseudostratifiedcolumnar epithelium?

A. corpora cavemosa

B. corpus spongiosum

C. both corpora cavernosa and corpus spongiosum

D. neither corpora cavernosa nor corpus spongiosum

E. penile urethra

16.057 The lumen/spaces of which of the following is/are lined by unfenestratedendothelium?

A. corpora cavernosa

B. corpus spongiosum

C. both corpora cavernosa and corpus spongiosum

D. neither corpora cavernosa nor corpus spongiosum

E. penile urethra

Page 385: Review Questions for Human Histology

16.058 Which of the following forms the prepuce?

A. corpora cavemosa

B. corpus spongiosum

C. both corpora cavernosa and corpus spongiosum

D. neither corpora cavemosa nor corpus spongiosum

E. penile urethra

16.059 When do secondary spermatocytes appear?

A. during the fetal period

B. at puberty-in general

C. at the time and place of fertilization

D. none of the above

E. during both the fetal period AND at puberty

16.060 Which of the following contains a mediastinal area?

A. testis

B. efferent ductule

C. ductus epididymis

D. prostate gland

E. seminal vesicle

F. ductus deferens

E. is correct.

The glands of Littre are mucous glands which occur throughout the length of thepenile urethra. They secrete mucus into the lumen of the penile urethra whichlubricates the penile urethra.

E. is correct.

The majority of the epithelial lining of the penile urethra is of the pseudostratified

Page 386: Review Questions for Human Histology

columnar type. However, occasionally there are patches of simple columnarepithelium and/or stratified columnar epithelium. At the distal end of the penileurethra, the urethra is expanded into a fossa, the fossa navicularis. This region ofthe penile urethra is lined by stratified squamous non-keratinized (wet) epithelium.

C. is correct.

The vascular spaces which characterize the erectile tissue of the corporacavernosae and the corpus spongiosum (corpus cavemosum urethra) are lined byendothelial cells which are of the continuous, not the fenestrated type.

D. is correct.

The prepuce or foreskin is a fold of skin covering the glans penis and it is notderived from any of the structures listed.

B. is correct.

Spermatogenesis begins at puberty so secondary spermatocytes cannot be foundin the testis prior to puberty. Prior to puberty the seminiferous tubules are lined byonly Sertoli cells and spermatogonia.

A. is correct.

Of all the organs listed only the testis contains a mediastinal connective tissuearea. Actually the connective tissue capsule, the tunica albuginea, sweeps into theorgan on the posterior aspect forming a connective tissue area called themediastinum of the testis. This mediastinal area contains the rete testis.Connective tissue septa radiate outward from this mediastinal area forming septawhich demarcate lobules.

16.061 The epithelial lining of which of the following is composed of alternatinggroups of cuboidal cells with motile cilia and cuboidal cells with no cilia?

A. testis

B. efferent ductule

C. ductus epididymis

D. prostate gland

E. seminal vesicle

F. ductus deferens

Page 387: Review Questions for Human Histology

16.062 Some cells found in which of the following contain cytoplasmic crystalloidsof stored steroid hormone?

A. testis

B. efferent ductule

C. ductus epididymis

D. prostate gland

E. seminal vesicle

F. ductus deferens

16.063 In which of the following organs do zonula occludens type junctions form ablood-parenchyma barrier?

A. testis

B. efferent ductule

C. ductus epididymis

D. prostate gland

E. seminal vesicle

F. ductus deferens

16.064 The wall of this organ contains inner and outer longitudinal layers ofsmooth muscle with a thick middle circular layer of s.m.:

A. testis

B. efferent ductule

C. ductus epididymis

D. prostate gland

E. seminal vesicle

F. ductus deferens

16.065 A tunica albuginea is found in this organ:

Page 388: Review Questions for Human Histology

A. testis

B. efferent ductule

C. ductus epididymis

D. prostate gland

E. seminal vesicle

F. ductus deferens

B. is correct.

This irregular epithelial border lining the efferent ductules is quite characteristic ofthese tubules.

A. is correct.

Of all the organs listed only one of them, the testis, is an endocrine organ. In thiscase the steroid hormone testosterone is produced by the interstitial cells of Leydigand these cells actually store testosterone in crystalloid structures in the cytoplasm.

A. is correct.

The blood-testis barrier is formed by the Sertoli cells which are connected to eachother by zonula occludens type junctions. In this way the spenmatogonia areplaced on one side of this barrier and the cells of spermatogenesis are on the"protected" side of the barrier. Thus, tissue fluid from the connective tissue areas atthe periphery of the seminiferous tubules must pass through the cytoplasm of theSertoli cells before it can approach the spermatocytes, spermatids andspermatozoa.

F. is correct.

This description best fits the ductus or vas deferens.

A. is correct.

Of the structures listed only the testis has a thick layer of fibrous connective tissueforming part of the capsular arrangement of the organ. Other organs have tunicaalbuginea, such as the penis and the ovary.

16.066 In this organ concretions in the glandular lumen increase with age:

A. testis

Page 389: Review Questions for Human Histology

B. efferent ductule

C. ductus epididymis

D. prostate gland

E. seminal vesicle

F. ductus deferens

D. is correct.

The prostatic concretions located in the secretory alveoli increase with increasingage of the individual.

Page 390: Review Questions for Human Histology

SECTION 17: FEMALE REPRODUCTIVE SYSTEM

17.001 When do oogonia undergo cell division producing more oogonia, some ofwhich differentiate into primary oocytes?

A. after menopause

B. after menarche up to menopause

C. from birth to menarche

D. from 4-7th month in fetal ovary

E. once/month on day 14 of typical menstrual cycle

17.002 As a pin passes from the nucleus of a developing oocyte into thesurrounding ovarian tissue it would encounter the following layers in what correctanatomical sequence: l=nucleus, 2=theca interna, 3=zona pellucida, 4=vitellinemembrane, 5=follicular or granulosa cells, 6=theca externa.

A. 1-3-213-5-6

B. 1-4-5-2-3-6

C. 1-4-3-5-2-6

D. 1-4-5-3-2-6

E. 1-3-4-2-5-6

17.003 What is the cumulus oophorus?

A. follicle cells lining a primordial follicle

B. follicle cells lining a primary follicle

C. follicle cells lining a preantral follicle

D. follicle cells in a mature follicle not associated with the oocyte

E. follicle cells in a mature follicle which are associated with the oocyte

17.004 Exactly what is released at the moment of ovulation?

A. primordial follicle

Page 391: Review Questions for Human Histology

B. primary oocyte

C. secondary oocyte

D. secondary follicle

E. ootid with 23 chromosomes, lC amount of DNA

17.005 What event causes the secondary oocyte to complete meiosis II (anaphaseII - telophase II)?

A. ovulation

B. menarche

C. menses

D. fertilization

E. formation of liquor folliculi

D. is correct.

Unlike the male in which spermatogonia continuously divide from the time ofpuberty up to perhaps the 9th decade of life, the female is born with all of theprimary oocytes she will ever have and these are in prophase I of meiosis.

C. is correct.

E. is correct.

The cumulus oophorus is the mound of follicle cells associated with the oocyteafter the antrum is formed and begins to fill with follicular liquid.

C. is correct.

The ovulated "egg" is a secondary oocyte in metaphase II of meiosis, encased inits zona pellucida which also encloses the first polar body. Outside the z.p. aresome attached follicle cells, the corona radiata.

D. is correct.

Fertilization is the stimulus for the secondary oocyte to complete meiosis H. If nofertilization occurs the secondary oocyte will die in metaphase II in a matter of afew days.

Page 392: Review Questions for Human Histology

17.006 Immediately after fertilization when the sperm head has just entered thecytoplasm of the secondary oocyte and anaphase II and telophase II have not beencompleted, which of the following situations could apply to the fertilized cell?

A. 3 C amount of DNA

B. 66 autosomal chromatids

C. 3 sex chromatids which can be XXX or XXY

D. A & B are correct

E. A & B & C are correct

17.007 All of the following correctly apply to the corpus luteum EXCEPT:

A. LH is required for its formation

B. luteinization of the granulosa and theca interna cells involves the appearance oflots of rough ER

C. luteinized cells produce progesterone and estrogen

D. rising level of progesterone increasingly inhibits hypophyseal LH production

E. it is formed from the collapsed remains of the ovulated follicle

17.008 The surge or abrupt rise in the blood level of this hormone occurs just priorto ovulation:

A. progesterone

B. FSH

C. estrogen

D. LH

E. HCG

17.009 Arrange the following structures in proper sequence for the transmission ofoxygen from maternal blood to fetal blood across the placental membrane:1=maternal blood, 2=fused basal laminae, 3=cytotrophoblast, 4=endothelialcytoplasm, 5=syncytiotrophoblast, 6=fetal hemoglobin.

A. 1-2-34-5-6

Page 393: Review Questions for Human Histology

B. 1-3-5-2-4-6

C. 1-2-3-5-4-6

D. 1-4-2-3-5-6

E. 1-3-4-5-2-6

17.010 What cells secrete substances which form the zona pellucida?

A. oocyte

B. follicular cells

C. "germinal" epithelial cells

D. A & B are correct

E. A & B & C are correct.

E. is correct.

There is a small window of time during fertilization when the sperm head hasentered and the secondary oocyte has not yet completed anaphase II andtelophase H. The sperm brings in 1 C amount of DNA and 23 chromatids, one ofwhich is either a X or a Y. The metaphase II of the secondary oocyte contains 2Camount of DNA and 23 chromosomes carrying 46 chromatids, two of which are sexchromatids which are X chromatids which will be separated(mitotic disjunction)during anaphase II.

B. is correct.

Luteinization of the granulosa and theca interna cells involves the appearance oflots of not rough but smooth endoplasmic reticulum.

D. is correct.

The preovulatory surge of LH changes the LH:FSH ratio to that which inducesovulation.

B. is correct.

D. is correct.

The zona pellucida is formed by contributions from both the oocyte and thesurrounding follicular cells. Both of these cell types send surface projections intothe zona pellucida where they contact each other.

Page 394: Review Questions for Human Histology

17.011 About how many oogonia are found in each ovary at the end of the 7th fetalmonth?

A. 1 million

B. 3 million

C. 6 million

D. 500,000

E. 500

17.012 In the normal reproductive "life" of the human female from menarche tomenopause, about how many ova are ovulated?

A. 150

B. 450

C. 1000

D. 10,000

E. 100,000

17.013 The corpus luteum of menstruation eventually dies because:

A. HCG is present

B. increasing levels of progesterone increasingly inhibit LH production

C. LH:FSH ratio becomes inappropriate

D. estrogen levels remain steady for too long

E. follicular cells secrete too much androstenedione

17.014 All of the following are true of the smooth muscle cells of the myometriumEXCEPT:

A. rising levels of progesterone cause less contractions

B. rising levels of estrogen cause less contractions

C. relaxin causes less contractions

Page 395: Review Questions for Human Histology

D. capable of hypertrophy

E. capable of hyperplasia

17.015 Which layers of the uterus are "lost" or sloughed at menses?

A. perimetrium

B. inner myometrium

C. stratum basalis of endometrium

D. stratum functionalis of endometrium

E. outer myometrium

17.016 In the pregnant uterus the endometrial stroma contains many decidualcells. What is this tissue layer called when it is in contact with the chorion laeve?

A. amnion

B. decidua basalis

C. decidua parietalis

D. decidua capsularis

E. syncytiotrophoblast

B. is correct.

B. is correct.

Only 450 ova are ovulated; the rest undergo atresia.

B. is correct.

Rising levels of progesterone negatively feedback to the pituitary gland causing adecrease in the production of LH. Decreasing the level of LH, which is essential forthe maintenance of the corpus luteum, causes the death ("suicide") of the corpusluteum.

B. is correct.

Increasing levels of estrogen, as during days 4-14 of the menstrual cycle enhancescontractions of the myometrial smooth muscle.

Page 396: Review Questions for Human Histology

D. is correct.

Most of the endometrium is sloughed during menses. However, some of theendometrium closest to the myometrium is retained, the stratum basalis, from whicha new functionalis will be regenerated.

D. is correct.

As the embryo embeds into the endometrium and forms a chorion around itself thevillous chorion faces the decidua basalis, whereas the smooth chorion (laeve)faces the decidua capsularis. The rest of the endometrium minus the deciduacapsularis and basalis, is the decidua parietalis.

17.017 A uterine "fibroid" is composed of what?

A. smooth muscle cells

B. fibroblasts

C. decidual cells of endometrium

D. A & B are correct

E. A & B & C are correct.

17.018 During days 4-14 the functionalis layer of the endometrium is rebuilt fromthe basalis layer primarily because of the presence of what?

A. FSH

B. estrogen

C. LH

D. progesterone

E. HCG

17.019 What substance causes the corpus luteum of menstruation to become thelonger lived corpus luteum of pregnancy?

A. estrogen

B. progesterone

C. LH

Page 397: Review Questions for Human Histology

D. FSH

E. HCG

17.020 During the proliferative phase of the menstrual cycle the rising levels ofestrogen coming from the group of follicles undergoing maturation causes all of thefollowing EXCEPT?

A. growth of endometrial glands and stroma

B. negative feedback on FSH-RH production

C. decrease in progesterone release

D. positive feedback on LH-RH

E. contraction of smooth muscle in wall of oviduct

17.021 As the fertilized egg begins its trip down the oviduct to the uterine cavity ittraverses the following regions of the oviduct in which order: 1=ampulla,2=isthmus, 3=infundibulum, 4=pars intramuralis.

A. 4-3-2-1

B. 1-2-3-4

C. 3-1-2-4

D. 211-3-1

E. 2-3-1-4

D. is correct.

Fibroids are benign tumors composed of both smooth muscle cells and fibroblastsand therefore are fibromyomas.

B. is correct.

As FSH stimulates a group of follicles to begin growth and maturation the numberof follicular or granulosa cells increases and since these cells manufacture and/orrelease estrogen, the estrogen level rises. The rising estrogen level stimulates theendometrium to repair and regenerate a new functionalis layer.

E. is correct.

HCG or human chorionic gonadotrophin is released by the early trophoblast cells

Page 398: Review Questions for Human Histology

of the embryo. It can be detected in maternal serum as early as 10 days afterfertilization (day 24 of the average menstrual cycle), just in time to convert thecorpus luteum of menstruation into one which will support the pregnancy (corpusluteum of pregnancy).

C. is correct.

Progesterone levels decrease during the late secretory phase due to increasinginhibition of LH-RH.

C. is correct.

17.022 Arrange the following cell types found in the epithelium covering the portiovaginalis of the cervix from basement membrane to vaginal lumen: 1=basementmembrane, 2=precornified, 3=intermediate, 4=basal, 5=comified, 6 parabasal.

A. 1-4-6-3-2-5

B. 1-2-3-4-5-6

C. 1-3-2-6-4-5

D. 1-5-4-3-2-6

E. 1-6-5-4-3-2

17.023 "Epithelial cells accumulate glycogen and slough to the lumen where non-pathogenic bacteria utilize the glycogen to produce lactic acid." Identify the organ?

A. secretory endometrium

B. ampulla of oviduct

C. vagina

D. cervical canal

E. proliferative endometrium

17.024 "During the secretory phase of the menstrual cycle the presence ofestrogen and progesterone causes fat deposition, stromal development and somegrowth of ducts and alveoli." Identify the organ?

A. endometrium

B. vagina

Page 399: Review Questions for Human Histology

C. oviduct

D. breast

E. ovary

17.025 All of the following describe the breast EXCEPT:

A. alveoli basically dormant until pregnancy

B. myoepithelial cells contract in response to oxytocin release

C. during lactation secretion is of the holocrine type

D. secretory product contains caseine, lactoalbumin and in munoglobulins

E. most common malignancy involves the ductal, not the alveolar epithelium

17.026 What causes the preovulatory LH surge?

A. high levels of FSH

B. high levels of estrogen

C. high levels of progesterone

D. high levels of human placental lactogen

E. high levels of oxytocin

A. is correct.

C. is correct.

The lactic acid produced by the bacterial action on the glycogen in the sloughedcells keeps the pH of the vagina low and this is thought to limit the growth ofpathogenic bacteria.

D. is correct.

Thus during the secretory phase of the menstrual cycle the breast enlarges andbecomes slightly more nodular.

C. is correct.

During lactation the alveolar cells secrete by the apocrine method in which a smallportion of the apical part of the cell is part of the secretory product.

Page 400: Review Questions for Human Histology

B. is correct.

As the group of follicles grow in response to FSH the increasing number ofgranulosa cells, which produce estrogen, causes the estrogen level to rise. Thisrising level of estrogen inhibits FSH-RH, but is temporarily a positive feedback forLH.

17.027 In the group of developing oocytes stimulated by FSH all of the followingevents are taking place EXCEPT:

A. synthesis of large quantities of mRNA

B. synthesis of large quantities of rRNA

C. synthesis of large quantities of tRNA

D. synthesis of large numbers of mitochondria

E. synthesis of DNA

17.028 If you extracted DNA from a fully mature oocyte prior to ovulation themajority of that DNA would be derived from?

A. paternal chromosomal complement

B. maternal chromosomal complement

C. mitochondria

D. multiple copies of rRNA genes located in the nucleolar organizer regions

E. genes coding for the large amounts of mRNA characteristic of the oocyte

17.029 Most of the huge quantities of rRNA synthesized by the maturing oocyteassemble into ribosomes which?

A. are active

B. are inactive

17.030 If antibodies are made to the glycoproteins comprising the zona pellucidaand the technique of immunofluorescence is used on maturing oocytes what wouldstain positive?

A. zona pellucida

Page 401: Review Questions for Human Histology

B. cytoplasm of the follicle cells immediately outside zona pellucida

C. cytoplasm of the oocyte

D. A & B are correct

E. A & B & C are correct

17.031 This cell is enclosed in follicle cells which contain an early antrum:

A. primary oocyte

B. secondary oocyte

C. first polar body

D. second polar body

17.032 The nucleus of this cell has 4C amount of DNA:

A. primary oocyte

B. secondary oocyte

C. first polar body

D. second polar body

E. is correct.

DNA synthesis is completed in all oocytes in the 7th fetal month.

C. is correct.

During the maturation of the oocyte so many mitochondria are produced thatmitochondrial DNA can constitute up to 90% of the total DNA content in a matureoocyte.

B. is correct.

These ribosomes remain inactive until after fertilization when they becomeactivated and support the synthesis of proteins necessary for early embryogenesis.

E. is correct.

Both the oocyte and the follicle cells synthesize the glycoproteins found in the zonapellucida.

Page 402: Review Questions for Human Histology

A. is correct.

When an antrum appears in the granulosa cells of a multilaminar primary follicle(containing a primary oocyte), the primary follicle changes its name to a secondaryfollicle.

A. is correct.

The only choice which correctly applies here is the primary oocyte which is lockedin a prolonged prophase I of meiosis. Since it has not started the meiotic process,but has completed DNA synthesis (occurred when the woman was a 5-7 month oldfetus), it contains 4C DNA, 46 chromosomes, 92 chromatids.

17.033 This is the cell formed subsequent to fertilization:

A. primary oocyte

B. secondary oocyte

C. first polar body

D. second polar body

17.034 This cell is enclosed in a single layer of squamosal follicle cells:

A. primary oocyte

B. secondary oocyte

C. first polar body

D. second polar body

17.035 This cell has 92 chromatids in it:

A. primary oocyte

B. secondary oocyte

C. first polar body

D. second polar body

17.036 Which of the following is a large cell formed just prior to ovulation whichcontains 23 chromosomes?

Page 403: Review Questions for Human Histology

A. primary oocyte

B. secondary oocyte

C. first polar body

D. second polar body

E. more than one of the above

17.037 Which of the following is another name for the ovulated oocyte which willbe fertilized?

A. primary oocyte

B. secondary oocyte

C. first polar body

D. second polar body

17.038 This cell is enclosed in a single layer of cuboidal follicle cells:

A. primary oocyte

B. secondary oocyte

C. first polar body

D. second polar body

D. is correct.

Fertilization is the stimulus for the ovulated ovum, which actually is a secondaryoocyte, to undergo the equational division of meiosis (meiosis II). When meiosis IIis completed a second polar body is formed and it would have 23 chromosomes,23 chromatids and 1C amount of DNA. Sometimes the first polar body (23chromosomes, 46 chromatids and 2C DNA) also undergoes meiosis II resulting intwo cells each with the gamete amount of DNA, chromosomes and chromatids.

A. is correct.

This is a description of a primordial follicle containing a primary oocyte. Primordialfollicles are "resting", awaiting their turn to be "selected" by FSH to begin to growand differentiate into primary, then secondary follicles; only one of which will winthe "race" to become a mature Graffian follicle ready for ovulation.

Page 404: Review Questions for Human Histology

A. is correct.

The primary oocyte is in prophase I of meiosis I. Having complete DNA synthesis itcontains 46 chromosomes, but each chromosome is composed of two chromatids,so this cell contains 92 chromatids and 4C amount of DNA.

E. is correct.

Hours before ovulation the primary oocyte completes meiosis I and in doing soforms a large secondary oocyte and a smaller first polar body, both of which arehaploid, having only 23, not 46 chromosomes.

B. is correct.

The first meiotic division or the reductional division occurs hours before ovulation.Therefore, the newly ovulated "ovum" is actually a secondary oocyte.

A. is correct.

A single layer of cuboidal follicle cells describes a unilaminar primary follicle whichcontains a primary oocyte in prophase I of meiosis.

17.039 Which of the following is a large cell with 23 chromosomes, 46 chromatids?

A. primary oocyte

B. secondary oocyte

C. first polar body

D. second polar body

17.040 Endometrial glands are straight and increasingly higher during:

A. menses

B. proliferative phase

C. secretory phase

D. premenstrual phase

17.041 Effects of FSH are evident in the ovary during:

A. menses

B. proliferative phase

Page 405: Review Questions for Human Histology

C. secretory phase

D. premenstrual phase

17.042 Endometrial glands become coiled and filled with glycogen during:

A. menses

B. proliferative phase

C. secretory phase

D. premenstrual phase

17.043 Progesterone blood levels and activity are greatest during:

A. menses

B. proliferative phase

C. secretory phase

D. premenstrual phase

B. is correct.

This description fits the secondary oocyte. It is haploid, having 23, not 46chromosomes. However, each of the 23 chromosomes is composed of twochromatids, thus the reductional division has occurred but the equational divisionhas not yet occurred. The ovum is ovulated as a secondary oocyte and will die inthis stage unless it is fertilized. Thus, fertilization is the stimulus for the secondaryoocyte to complete the second half of meiosis.

B. is correct.

This is histological evidence of an increasing level of estrogen coming from theincreasing numbers of granulosa cells in the developing ovarian follicles. This isthe estrogenic or proliferative or repair phase of the menstrual cycle which is thetime between the end of menses (day 4) to the day of ovulation (day 14).

B. is correct.

FSH activity is detected by seeing several (15 or so) primordial ovarian folliclesdifferentiate into unilaminar and multilaminar primary follicles, each containing aprimary oocyte. This would be observed in the ovary in the early proliferative phaseof the menstrual cycle.

Page 406: Review Questions for Human Histology

C. is correct.

These are the histological and cytological effects of progesterone on an estrogenprimed endometrium. This is characteristic of the secretory phase of the menstrualcycle.

C. is correct.

Progesterone is synthesized from the granulosa lutein cells of the corpus luteum.One can not have a corpus luteum until after ovulation so progesterone levels andactivity characterize the secretory phase of the menstrual cycle.

17.044 During this time period the coiled arteries contract and cause extendedperiods of little to no blood flow to tissue distal to the contraction:

A. menses

B. proliferative phase

C. secretory phase

D. postmenstrual phase

17.045 When does the LH surge occur?

A. menses

B. proliferative phase

C. secretory phase

D. premenstrual phase

17.046 Extravascular blood does not clot in which of the following phases?

A. menses

B. proliferative phase

C. secretory phase

D. postmenstrual phase

17.047 The functionalis layer is lost during which of the following phases?

A. menses

Page 407: Review Questions for Human Histology

B. proliferative phase

C. secretory phase

D. premenstrual phase

17.048 During which of the following phases of the menstrual cycle dodesquamated cells with intracytoplasmic keratin and pyknotic nuclei predominatein a Pap smear?

A. menses

B. proliferative phase

C. secretory phase

D. premenstrual phase

C. is correct.

As the levels of progesterone and estrogen drop due to the degeneration of thecorpus luteum, the coiled arteries contract for hours at a time resulting in ischemiato the endometrial tissue distal to the closed vessel. This occurs late in thesecretory phase or some authors would place this early in the premenstrual phase..After repeated ischemic episodes the anoxic vessel walls first will leak plasma andinterstitial edema occurs in the ovarian stroma. Then the damaged vessel walls willrupture and frank hemorrhage will result.

B. is correct.

The LH surge occurs late in the proliferative phase of the menstrual cycle but justprior to ovulation. In fact the LH surge is required for ovulation. As the estrogenlevel increases during the development of the group of follicles in the ovary, theincreasing estrogen level is a positive feedback on LH production.

A. is correct.

Blood cells outside of the vascular system is hemorrhage. In menses thisextravascular hemorrhage, interestingly enough, does not clot which allows themenstrual blood to "flow". Premenstrual phase probably would be correct alsobecause as the ischemic vessels rupture, blood enters connective tissue areas anddoes not clot.

A. is correct.

The functionalis layer of the endometrium is the upper or luminal 75% of the

Page 408: Review Questions for Human Histology

endometrium and it is this layer which is sloughed off during menses.

B. is correct.

This is characteristic of the estrogenic or proliferative phase of the menstrual cyclein which the estrogen causes more complete differentiation of the surface epithelialcells.

17.049 During this phase of the menstrual cycle parabasal and intermediate cellspredominate in a Pap smear:

A. menses

B. proliferative phase

C. secretory phase

D. premenstrual phase

17.050 This structure has a core of embryonic mesoderm, covered bycytotrophoblast which in turn is covered by syncytiotrophoblast:

A. primary villus

B. secondary villus

C. tertiary villus

17.051 This structure has a core of cytotrophoblast covered by syncytiotrophoblast:

A. primary villus

B. secondary villus

C. tertiary villus

17.052 Hofbauer cells and capillaries are found here:

A. primary villus

B. secondary villus

C. tertiary villus

17.053 Which of the following contain(s) desmosomes?

A. cytotrophoblast

Page 409: Review Questions for Human Histology

B. syncytiotrophoblast

C. both of the above

D. neither of the above

17.054 Which of the following is (are) of maternal origin?

A. cytotrophoblast

B. syncytiotrophoblast

C. both of the above

D. neither of the above

17.055 Which of the following is (are) of fetal origin?

A. cytotrophoblast

B. syncytiotrophoblast

C. both of the above

D. neither of the above

C. is correct.

Estrogen causes the epithelial cells of the cervix and vagina to completelydifferentiate which means they have pyknotic nuclei and the obvious presence ofcytoplasmic keratin. Progesterone does not allow complete differentiation of theepithelial cells and the less differentiated cell types appear in the Pap smear.

B. is correct.

This is the histological appearance of a secondary villus, i.e., extra-embryonicmesoderm has invaded the villus but has not yet differentiated into more matureelements.

A. is correct.

This is the histological appearance of a primary villus which has not yet beeninvaded by extraembryonic mesoderm.

C. is correct.

Page 410: Review Questions for Human Histology

The tertiary villus demonstrates complete differentiation of the embryonicmesenchyme in its core into capillaries, embryonic connective tissue, andembryonic macrophages, the Hofbauer cells.

A. is correct.

Since the cytotrophoblast is composed of individual cells whereas thesyncytiotrophoblast is a true syncytium (a mass of cytoplasm with many nuclei init), it is the cytotrophoblastic cells which are held together with desmosomes.

D. is correct.

The trophoblast is derived from the outer cell mass of the embryo and therefore isnot of maternal, but is of embryonic/fetal origin.

C. is correct.

Both the syncytiotrophoblast and the cytotrophoblast are derived from the moreprimitive trophoblast which is derived from the outer cell mass of the embryo.Therefore both of these trophoblasts are derived from embryonic/fetal origin, notmaternal origin.

17.056 Which of the following is (are) mitotically active?

A. cytotrophoblast

B. syncytiotrophoblast

C. both of the above

D. neither of the above

17.057 Which of the following demonstrate(s) cytological evidence for synthesisand secretion of steroids?

A. cytotrophoblast

B. syncytiotrophoblast

C. both of the above

D. neither of the above

17.058 Many surface microvilli are found on which of the following?

A. cytotrophoblast

Page 411: Review Questions for Human Histology

B. syncytiotrophoblast

C. both of the above

D. neither of the above

17.059 Which of the following become(s) discontinuous in later pregnancy?

A. cytotrophoblast

B. syncytiotrophoblast

C. both of the above

D. neither of the above

17.060 Which of the following produce(s) human placental lactogen?

A. cytotrophoblast

B. syncytiotrophoblast

C. both of the above

D. neither of the above

17.061 Which of the following produce(s) human chorionic gonadotrophin?

A. cytotrophoblast

B. syncytiotrophoblast

C. both of the above

D. neither of the above

17.062 When do primary oocytes first appear?

A. during the fetal period

B. at puberty-in general

C. at the time and place of fertilization

D. none of the above

E. during both the fetal period AND at puberty

Page 412: Review Questions for Human Histology

A. is correct.

The cytotrophoblast is the "stem" cell component of the trophoblast layers of theplacenta. DNA synthesis and mitotic events are found in the cytotrophoblast, not inthe more highly differentiated syncytiotrophoblast.

B. is correct.

The syncytiotrophoblast has the full complement of cytoplasmic organelles for thesynthesis of steroid hormones, namely plenty of smooth endoplasmic reticulum.

B. is correct.

The syncytiotrophoblast is in contact with maternal blood and increases thesurface area for metabolic and gaseous exchange by having microvilli present onits free surface.

A. is correct.

The cytotrophoblast becomes discontinuous as pregnancy proceeds. Thesyncytiotrophoblast remains as a continuous layer, even piling up into syncytialknots here and there.

B. is correct.

The syncytiotrophoblast has the organelles necessary for the production of HPL,whereas the cytotrophoblast does not.

B. is correct.

The syncytiotrophoblast has the ultrastructural machinery for the production ofHCG whereas the cytotrophoblast does not.

A. is correct.

During the embryonic/fetal life of a female, oogonia migrate into the developingovary from the yolk sac. Once in the developing ovary the oogonia differentiate intoprimary oocytes housed in primordial follicles. This is completed by the 7th fetalmonth. The primordial follicles, each containing a primary oocyte in prophase I ofmeiosis I remain dormant until puberty when small groups of them (10-15) arestimulated to grow and develop by FSH.

17.063 When does the very first secondary oocyte appear?

A. during the fetal period

Page 413: Review Questions for Human Histology

B. at puberty-in general

C. at the time and place of fertilization

D. none of the above

E. during both the fetal period AND at puberty

B. is correct.

The ovulated ovum actually is a secondary oocyte so the very first secondaryoocyte would appear at puberty or at the time of the first ovulation and monthly afterthat until menopause. Secondary oocytes also appear at the time of fertilizationbecause the act of fertilization actually is the combining of a mature, haploid spermcontaining I C amount of DNA with a secondary oocyte which is haploid but sinceeach of the 23 chromosomes is composed of 2 chromatids (46 chromatids total) theC amount of DNA in the secondary oocyte is 2C. Fertilization is the stimulus for thesecondary oocyte to complete meiosis II.

Page 414: Review Questions for Human Histology

SECTION 18: EYE AND EAR

18.001 Arrange the following layers in proper sequence for light transmission fromoutside to inside the eye through the cornea: 1=tears, 2=Bowman's membrane,3=Descemet's membrane, 4=stratified squamous epithelium, 5=endothelium,6=substantia propria.

A. 1-4-2-6-3-5

B. 1-213-3-6-5

C. 1-3-6-2-4-5

D. 1-6-4-3-2-5

18.002 Which of the following are members of the vascular middle coat (uvea) ofthe eyeball?

A. choroid

B. ciliary body & ciliary processes

C. iris

D. A & B are correct

E. A & B & C are correct

18.003 Arrange the following in proper sequence for circulation of aqueous humor:1=made on surface of ciliary processes, 2=enters episcleral veins, 3 =pupil,4=anterior chamber, 5=canal of Schlemm, 6=posterior chamber.

A. 1-3-4-5-6-2

B. 1-4-3-6-5-2

C. 1-5-4-6-3-2

D. 1-6-3-4-5-2

18.004 Children born with a deletion located on chromosome # 11 usually havewhich of the following conditions?

A. aniridia

Page 415: Review Questions for Human Histology

B. Wilm's tumor (a form of cancer of the kidney)

C. congenital cataract

D. A & B are correct

E. A & B & C are correct

18.005 Arrange the following structures in correct sequence for light transmissionthrough the retina to the outer segments of the rods and cones: 1=lens, 2=ganglioncell layer, 3=bi-polar neurons, 4=outer plexiform layer, 5=outer nuclear layer,6=layer of optic nerve fibers, 7=stack of membranous discs.

A. 1-2-3-4-5-6-7

B. 1-3-5-4-6-2-7

C. 1-4-3-2-5-6-7

D. 1-5-3-4-2-6-7

E. 1-6-2-34-5-7

A. is correct.

E. is correct.

The uvea is composed of these three subdivisions.

D. is correct.

D. is correct.

Congenital cataract is associated with Rubella infection, not with a deletion onchromosome #I I

E. is correct.

18.006 In a patient with Homer's ptosis (drooping of the upper eyelid) due tocarcinoma of the apex of the lung invading regional tissue, tension is lost in whatstructure?

A. orbicularis oculi

B. Mueller's muscle

C. palpebral conjunctiva

Page 416: Review Questions for Human Histology

D. lacrimal duct

E. tarsal plate

18.007 In which of the following regions would you expect to find photosensitiveretina?

A. innermost layer anterior to the ora serrata

B. innermost layer posterior to the ora serrata

C. innermost layer at the optic disc

D. innermost layer at point of exit of the optic n.

18.008 Where in the eye is the "blind spot" located?

A. retinal extensions anterior to the ora serrata

B. fovea centralis

C. optic disc

D. retinal layer posterior to the ora serrata

18.009 Tenon's space permits the eyeball to move in all directions in response tocontraction of the appropriate extraocular muscles. Between what structures orlayers is Tenon's space located?

A. choroid and suprachoroid layers

B. sclera and suprachoroid layers

C. episclera and the dense connective tissue layer outward from it

D. sclera and episclera

E. dura and pia mater associated with the optic n.

18.010 Where in the eye do collagen fibers run parallel to each other in their ownlayer, but at about 90 degrees to those fibers running parallel in an adjacent layer?

A. sclera

B. episclera

Page 417: Review Questions for Human Histology

C. Tenons capsule

D. choroid

E. substantia propria (stroma) of the cornea

18.011 What two structures join or meet each other at the limbus of the eye?

A. bulbar conjunctiva and palpebral conjunctiva

B. nonphotosensitive and photosensitive retinas

C. ciliary body and ciliary processes

D. iris and ciliary body

E. cornea and sclera

B. is correct.

Mueller's muscle is a few strands of smooth muscle superiorly attached to thesuperior edge of the tarsal plate or the palpebral fascia. Mueller's muscle isinnervated by sympathetic fibers arising from the cervical sympathetic gangliawhich can be destroyed by carcinoma of the apex of the lung. Normalcontraction/tension of Mueller's muscle helps keep the upper eyelid from closing.Loss of this stimulation results in ptosis.

B. is correct.

The photosensitive retina is a multilayered structure which "begins" at the oraserrate as one "proceeds" from anterior to posterior in the eyeball.

C. is correct.

The optic disc is the blind spot because it does not contain the 10 layers typical ofthe photosensitive retina. The optic disc is where the optic nerve exits the eyeballheaded to the brain. The optic nerve is a collection of the axons from the ganglioncells, exiting the eye.

C. is correct.

The external or outer surface of the sclera is called the episclera. It is separatedfrom a dense connective tissue layer peripheral to it (Tenons capsule) by a space,Tenons space.

E. is correct.

Page 418: Review Questions for Human Histology

This is a characteristic of the stroma of the cornea which contributes to the fact thatin the living state it is transparent.

E. is correct.

For choice "A" superior or inferior fornix would apply. For choice "B" ora serratawould apply. Choices "C" and "D" have no official name. The limbus is that circularregion where the cornea meets the sclera.

18.012 Arrange the following layers/structures in proper sequence encountered bya pin as it moves from the center of the eye outward: 1=vitreous, 2=Tenon'scapsule, 3=pigmented layer of retina; 4=choriocapsularis of choroid, 5=Bruch'smembrane, 6=suprachoroidal lamina.

A. 1-3-54-6-2

B. 1-4-3-5-6-2

C. 1-6-4-5-3-2

D. 1-5-6-3-4-2

E. 1-5-4-3-6-2

18.013 Which of the following provides the retina with nourishment?

A. pigmented layer of retina

B. vitreous body

C. suprachoroidal lamina

D. sclera

E. choriocapillary layer of choroid

18.014 As you look at the top of Pike's Peak from the plain below what ishappening in your eyes?

A. lens is stretched thin

B. ciliary muscle is relaxed

C. ciliary muscle is contracted

D. A & B are correct

Page 419: Review Questions for Human Histology

E. A & B & C are correct

18.015 Where in the eye is vitamin A esterified for the rods and cones?

A. choriocapillary layer of choroid

B. outer plexiform layer of retina

C. inner plexiform layer of retina

D. pigmented layer of retina

E. Amacrine cells of retina

18.016 Normally what retains the tear film in the conjunctival sac and prevents itfrom moving anteriorly onto the skin surface of the eyelids?

A. secretion of the glands of Zeiss

B. secretion of the glands of Moll

C. secretion of the meibomian glands

D. eyelashes

E. tarsal plate

A. is correct.

E. is correct.

The outer regions of the retina receive nourishment from the many small vesselsfound in the innermost layer of the choroid. The inner regions of the retina receivenourishment from the retinal blood vessels.

D. is correct.

In accommodation for far or distant vision the ciliary muscle relaxes. Relaxing acircle, the ciliary muscle, increases the tension on the ciliary zonules which in turnstretches the lens thin. In accommodation for near vision such as reading thesequestions, the ciliary muscle contracts. Contracting a circle, takes the tension offthe ciliary zonules and permits the lens to attain its normal shape=thick. Constantcontraction of the ciliary muscle for prolonged periods of time can be detected as"tired eyes". Tired eyes can be relaxed by looking at distant objects, i.e.accommodating for far vision.

D. is correct.

Page 420: Review Questions for Human Histology

One of the functions of the pigmented cells of the outermost layer of the retina is toprepare vitamin A and transport it to the rods and cones. The cells of the pigmentedlayer of the retina also phagocytize and digest the released apical parts of the outersegments of the rods and cones.

C. is correct.

The meibomian glands generate an oily substance which acts as a moisturebarrier to evaporation of the tear film.

18.017 Arrange the following structures in proper anatomical sequence for theproduction and transport of tears: 1=serous acinus, 2=inferior meatus,3=nasolacrimal duct, 4=lacrimal canaliculi, 5=lacrimal sac, 6=lacrimal puncta.

A. 1-3-4-5-6-2

B. 1-4-5-3-6-2

C. 1-5-4-6-3-2

D. 1-4-6-5-3-2

E. 1-6-4-5-3-2

18.018 Place the following structures in proper sequence for the passage of soundwaves into the ear: 1=external auditory canal, 2=incur, 3=foot plate of the stapes,4=tympanic membrane, 5=oval window, 6=malleus.

A. 1-6-4-3-2-5

B. 1-6-2-3-4-5

C. 1-4-6-2-3-5

D. 1-3-6-2-4-5

E. 1-2-3-4-6-5

18.019 A slow change in the position of the head will cause which of the followingto occur?

A. movement of the otoliths

B. movement of the cupulae

C. movement of the basilar membrane

Page 421: Review Questions for Human Histology

D. movement of the tectorial membrane

E. movement of the foot plate of the stapes

18.020 Where do the scala tympani and Scala vestibuli communicate with eachother?

A. inner tunnel

B. outer tunnel

C. endolymphatic duct

D. helicotrema

18.021 A high pitched frequency causes which part of the basilar membrane tovibrate?

A. lower part near round window

B. middle region

C. upper part near helicotrema

18.022 The vestibular membrane moves downward toward the Scala tympani:

A. when the foot plate of the stapes moves inward toward the scala vestibuli

B. when the foot plate of the stapes moves outward into the tympanic cavity

E. is correct.

C. is correct.

A. is correct.

The otoliths in the maculae of the saccule and utricle are heavy enough to bedisplaced by gravity whereas the cupulae of the cristae ampullares, being muchlighter, need more acceleration to move them. Changes in the position of the headare detected by the otoliths displacing the hair cells of the maculae. Changes inangular acceleration are detected by the displacement of the hair cells of thecristae ampullares.

D. is correct.

The scala tympani and the scala vestibuli share a common opening at the top of

Page 422: Review Questions for Human Histology

the modiolus.

A. is correct.

The basilar membrane is "tuned" to be maximally receptive to differences in thefrequency of the vibrations entering the ear. Low pitch sounds cause maximaloscillations of the basilar membrane near the helicotrema.

A. is correct.

18.023 The basilar membrane moves upward toward the scala vestibuli:

A. when the foot plate of the stapes moves inward toward the scala vestibuli

B. when the foot plate of the stapes moves outward into the tympanic cavity

18.024 The covering of the round window bulges outward into the tympanic cavity:

A. when the foot plate of the stapes moves inward toward the Scala vestibuli

B. when the foot plate of the stapes moves outward into the tympanic cavity

18.025 The basilar membrane moves downward toward the scala tympani:

A. when the foot plate of the stapes moves inward toward the scala vestibuli

B. when the foot plate of the stapes moves outward into the tympanic cavity

18.026 The stereocilia on the hair cells in the organ of Corti are bent toward themodiolus:

A. when the foot plate of the stapes moves inward toward the Scala vestibuli

B. when the foot plate of the stapes moves outward into the tympanic cavity

18.027 The stereocilia on the hair cells in the organ of Corti are bent toward thestria vascularis:

A. when the foot plate of the stapes moves inward toward the scala vestibuli

B. when the foot plate of the stapes moves outward into the tympanic cavity

18.028 The hair cells in the organ of Corti release some neurotransmitter whichcauses depolarization of bipolar neurons in the spiral ganglion:

A. when the foot plate of the stapes moves inward toward the scala vestibuli

Page 423: Review Questions for Human Histology

B. when the foot plate of the stapes moves outward into the tympanic cavity

18.029 The posterior capsule of the lens:

A. touches or contains aqueous humor

B. does NOT touch or contain aqueous humor

18.030 Ciliary processes:

A. touch or contains aqueous humor

B. do NOT touch or contain aqueous humor

B. is correct.

A. is correct.

A. is correct.

A. is correct.

B. is correct.

B. is correct.

B. is correct.

The posterior capsule of the lens would face or touch the vitreous body, notaqueous humor.

A. is correct.

The ciliary processes are the site of manufacture for the aqueous humor.

18.031 The inner limiting membrane of the retina:

A. touches or contains aqueous humor

B. does NOT touch or contain aqueous humor

18.032 The canal of Schlemm:

A. touches or contains aqueous humor

B. does NOT touch or contain aqueous humor

Page 424: Review Questions for Human Histology

18.033 The pupil:

A. touches or contains aqueous humor

B. does NOT touch or contain aqueous humor

18.034 The fovea centralis:

A. touches or contains aqueous humor

B. does NOT touch or contain aqueous humor

18.035 The ora serrata:

A. touches or contains aqueous humor

B. does NOT touch or contain aqueous humor

18.036 The endothelium of cornea:

A. touches or contains aqueous humor

B. does NOT touch or contain aqueous humor

18.037 The posterior chamber:

A. touches or contains aqueous humor

B. does NOT touch or contain aqueous humor

18.038 The posterior pigmented layer of the iris (iridial retina):

A. touches or contains aqueous humor

B. does NOT touch or contain aqueous humor

18.039 The anterior capsule of the lens:

A. touches or contains aqueous humor

B. does NOT touch or contain aqueous humor

B. is correct.

The inner most layer of the ten layered retina is its inner limiting membrane. Thispart of the retina therefore touches the vitreous body.

Page 425: Review Questions for Human Histology

A. is correct.

The canal of Schlemm is the drainage route for aqueous humor exiting the anteriorchamber.

A. is correct.

The pupil is the hole through which aqueous humor flows from the posteriorchamber into the anterior chamber.

B. is correct.

The fovea centralis is the region of most acute vision in the retina. It touches themost posterior aspect of the vitreous body.

B. is correct.

The ora serrata is the junction between the 10 layered photosensitive retinaposteriorly and the 2 layered non-photosensitive retina anterior to it, i.e. on thesurface of the ciliary processes and on the posterior side of the iris. Since the oraserrata is posterior to the ciliary process and their zonules, the ora seirata touchesthe vitreous body.

A. is correct.

The most posterior aspect of the cornea is its endothelium. This endotheliumtouches aqueous humor filling the anterior chamber of the eye.

A. is correct.

The posterior chamber of the eye is filled with aqueous humor which is producedby the ciliary processes.

A. is correct.

The posterior pigmented layer of the iris (iridial retina) is one of the anteriorboundaries of the posterior chamber and therefore touches aqueous humor in theposterior chamber.

A. is correct.

The anterior capsule of the lens forms part of the posterior boundary of theposterior chamber of the eye. Since the posterior chamber is filled with aqueoushumor the anterior surface of the lens touches this aqueous humor.

18.040 The blind spot:

Page 426: Review Questions for Human Histology

A. touches or contains aqueous humor

B. does NOT touch or contain aqueous humor

18.041 The pigmented epithelium of the retina:

A. touches or contains aqueous humor

B. does NOT touch or contain aqueous humor

18.042 Contraction of the ciliary smooth muscle occurs if

A. the short ciliary nerves have fired

B. the short ciliary nerves have NOT fired

18.043 Contraction of the sphincter of the pupil occurs if

A. the short ciliary nerves have fired

B. the short ciliar nerves have NOT fired

18.044 Contraction of the dilator of the pupil occurs if

A. the short ciliary nerves have fired

B. the short ciliar nerves have NOT fired

18.045 Tension on the ciliary zonules is increased when:

A. the short ciliary nerves have fired

B. the short ciliary nerves have NOT fired

18.046 The lens becomes stretched and thinner when:

A. the short ciliary nerves have fired

B. the short ciliary nerves have NOT fired

18.047 The lens becomes unstretched and thicker if

A. the short ciliary nerves have fired

B. the short ciliary nerves have NOT fired

B. is correct.

Page 427: Review Questions for Human Histology

The blind spot is the area of the eye where the optic nerve exits the eye on its wayto the brain. There is no photosensitive retina in this region of the optic disc. Theoptic disc or blind spot touches the posterior aspect of the vitreous body, notaqueous humor.

B. is correct.

The pigmented epithelium comprising the outermost layer of the retina is far awayfrom the aqueous humor in the anterior and posterior chambers of the eye.

A. is correct.

The short ciliary nerves innervate the ciliary muscle. So, when the short ciliarynerves fire, the ciliary smooth muscle contracts.

A. is correct.

Since the sphincter of the pupil is innervated by the short ciliary nerves, when theyfire to contract the ciliary body, the sphincter of the pupil also contracts.

B. is correct.

The dilator of the pupil is innervated by sympathetic postganglionic fibers. Sincethe short ciliary nerves are parasympathetic postganglinic fibers, firing of thesenerves would have no effect on the dilator of the pupil. To get a contraction of thedilator smooth muscle, the sympathetic postganglionic fibers innervating the dilatorneed to fire.

B. is correct.

Increasing tension on the ciliary zonules will pull the lens flat. This is accomplishedby relaxing the circular ciliary body.

B. is correct.

By not firing the short ciliary nerves, the circular ciliary body relaxes and thisincreases the diameter of the circle in which the lens is suspended by the ciliaryzonules. This pulls at the edges of the lens and stretches the lens to a thinnershape.

A. is correct.

Firing the short ciliary nerves contracts the circular ciliary body and this releasesthe tension on the lens allowing it to attain its normal "fat" shape.

18.048 Tension on the ciliary zonules is decreased if

Page 428: Review Questions for Human Histology

A. the short ciliary nerves have fired

B. the short ciliary nerves have NOT fired

18.049 The rod is:

A. a first order neuron in the visual-neural pathway

B. a second order neuron in the visual-neural pathway

C. a third order neuron in the visual-neural pathway

18.050 The ganglion cell is:

A. a first order neuron in the visual-neural pathway

B. a second order neuron in the visual-neural pathway

C. a third order neuron in the visual-neural pathway

18.051 The bipolar neuron, the cell body of which is in the inner nuclear layer ofthe retina, is:

A. a first order neuron in the visual-neural pathway

B. a second order neuron in the visual-neural pathway

C. a third order neuron in the visual-neural pathway

18.052 The cone is:

A. a first order neuron in the visual-neural pathway

B. a second order neuron in the visual-neural pathway

C. a third order neuron in the visual-neural pathway

18.053 Which of the following contain(s) rhodopsin?

A. rod

B. cone

C. both of the above

D. neither of the above

Page 429: Review Questions for Human Histology

18.054 Which of the following traverse or pass through the external limitingmembrane of the retina?

A. rod

B. cone

C. both of the above

D. neither of the above

A. is correct.

Contraction of the circular ciliary body muscle will take tension off of the ciliaryzonules. This allows the lens to return to its normal fatter or thicker shape. Thisaccommodation is for near vision, as for reading. Your eyes get tired after a whilebecause of the sustained contraction of the ciliary muscle. To rest your eyes, lookat a distant object. In this accommodation for far vision the ciliary muscle is notcontracted. If you "uncontract" a circle, from which the lens is suspended by theciliary zonules, tension will be placed on the zonules as the circle widens oropens. This tension will pull the lens flat and a flat lens is needed for far vision.

A. is correct.

Rods and cones represent the first order neurons in the visual pathway.

C. is correct.

The ganglion cell is the third order neuron in the visual pathway as it receivessynaptic input from axons of the bipolar neurons.

B. is correct.

The bipolar neurons of the inner nuclear layer receive synaptic input from axons ofthe rods and cones.

A. is correct.

The rod or the cone actually represents the first neuron in the visual pathway.

A. is correct.

The photopigment of the rod is rhodopsin.

C. is correct.

The external limiting membrane is one of the ten layers of the retina. It actually is

Page 430: Review Questions for Human Histology

composed of junctional complexes between cells giving the faint effect of a linewhich can be seen with the LM. Both the rod and the cone "penetrate" the externallimiting membrane.

18.055 Which of the following contain(s) an outer segment?

A. rod

B. cone

C. both of the above

D. neither of the above

18.056 Which of the following contain(s) an inner segment?

A. rod

B. cone

C. both of the above

D. neither of the above

18.057 Iodopsin is found in which of the following?

A. rod

B. cone

C. both of the above

D. neither of the above

18.058 A cilium is found on which of the following?

A. rod

B. cone

C. both of the above

D. neither of the above

18.059 Flattened membranous discs are found in which of the following?

A. rod

Page 431: Review Questions for Human Histology

B. cone

C. both of the above

D. neither of the above

18.060 Flattened membranous discs continuous with or connected to theplasmalemma are found in?

A. rod

B. cone

C. both of the above

D. neither of the above

18.061 Flattened membranous discs which are independent from, or do notcommunicate or connect with, the plasmalemma are characteristic for which of thefollowing?

A. rod

B. cone

C. both of the above

D. neither of the above

C. is convect.

Both the rod and the cone contain outer segments, outward from the innersegments. The outer segments contain stacks of membranous discs which containthe photopigments characteristic of the rod or the cone.

C. is correct.

Both the rod and the cone contain a part of the entire cell referred to as the innersegment. The inner segments contain the regular organelle components such asmitochondria, RER, lysosomes, etc.

B. is correct.

Iodopsin is the cone photopigment, and chemical variations of the iodopsinaccount for the three color classes of cones, red, blue and green. Rhodopsin is thephotopigment of the rod cell.

Page 432: Review Questions for Human Histology

C. is correct.

Both the rod and the cone have a cilium arrangement connecting the inner andouter segments.

C. is correct.

Both the rod and the cone have flattened membranous discs stacked up in theirouter segments.

B. is correct.

Unlike the rod, the flattened membranous discs of the cone, are invaginations ofthe plasmalemma.

A. is correct.

This is an ultrastructural feature which can be used to distinguish between rodsand cones.

18.062 Which of the following is/are concentrated in the fovea centralis?

A. rod

B. cone

C. both rods and cones in equal numbers

18.063 Which of the following is/are fixed postmitotics?

A. rod

B. cone

C. both of the above

D. neither of the above

18.064 Which of the following is (are) reverting postmitotics?

A. rod

B. cone

C. both of the above

D. neither of the above

Page 433: Review Questions for Human Histology

18.065 Which of the following is found in the scala media?

A. endolymph

B. perilymph

C. air

D. both endolymph and perilymph

E. cerebrospinal fluid

18.066 Which of the following is found in the scala tympani?

A. endolymph

B. perilymph

C. air

D. both endolymph and perilymph

E. blood

18.067 Which of the following is found in the scala vestibuli?

A. endolymph

B. perilymph

C. air

D. both endolymph and perilymph

E. blood

18.068 Which of the following is found in the lumen of the auditory tube?

A. endolymph

B. perilymph

C. air

D. both endolymph and perilymph

Page 434: Review Questions for Human Histology

E. cerebrospinal fluid

B. is correct.

The region of most acute vision in the retina is the fovea centralis. It is a rod-freearea. In addition to being composed of only cone cells, the innermost severallayers of the retina are missing in this region giving light a relatively unobstructedpath to the outer segments of the cones.

C. is correct.

Rods and cones are non-replicators like neurons, and therefore are classified asfixed postmitotics from a cell cycle kinetic point of view.

D. is correct.

Rods and cones are very specialized cells which in effect are the first orderneurons in the visual-neural pathway. They are non-replicating cells or fixedpostmitotic cells like all other adult neurons.

A. is correct.

The scala media is another name for the cochlear duct. The cochlear duct is amembranous labyrinth structure and therefore is filled with endolymph.

B. is correct.

B. is correct.

The scala vestibuli is a named space or division of the bony labyrinth which liesoutside of the cochlear duct on the vestibular membrane side of the cochlear duct.

C. is correct.

The lumen of the Eustachian tube contains air, not endolymph or perilymph.

18.069 Which of the following is found inside the bony labyrinth but outside themembranous labyrinth?

A. endolymph

B. perilymph

18.070 Which of the following has a relatively high concentration of potassium?

A. endolymph

Page 435: Review Questions for Human Histology

B. perilymph

18.071 Which of the following contains a relatively high concentration of sodium?

A. endolymph

B. perilymph

18.072 Which of the following "touches" the otoliths of the maculae?

A. endolymph

B. perilymph

C. air

D. both endolymph and perilymph

E. cerebrospinal fluid

18.073 Which of the following "touches" the cupulae of the cristae ampullares?

A. endolymph

B. perilymph

C. air

D. both endolymph and perilymph

E. blood

18.074 Which of the following flows through both the inner and the outer tunnels inorgan of Corti?

A. endolymph

B. perilymph

C. air

D. blood

E. cerebrospinal fluid

18.075 Which of the following "touches" the vestibular membrane?

Page 436: Review Questions for Human Histology

A. endolymph

B. perilymph

C. air

D. both endolymph and perilymph

E. blood

B. is correct.

The bony labyrinth is filled with perilymph. The membranous labyrinth, which issuspended from and lies within the bony labyrinth, is filled with endolymph.

A. is correct.

The endolymph has a high concentration of potassium. Since potassium is "K" it isclose to "E" in the alphabet and that makes it easy to associate potassium withendolymph.

B. is correct.

Perilymph has a high amount of sodium in it. One way to remember if it is theendolymph or the perilymph which has the high sodium concentration is as follows:since the "n" of Na is close to P in the alphabet it is easy to associate the two andremember that perilymph has the Na.

A. is correct.

The maculae of the utricle and saccule are components of the membranouslabyrinth and therefore are filled with endolymph.

A. is correct.

The cristae ampullares are ridges of specialized cells which protrude into theendolymph of the semicircular ducts.

A. is correct.

Since the organ of Corti "sits" on the basilar membrane it "sits" in the endolymph ofthe cochlear duct and this endolymph flows through the various tunnels in theorgan of Corti.

D. is correct.

Since the vestibular membrane "divides" the scala vestibuli from the scala media

Page 437: Review Questions for Human Histology

(cochlear duct) it (the vestibular membrane) is "touched" by both endolymph andperilymph.

18.076 Which of the following fills the semicircular canals?

A. endolymph

B. perilymph applies

C. air

D. blood

E. cerebrospinal fluid

18.077 Which of the following fills the semicircular ducts?

A. endolymph

B. perilymph

C. air

D. blood

E. cerebrospinal fluid

18.078 Which of the following fills the middle ear cavity?

A. endolymph

B. perilymph

C. air

D. blood

E. cerebrospinal fluid

18.079 Which of the following is/are secreted by the stria vascularis?

A. endolymph

B. perilymph

C. neither of the above

Page 438: Review Questions for Human Histology

D. both of the above

B. is correct.

The semicircular canals (not ducts) are components of the bony labyrinth (notmembranous labyrinth) and are filled with perilymph, not endolymph.

A. is correct.

The semicircular ducts represent components of the membranous labyrinth, not thebony labyrinth. The bony labyrinth is filled with perilymph which is outside of thewalls of the membranous labyrinth. It is the structures of the membranous labyrinthwhich are filled with endolymph.

C. is correct.

The middle ear cavity is filled with air and the air pressure in this cavity can beequalized with that of the nasopharynx via the auditory (Eustachian) tube.

A. is correct.

The stria vascularis is a highly vascularized area in the lateral wall of the cochlearduct. Many of these epithelial cells contain basal infoldings and lots ofmitochondria and are believed to be the source of endolymph.